Professions

¡Supera tus tareas y exámenes ahora con Quizwiz!

A. Concealment of Physical Evidence and Documents

1. How should laws balance their duty to protect client confidences against their responsibilities to the system of justice? 2. Duties of Criminal Defense Lawyers with Respect to Evidence of Crimes a) What should a lawyer do when a criminal D hands the lawyer a weapon or other tangible evidence of a crime? (1) Recall that in criminal cases, prosecutors may not use discovery to obtain information from D's. (2) Because it would likely breach the attorney-client privilege and because judges are hesitant to grant the prosecution a search warrant to search the D's lawyer's office, the rules restrict lawyers from hiding evidence of criminal misconduct. (3) Why might a client want to park criminal evidence with the lawyer? The client might think that the lawyer could conceal it. If you could use an attorney as a way to protect criminal evidence of crimes, what might happen to the lawyer, if this could be done legally? If your clients were criminals, what might happen over time? Your law office might have an annex in it where you could hide this evidence. (4) The lawyer cannot make the policeman's job harder! (5) See 3.4(a). Rule 3.4 Fairness to Opposing Party and Counsel A lawyer shall not: (a) unlawfully obstruct another party' s access to evidence or unlawfully alter, destroy or conceal a document or other material having potential evidentiary value. A lawyer shall not counsel or assist another person to do any such act. (a) The thrust of the La. rule is not to alter, hide, etc. ANYTHING that has potential value to a legal proceeding. (b) Notice that the rule only prohibits unlawful concealment or destruction. The ban will only apply where some law makes the concealment/destruction unlawful (like where it would violate a criminal obstruction of justice statute or where it would violate a court order). (c) It may also be unlawful where it would result in the state law tort of spoliation - however this is unclear because tortious conduct may result in the payment of damages, but not criminal prosecution. (d) It is also limited to material having potential evidentiary value. (e) Violations of this rule are often difficult to detect. Even if a prosecutor finds out about it, the lawyer is likely going to assert the attorney-client and work product privileges to avoid revealing information. (f) You have to tell your client that if given to you than have to report. What if they ask how to get rid of evidence? Inform them that is obstruction of justice. (i) You have assisted in fraud or crime if counsel them to obstruct evidence! Attorney client privilege just falls out! (g) Allowed to do reasonable testing on evidence before turning it over. (6) State v. Olwell (Wash. 1964): Lawyers cannot resist a coroner's subpoena - he should turn it over to the prosecution, even at his own initiative. The attorney should not be a depository for criminal evidence, which in itself has little, if any, material value for the purposes of aiding counsel in the preparation of the defense of his client's case. (7) In re Ryder (4th Cir. 1967): Lawyer disbarred where he moved contents of D's safe deposit box into his own in order to keep the D from disposing of it - the court held that his motivation for moving the money was deemed irrelevant because he helped conceal the evidence. The lawyer was suspended from practice in federal court, and the evidence was eventually introduced in the D's case. (8) People v. Meredith (Cal. 1981): Lawyer could have left evidence where D had told him it was and not revealed this fact by using the attorney-client privilege. However, the privilege was partially abrogated when the lawyer's investigator went after the evidence, and therefore the lawyer's act of turning it over to the police and withdrawing from the representation was proper. (9) Green (La.): Someone had killed an LSU student and came to a lawyer. The client gave the lawyer a bag of clothes with the murder weapon in it. What is the obligation of a lawyer in this instance? The lawyer could not hold on to the gun. The general obligation is to turn it over to the police, but the lawyer cannot hold on to it. At what point does the lawyer have to turn it over? It might be able to be kept by the lawyer for a reasonable period of time to see if it has evidentiary value. What if the client wants to throw the gun in the river? You have to tell him no - if you say that this would be a good idea, you are assisting him in the commission of a crime. You don't want to get involved! The lawyer in Green found the gun involuntarily and turned it over to the police. (a) The lawyer's duty stops at turning the gun over - you don't have to say where it came from. This would be whistleblowing on your own client. Can the client be compelled to testify about where the gun came from, if the lawyer turns the gun in? What does the attorney-client privilege protect? Communications. Is the giving of the gun a communication? Recall the LAST LINK doctrine to the attorney client privilege - normally the identity of the client isn't privileged, but the client's identity would be protected here if the lawyer was being compelled to testify about who gave him the gun. (b) The lawyer cannot wipe off the fingerprints on the gun before turning it in, and neither can the client - would be tampering with evidence or obstructing justice. (c) Can you take the gun from your client, tell him you won't turn it over to the client, and then turn it in anyway? The client should probably be informed about what's going on. You should probably tell your client before he hands it over about your duty to hand it over to the authorities. (d) Suppose the client hides the gun, tells you where it is, you go check it out and see that it is in fact in this spot. What do you say when the prosecution asks you where the murder weapon is? Is the communication protected by the attorney-client privilege? These are communications about past crimes, so they're protected. (e) If the client says that he's going to throw it in the river, and you tell him which one to throw it in, then this would be furthering the commission of a future crime and not protected.

Rule 1.2 Scope Of Representation And Allocation Of Authority Between Client And Lawyer

(a) Subject to paragraphs (c) and (d), a lawyer shall abide by a client's decisions concerning the objectives of representation and, as required by Rule 1.4, shall consult with the client as to the means by which they are to be pursued. A lawyer may take such action on behalf of the client as is impliedly authorized to carry out the representation. A lawyer shall abide by a client's decision whether to settle a matter. In a criminal case, the lawyer shall abide by the client's decision, after consultation with the lawyer, as to a plea to be entered, whether to waive jury trial and whether the client will testify. (b) A lawyer's representation of a client, including representation by appointment, does not constitute an endorsement of the client's political, economic, social or moral views or activities. (c) A lawyer may limit the scope of the representation if the limitation is reasonable under the circumstances and the client gives informed consent. (d) A lawyer shall not counsel a client to engage, or assist a client, in conduct that the lawyer knows is criminal or fraudulent, but a lawyer may discuss the legal consequences of any proposed course of conduct with a client and may counsel or assist a client to make a good faith effort to determine the validity, scope, meaning or application of the law. (1) Clients decide the objectives of the representation, and the lawyer must consult the client as to the means used to pursue objectives. This line between objectives and means may not always be very clear. (2) Big decisions that the client has the explicit right to make: (a) Civil: Client decides whether to settle. A lawyer cannot decide against the client's instructions to settle. (b) Criminal: Client decides whether to: (i) Plead guilty; (ii) Waive jury trial; (iii) Testify. (3) Objective or means? (a) The decision to file a lawsuit: This appears to be a means to get relief. Does a lawyer have implied authority to file a lawsuit? (b) Decision to send out a set of interrogatories: Means. (c) Take a deposition: Means. (d) Object to a particular piece of evidence: Means. (e) To offer a settlement: This is unclear. The rules treat it as an objective, but isn't it just a means to receive the client's objective (like filing a lawsuit)? (4) One way lawyers deal with this tension is to visit with the client, tell them what they would like to do, and then ask the client if they are comfortable with the plan. This is good for a first time, unsophisticated client. (5) A lawyer and client may agree that the lawyer will provide less than the full range of services. The client may prefer this arrangement to reduce costs or for other reasons. (6) We've seen 1.2(d) before: Recall that it bars lawyers from advising or assisting clients in illegal or fraudulent activity. b) Recall that the lawyer can take all actions that are impliedly authorized, although we're not really sure what these actions are. Cmt. 2 says that clients normally defer to the special knowledge and skill of their lawyer with respect to the means to be used to accomplish their objectives, particularly with respect to technical, legal, and tactical matters. Lawyers will usually defer to clients with respect to questions such as the expense to be incurred and concern for third persons who might be adversely affected. c) What happens if lawyer and client disagree as to the means to be employed? The Restatement states that except for decisions reserved for clients and in the absence of an agreement on these matters, a lawyer may take "any lawful measure within the scope of representation that is reasonably calculated to advance a client's objective." The Restatement suggests that unless a lawyer and client have agreed otherwise, the lawyer, not the client, should make decisions that "involve technical legal and strategic considerations difficult for a client to assess." (1) Hypo: What if your client refuses to give consent to an extension for your opponent whose mother just died? You're reputation is on the line now, so do you still listen to your client? Under the Restatement standard, the lawyer would be able to consent to the extension regardless of what the client said. d) Jones v. Barnes (1983) (1) The client had a lot of points that he wanted raised at trial, and the lawyer used discretion to discard some of them. The lawyer didn't raise two nonfrivolous claims that the client wanted him to. (2) Issue: Does defense counsel assigned to prosecute an appeal from a criminal conviction have a constitutional duty to raise every nonfrivolous issue requested by the D? (3) Held: The attorney in this dispute supported his client's appeal to the best of his ability. He satisfied his duty to take professional responsibility for the conduct of the case after consulting with the client. Lawyers do not have a constitutional duty to raise every argument that the client suggests. (4) Under principal-agency law, this is the client's case. The constitution says we provide "assistance of counsel" backing up this idea that the client makes the calls. However, the client may not have the overall strategic sense of what to do in an oral argument. The more arguments you have, the more the silly arguments will detract from the strength of your best arguments. This is consistent with rule 1.2 (5) You can reject what your client senses if you have a decent reason for it. (6) Justice Brennan and Marshall's dissent: Since client's bear the consequences of lawyers' decisions, lawyers should defer to their clients' judgments. The client should call the shots (autonomy), and the lawyer should just assist the client in representation (effectiveness). The majority seems to subordinate the client's case to the lawyer's ability to make judgments. (7) The lawyer does not have to follow the client's wishes to introduce non-frivolous appeals (8) Is ineffective assistance to refuse to raise the errors that your client wants? (a) The court says no! (b) Could dilute the ones that could work (9) Autonomy à of the client to control their case (10) Competency à that you are the one with training and the know how (11) The client can always fire you (12) Effectiveness is the big term in this case e) Should deference to client wishes depend on: (1) Whether the case is criminal or civil? While in criminal actions, the client's liberty is at stake, the parties to a civil action can also suffer extreme consequences. (2) Whether the client has enough money to fire you and retain another lawyer? Even if the client doesn't have the money to find another lawyer, it will still be impractical to try to do so because of the time required. (3) Whether the client is wealthy or poor? Would you be more deferential to a client with more money? (4) What if client wants the death penalty, would there be a reason the opposition might file an opposition to the death penalty? Maybe. The court in Bloom, said the attorneys must obey the clients wishes for the death penalty, because they are the clients wishes.

legal fees

1. Lawyer-client fee contracts a) Types of agreements (1) Most common: The lawyer bills for the amount of time the lawyer spends on the matter. (2) Contingency fees are usually charged in personal injury P work, where the lawyer gets a percentage of the damages (if any) that are paid out to the client). (a) Has to be in writing!!!! (3) Recent developments: (a) Flat fees for standard services (b) Schedule of fees (4) Corporate clients have become more concerned about how much is spent on outside counsel fees and will issue policies restricting what outside counsel may bill for. (5) Many clients wind up feeling blind-sided by steep legal bills, because lawyers fails to notify them at the outset as to how much the services may cost (difficult to predict?). (6) Can you bill someone for the time spent billing them?! NO!!!! maybe if you get permission from client but still might not fly. b) Reasonable fees (1) Lawyers generally have wide discretion in deciding how much to charge. (2) See 1.5(a). Rule 1.5 Fees (a) A lawyer shall not make an agreement for, charge, or collect an unreasonable fee or an unreasonable amount for expenses. The factors to be considered in determining the reasonableness of a fee include the following: (1) the time and labor required, the novelty and difficulty of the questions involved, and the skill requisite to perform the legal service properly; (2) the likelihood, if apparent to the client, that the acceptance of the particular employment will preclude other employment by the lawyer; (3) the fee customarily charged in the locality for similar legal services; (4) the amount involved and the results obtained; (5) the time limitations imposed by the client or by the circumstances; (6) the nature and length of the professional relationship with the client; (7) the experience, reputation, and ability of the lawyer or lawyers performing the services; and (8) whether the fee is fixed or contingent. (3) This is a squishy factor test that avoids accuracy. (4) Some lawyers have been disciplined for charging unreasonable fees. In determining the reasonableness of a fee, courts tend to ask whether the fee is comparable to fees charged by other lawyers in similar cases, and whether the fee makes sense in light of the work performed and the results obtained. (5) A fee might be unreasonable if: (a) The lawyer has no records to substantiate it or (b) The fee is disproportionate to the services provided. (6) Much of this question of reasonableness depends on the norms in the legal community in question and in a particular are of practice. (7) In the Matter of Fordham (Mass. 1996) (a) Lawyer charged 5 times more than the usual fee charge for a DWI, but he came up with a very novel argument to get the kid off. The lawyer was not an expert in this field. (b) Held: Fordham's fee was clearly excessive, despite the fact that Fordham worked the hours billed. (i) Since other people in this field only charge $10K max for this kind of work, the fee was unreasonable under 1.5. (ii) "His inexperience in criminal defense work and OUI cases in particular cannot justify the extraordinarily high fee. It cannot be that an inexperienced lawyer is entitled to charge 3 or 4 times as much as an experienced lawyer for the same service." (iii) The lawyer should not have charged for the research time needed to get himself up to speed. (iv) The lawyer was charging, in part, to educate himself, and this is unreasonable. (c) Sometimes a safe harbor is applied to these situations, which will make the fee per se reasonable: Where the lawyer contracts to bill by the hour, the client is aware of the lawyer's hourly fee and honestly and diligently works the actual number of hours billed to the client, then the lawyer will be protected. The court didn't buy into this rule in this case. (d) Note that while the fee is unreasonable, and outcome is not usual. The outcome of the case is one of the factors to be considered.

A. Studying Conflicts of Interest

1. When a lawyer undertakes representation of a client, the lawyer owes that client a duty of loyalty and a duty to protect confidential information. a) It could be disloyal for the lawyer to agree to represent another client whose interest are adverse to the first. b) In the course of representing the second client, the lawyer might use or reveal confidential information learned from the first client. 2. There are three general outcomes when there is a conflict of interest: a) The conflict is so serious that the lawyer must turn down one or both clients. b) Some conflicts will be adequately addressed by obtaining in formed consent from the affected clients. c) The conflict is not serious enough to require informed consent. 3. Other remedies for conflicts may include signing agreements that limit representation and screening. 4. It is not sufficient for a firm to have a good system for reviewing incoming matters to check for conflicts - it must also have a system to monitor for emerging conflicts as the representation is under way. 5. Hypo: Jay comes to you and says he's been in an accident. Sally was the other driver, and he wants to sue her. Sally's insured by State Farm, and so State Farm will also be a D (under La.'s direct action statute). Suppose you keep a computer in your office that keeps track of all your matters. The computer generates hits for every time a particular party shows up that you have done work for before. a) What if a hit shows up that you drafted Sally's will? This is a DIRECT conflict of interest. It is very very bad to sue your own client. However, if Sally is a former client, then it may be ok to sue if her you can obtain informed consent. b) What if your computer says that you are representing Irma in a suit against Jay? This is also bad. c) What if State Farm is your client in other matters? Also a conflict. 6. Change the facts: Suppose you have a partner, Gayle, who is doing the representation of Irma v. Jay. Is there still a problem? YES! You two are in the same law firm and Gayle's conflicted are imputed upon you. Everyone in the firm shares the same conflicts. a) What if you're in a firm with 500 lawyers, across the world? The same analysis applies. The client databases MUST be linked to avoid these conflicts. b) Keep in mind that small firms will not have a sophisticated system to keep tract of conflicts! 7. Categories of conflicts: a) Concurrent conflict: Rules 1.7 & 1.8. Conflict between 2 present obligations of a lawyer or between one present client and one prospective client. Sometimes informed consent will solve, but other times representation is prohibited altogether. 1.7 provides the basic rules and 1.8 provides tailored rules. b) Successive conflict of interest: Rule 1.9. Conflict between an obligation to a present client and an obligation to a former client. Informed consent will usually solve all scenarios. c) Imputed conflict: Rule 1.10. Conflict between an obligation of one lawyer to a client and an obligation of an affiliated lawyer. This is the "typhoid" or "infection" scenarios. An imputed conflict can be waived by a client under the conditions stated in 1.7. d) Conflicts for government lawyers: Rule 1.11. The rule addresses both successive and imputed conflict of interest for lawyers who move between jobs in government and the private sector. Many of these conflicts are resolved by informed consent.

1. Forbidden and restricted fee and expense arrangements

a) Buying legal claims (1) See rule 1.8(i), which bars lawyers from acquiring an interest in litigation on behalf of a client, except for permitted liens and contingent fees. (i) A lawyer shall not acquire a proprietary interest in the cause of action or subject matter of litigation the lawyer is conducting for a client, except that the lawyer may: (1) acquire a lien authorized by law to secure the lawyer's fee or expenses; and (2) contract with a client for a reasonable contingent fee in a civil case. (2) A lawyer may not purchase an interest in a claim on which the lawyer is representing the client. (3) However, if a lawyer purchases a claim from a client, and ceases to represent the client in the collection of the claim, then the transaction is no longer technically barred by 1.8(i). A lawyer may purchase collection claims from a client, so long as the lawyer complies with rule 1.8(a) and (j) - the terms must be reasonable, the client must have the opportunity to seek independent legal advice, the lawyer must disclose all possible conflicts, and the client must consent in writing. b) Financial assistance to a client (1) See 1.8(e). (e) A lawyer shall not provide financial assistance to a client in connection with pending or contemplated litigation, except that: (1) a lawyer may advance court costs and expenses of litigation, the repayment of which may be contingent on the outcome of the matter; and (2) a lawyer representing an indigent client may pay court costs and expenses of litigation on behalf of the client. (a) The goal here is to prevent lawyers from having too big a stake in the outcome of litigation - it might produce an improper level of zeal. (b) A client might also pursue a frivolous suit to obtain the offered financial support. (c) Note that court costs and litigation expenses may be paid by the attorney. (i) If the client is not indigent, the client is ultimately responsible for paying these costs, but they may be taken out of the client's recovery. (ii) If the client is indigent, the lawyer need not attempt to collect these sums from the client. (2) Louisiana has adopted a different rule on financial assistance to clients, which is much more client friendly. (e) A lawyer shall not provide financial assistance to a client in connection with pending or contemplated litigation, except as follows. (1) A lawyer may advance court costs and expenses of litigation, the repayment of which may be contingent on the outcome of the matter, provided that the expenses were reasonably incurred. Court costs and expenses of litigation include, but are not necessarily limited to, filing fees; deposition costs; expert witness fees; transcript costs; witness fees; copy costs; photographic, electronic, or digital evidence production; investigation fees; related travel expenses; litigation related medical expenses; and any other case specific expenses directly related to the representation undertaken, including those set out in Rule 1.8(e)(3). (2) A lawyer representing an indigent client may pay court costs and expenses of litigation on behalf of the client. (3) Overhead costs of a lawyer's practice which are those not incurred by the lawyer solely for the purposes of a particular representation, shall not be passed on to a client. Overhead costs include, but are not necessarily limited to, office rent, utility costs, charges for local telephone service, office supplies, fixed asset expenses, and ordinary secretarial and staff services. With the informed consent of the client, the lawyer may charge as recoverable costs such items as computer legal research charges, long distance telephone expenses, postage charges, copying charges, mileage and outside courier service charges, incurred solely for the purposes of the representation undertaken for that client, provided they are charged at the lawyer's actual, invoiced costs for these expenses. With client consent and where the lawyer's fee is based upon an hourly rate, a reasonable charge for paralegal services may be chargeable to the client. In all other instances, paralegal services shall be considered an overhead cost of the lawyer. (4) In addition to costs of court and expenses of litigation, a lawyer may provide financial assistance to a client who is in necessitous circumstances, subject however to the following restrictions. (i) Upon reasonable inquiry, the lawyer must determine that the client's necessitous circumstances, without minimal financial assistance, would adversely affect the client's ability to initiate and/or maintain the cause for which the lawyer's services were engaged.12 (ii) The advance or loan guarantee, or the offer thereof, shall not be used as an inducement by the lawyer, or anyone acting on the lawyer's behalf, to secure employment. (iii) Neither the lawyer nor anyone acting on the lawyer's behalf may offer to make advances or loan guarantees prior to being hired by a client, and the lawyer shall not publicize nor advertise a willingness to make advances or loan guarantees to clients. (iv) Financial assistance under this rule may provide but shall not exceed that minimum sum necessary to meet the client's, the client's spouse's, and/or dependents' documented obligations for food, shelter, utilities, insurance, non-litigation related medical care and treatment, transportation expenses, education, or other documented expenses necessary for subsistence. (5) Any financial assistance provided by a lawyer to a client, whether for court costs, expenses of litigation, or for necessitous circumstances, shall be subject to the following additional restrictions. (i) Any financial assistance provided directly from the funds of the lawyer to a client shall not bear interest, fees or charges of any nature. (ii) Financial assistance provided by a lawyer to a client may be made using a lawyer's line of credit or loans obtained from financial institutions in which the lawyer has no ownership, control and/or security interest; provided, however, that this prohibition shall not apply to any federally insured bank, savings and loan association, savings bank, or credit union where the lawyer's ownership, control and/or security interest is less than 15%. Where the lawyer uses such loans to provide financial assistance to a client, the lawyer should make reasonable, good faith efforts to procure a favorable interest rate for the client. (iii) Where the lawyer uses a line of credit or loans obtained from financial institutions to provide financial assistance to a client, the lawyer shall not pass on to the client interest charges, including any fees or other charges attendant to such loans, in an amount exceeding the actual charge by the third party lender, or ten percentage points above the bank prime loan rate of interest as reported by the Federal Reserve Board on January 15th of each year in which the loan is outstanding, whichever is less. (iv) A lawyer providing a guarantee or security on a loan made in favor of a client may do so only to the extent that the interest charges, including any fees or other charges attendant to such a loan, do not exceed ten percentage points (10%) above the bank prime loan rate of interest as reported by the Federal Reserve Board on January 15th of each year in which the loan is outstanding. Interest together with other charges attendant to such loans which exceeds this maximum may not be the subject of the lawyer's guarantee or security. (v) The lawyer shall procure the client's written consent to the terms and conditions under which such financial assistance is made. Nothing in this rule shall require client consent in those matters in which a court has certified a class under applicable state or federal law; provided, however, that the court must have accepted and exercised responsibility for making the determination that interest and fees are owed, and that the amount of interest and fees chargeable to the client is fair and reasonable considering the facts and circumstances presented. (vi) In every instance where the client has been provided financial assistance by the lawyer, the full text of this rule shall be provided to the client at the time of execution of any settlement documents, approval of any disbursement sheet as provided for in Rule 1.5, or upon submission of a bill for the lawyer's services. (vii) For purposes of Rule 1.8(e), the term "financial institution" shall include a federally insured financial institution and any of its affiliates, bank, savings and loan, credit union, savings bank, loan or finance company, thrift, and any other business or person that, for a commercial purpose, loans or advances money to attorneys and/or the clients of attorneys for court costs, litigation expenses, or for necessitous circumstances. (a) La.'s rule on this is quite different from the model rule. See part (4): Subject to some restrictions, a lawyer may provide financial assistance to a client who is in necessitous circumstances, subject however to the following restrictions. (b) The Edwin case came down, which held that lawyer could make advances of living expenses to clients. The practice came up because of the situation discussed in the problem below - clients are forced to settle cases quickly because of their personal situation. (c) The La. rule allows the advancement of living and medical expenses, and also allows lawyers to make loans. (d) E(4) is very broad: are transportation and education really necessary for subsistence? (e) There are a few prohibitions: (i) Must be reasonably necessary and imminent circumstances. (ii) The lawyer cannot charge an interest rate higher than what the bank is charging. (f) Note that the La. rule does not take anything away from the model rule, but only adds to it.

1. Conflicts of former government lawyers in private practice

a) In general (1) See rule 1.11(a). (a) Except as law may otherwise expressly permit, a lawyer who has formerly served as a public officer or employee of the government: (1) is subject to Rule 1.9(c); and (2) shall not otherwise represent a client in connection with a matter in which the lawyer participated personally and substantially as a public officer or employee, unless the appropriate government agency gives its informed consent, confirmed in writing, to the representation. (1) The "participated personally and substantially" standard requires no inquiry about the degree of adversity between the new client's interests and the government's interests, unlike rule 1.9 ("same/substantial relationship" and "materially adverse" standard). (2) The 1.11 standard only applies to the same matter as the previous work. Substantially related matter will not create a conflict in this context. (3) Note that consent in these circumstances must be received from the government (or a particular agency) rather than a typical individual or organizational former client. b) What is a "matter"? (1) If the work involves the same matter, then 1.11 might bar the former government lawyer from participation absent agency consent, and might bar the firm from participation, absent screening and notice to the agency. (2) Rule 1.11(e) gives a definition of "matter" for purposes of this rule. (e) As used in this Rule, the term "matter" includes: (1) any judicial or other proceeding, application, request for a ruling or other determination, contract, claim, controversy, investigation, charge, accusation, arrest or other particular matter involving a specific party or parties, and (2) any other matter covered by the conflict of interest rules of the appropriate government agency. (3) The comments offer further explanation: (a) 1.11(a)(2) and (d)(2) disqualify lawyers only from work on matters involving a specific party or parties, rather than extending disqualification to all substantive issues on which the lawyer worked. (b) For purposes of (e), a "matter" may continue in another form. In determining whether two particular matters are the same, the lawyer should consider the extent to which the matters involve the same basic facts, the same or related parties, and the time elapsed. c) Personal and substantial participation (1) The comments to 1.11 do not define this term. (2) The language is, however, borrowed from federal conflict of interest laws: (a) To participate "personally" means directly, and includes the participation of a subordinate when actually directed by the former government employee in the matter. (b) "Substantially" means that the employee's involvement must be of significance to the matter, or form a basis for a reasonable appearance of such significance. It requires more than official responsibility, knowledge, perfunctory involvement, or involvement on an administrative or peripheral issue. (c) It is essential that the participation be related to a "particular matter involving a specific party." (3) There are two basis for disqualification of former government lawyers: (a) 1.11(a): Subsequent work involves a matter in which the government lawyer was personally and substantially involved while in the government (can be cured by consent). (b) 1.11(c): Subsequent work could involve use of confidential government information about a person known to the lawyer in a way that would materially disadvantage that person (cannot be cured by consent). d) Screening of former government lawyers (1) See 1.11(b) as to avoiding imputation of a conflict of a former government lawyer who had been personally and substantially involved in a matter. (b) When a lawyer is disqualified from representation under paragraph (a), no lawyer in a firm with which that lawyer is associated may knowingly undertake or continue representation in such a matter unless: (1) the disqualified lawyer is timely screened from any participation in the matter and is apportioned no part of the fee therefrom; and (2) written notice is promptly given to the appropriate government agency to enable it to ascertain compliance with the provisions of this rule. (1) The former government lawyer's firm must screen him from any contact with the work and see that he earns no part of the fee. (2) Note that the firm need not seek consent from the government agency as a prerequisite to taking on the conflicting work, but must simply notify the government agency about its work on the matter. b) Confidential government information (1) Even if the former government lawyer did not personally and substantially participate in the matter, the lawyer might be precluded because, while in the government she learned "confidential government information" about a person and the new matter could involve material adverse use of that information. (2) See rule 1.11(c). (c) Except as law may otherwise expressly permit, a lawyer having information that the lawyer knows is confidential government information about a person acquired when the lawyer was a public officer or employee, may not represent a private client whose interests are adverse to that person in a matter in which the information could be used to the material disadvantage of that person. As used in this Rule, the term "confidential government information" means information that has been obtained under governmental authority and which, at the time this Rule is applied, the government is prohibited by law from disclosing to the public or has a legal privilege not to disclose and which is not otherwise available to the public. A firm with which that lawyer is associated may undertake or continue representation in the matter only if the disqualified lawyer is timely screened from any participation in the matter and is apportioned no part of the fee therefrom. (1) This rule is here to prevent the use of governmental power in aid of private interests and to avoid giving an unfair advantage to a party whose lawyer has confidential government information about individual citizens. (2) This type of conflict CANNOT be cured by obtaining consent. (3) The former government lawyer cannot take on the work where: (a) He has actually learned information about a person; (b) That was obtained under governmental authority; and, (c) That could be used to the material disadvantage of that person in a new representation. (4) Note the definition in the rule of "confidential government information" and the fact that such conflicts only include information "about a person." (5) The imputation rule for these conflicts is in 1.11(c). The requirements to avoid imputation of this kind of conflict are the same as under 1.11(b), except that the firm need not notify the government agency about its work on the matter.

1. Contingent fees

a) In general (1) Types of contingency arrangements: (a) A percentage of the client's recovery (b) Client pays an hourly fee or a flat fee, and then the lawyer gets an additional fee if a specified result happens (2) These arrangements have historically been closer regulated, due to the concern about lawyers who have a financial stake in their clients' recoveries. (3) Benefits of contingency fee arrangements: (a) Allows access to justice for people who are not wealthy (b) The use of contingent fee usually aligns the interests of lawyer and client (unlike hourly rates) (4) Despite the more recent acceptance of contingency fee agreements, the rules impose more specific disclosure requirements. (5) Rule 1.5 requires that a contingent fee be in writing. (6) Hypo: Suppose you look at a couple of documents and an accident report in a personal injury case. You fax the report to State Farm, and they offer $40K to settle the claim. You've only spent 2 hours on it. The client asks you whether it should be accepted. If you take the typical 1/3 of this on a contingency basis, then you'll make much more than you would if you were billing by the hour. The incentive for the lawyer is to settle right away, after not doing much work, while it might be in the better interest of the client to litigate the claim. (7) See 1.5(c). (c) A fee may be contingent on the outcome of the matter for which the service is rendered, except in a matter in which a contingent fee is prohibited by paragraph (d) or other law. A contingent fee agreement shall be in a writing signed by the client and shall state the method by which the fee is to be determined, including the percentage or percentages that shall accrue to the lawyer in the event of settlement, trial or appeal; litigation and other expenses to be deducted from the recovery; and whether such expenses are to be deducted before or after the contingent fee is calculated. The agreement must clearly notify the client of any expenses for which the client will be liable whether or not the client is the prevailing party. Upon conclusion of a contingent fee matter, the lawyer shall provide the client with a written statement stating the outcome of the matter and, if there is a recovery, showing the remittance to the client and the method of its determination. (a) Note the reference to 1.5(d): This provision bars contingent fees in many criminal and domestic relations cases. (b) Recall that all legal fees must be reasonable - this includes contingent fees! (c) The agreement must specify the percentage of the recovery to be earned by the lawyer and indicate whether the percentage to be charged depends on how the case proceeds. (d) The agreement must explain whether expenses are to be deducted from the total settlement or judgment before or after the lawyer's fee is calculated. (i) Note that the rule does not require lawyers to deduct expenses before calculating their fees or to offer clients an option to do so. (ii) Generally, if the expenses are deducted from the total recovery before the lawyer's fee is calculated, the client's recovery will be more (and the lawyer's fee will be less). (e) The agreement needs to explain which expenses the client must pay even if she gets no recovery. (f) After the matter has been concluded, the lawyer must provide the client with another writing explaining what fee and expenses were charged and how they were calculated. (8) Large contingency fee arrangements are often justified because of: (a) The large risk that the lawyer takes in agreeing to represent the P, in that the lawyer may invest substantial resources in the representation and not receive anything. (b) It is valuable for P's to have access to contingency fee lawyers since so many people could not afford to hire lawyers otherwise. b) Criminal and domestic relations cases (1) See 1.5(d). (d) A lawyer shall not enter into an arrangement for, charge, or collect: (1) any fee in a domestic relations matter, the payment or amount of which is contingent upon the securing of a divorce or upon the amount of alimony or support, or property settlement in lieu thereof; or (2) a contingent fee for representing a defendant in a criminal case. (2) Bar on contingency fee arrangements for domestic relations cases: (a) Lawyers should not have a financial stake in their clients' ultimate divorce. The public policy concern is domestic harmony. We want marriages to last. (b) Traditionally, lawyers were obligated to make efforts to encourage a divorcing couple to reconcile. If the lawyer's fee was contingent upon a divorce, the lawyer would be motivated to encourage the dispute. (c) After the divorce has been granted, a lawyer may charge a contingent fee to collect past due alimony or child support. (3) Bar on contingency fee arrangements for criminal cases: (a) The obvious concern is that in criminal cases, there is no settlement, no recovery. Any fee paid to the lawyer is necessarily going to come out of the criminal D's pocket. You have to rely on the good graces of the murderer you're representing to pay you - like that's going to happen! (b) The lawyer may have perverse incentives, like in domestic relations cases. (c) Also clients facing criminal charges may be vulnerable and may accept fee agreements that overcompensate the lawyer. Because a contingent fee is uncertain, a client might agree to a high contingent fee, doubting that it will ever come due. (d) For criminal defense cases, the lawyer will usually get a flat fee from the D upfront.

A. The Elements of the Attorney-Client Privilege

1. The following elements must be present for the privilege to be evoked: a) "Communication" b) between Privileged Persons c) Communication in confidence (the client must reasonably believe that the communication is confidential) d) Communication for the purpose of seeking legal assistance

the law governing lawyers advisory ethics opinions

1. Written by committees (of lawyers and non-lawyers) formed by the ABA, state bar associations, etc. These committees are separate from the Office of Disciplinary Counsel.

Typhoid Mary hypos from class: Apply 1.9(b) and 1.10 and (class handout problems.)

(1) Associate worked at Old Firm, but then moved to New Firm. One of Old's clients is Toyota. At New, the associate is asked to sue Toyota. The associate never did any work for Toyota at the Old Firm, and he was never in a managerial position. Is the associate "infected"? (a) The lawyer will only be infected where he knows something about Toyota that is confidential and material to the matter. (b) The associate becomes clean of the infection because he had no opportunity to acquire confidential information since he didn't work on Toyota matters at Old Firm. (2) Associate did do work for Toyota at Old. While it is clear that the associate could not take on the matter at New, could his co-associate, Sally? Is Sally infected because the associate is affected? (a) Under 1.10, no one at the New Firm could sue Toyota without informed consent. (b) Once the infected lawyer moves, we engage 1.10 to determine the outcome. (c) Screening? The model rules don't allow screening here, but the state rules may. If you are able to get informed consent, screening might be part of the consent deal. (3) What if moving lawyer (before moving to the new firm) wanted to represent Graphite while he was still at Old Firm? No, of course not. He is "imputed" with confidential information since he's currently working for the adverse firm. But once he moves, the imputation stays behind... (4) Associate did not do any work for Toyota at Old, but was asked to do a little research on a Toyota case at one point. The research was on the summary judgment procedure under state law. At New firm, the associate is asked to sue Toyota in the same matter. (a) Since this is the same matter, you must ask whether the information that the associate knows is materially adverse. (b) Didn't associate represent Toyota under 1.9(a)? This rule only requires the same matter for a conflict. It seems like that if associate was infected under 1.9(a), then he should infect the whole New Firm. However, this is not the case. For a moving lawyer story, we pretend 1.9(a) isn't there! Only look at 1.9(b) for people that move. There is some tension in the commentary about this. (5) At the Old Firm, associate didn't do any work for Toyota, but associate had lunch in the same room as attorneys that were extensively discussing the Toyota case. Is there a problem if associate moves to New Firm and is asked to sue Toyota? (a) YES. He is Typhoid Mary because he acquired information that was protected under 1.6. The rule doesn't say that you have to actually work on a case in order to be infected. (b) What if it was a paralegal in this situation rather than a lawyer? Cmt. 4 to 1.10 says that the rule in (a) does not prohibit representation by others in the law firm where the person prohibited from involvement in a matter is a non-lawyer. Such persons, however, must be screened from any personal participation in the matter to avoid communication to others in the firm of confidential information. (c) Keep in mind that La. has not adopted the comments to the model rules - we don't know where the LASC would come out on this issue. Other jurisdictions have disallowed the use of a screen in these situations for secretaries. (6) Suppose it is now a partner that moves firms, and when partner moves to New Firm, Toyota wants all the work to come with him. The only other person that worked on the Toyota matter was an associate that comes with the partner to New Firm. Right after partner leaves Old Firm, a lady approaches Old about suing Toyota, but it is in a totally different matter. Can the Old firm do so? (7) The 1.7(b) requirements if met will solve a lot of conflicts problems, or else we jump through the new 1.10 hoops to "quarantine the typhoid"

a) Restatement: §46

(2) On request, a lawyer must allow a client or former client to inspect and copy any document possessed by the lawyer relating to the representation, unless substantial grounds exist to refuse. (3) Unless a client or former client consent to non-delivery or substantial ground exist for refusing to make delivery, a lawyer must deliver to the client or former client, at an appropriate time and in any event promptly after the representation ends, such originals and copies of other documents possessed by the lawyer relating to the representation as the client or former client reasonably needs. (1) Retaining lien: Under the cmt. to this rule, certain types of documents that may be withheld form a client. (a) Certain law firm documents reasonably intended only for internal review (b) The lawyer's duty to inform the client may require the lawyer to disclose matters discussed in the document even when the document itself need not be disclosed. (2) The lawyer may retain these documents until the client pays up, unless retention would unreasonably harm the client. (3) In La., the retaining lien idea is a CML doctrine that comes out of the case law. We do not have a statute authorizing this. Under our law, if the client would be prejudiced, then you have to give up the lien. Basically, the lien loses out as soon as it starts to have effect. The fact that we even recognize this CML doctrine in La. is probably jarring for pure civilians. See La.'s rule 1.16(d): (d) Upon termination of representation, a lawyer shall take steps to the extent reasonably practicable to protect a client's interests, such as giving reasonable notice to the client, allowing time for employment of other counsel, surrendering papers and property to which the client is entitled and refunding any advance payment of fee or expense that has not been earned or incurred. Upon written request by the client, the lawyer shall promptly release to the client or the client's new lawyer the entire file relating to the matter. The lawyer may retain a copy of the file but shall not condition release over issues relating to the expense of copying the file or for any other reason. The responsibility for the cost of copying shall be determined in an appropriate proceeding. (a) This appears to say that you cannot withhold the client's files for ANY reason. (b) What is "the entire file"? (i) Tex. has looked at this same issue, and has held that every scrap of paper in the file must be handed over. (ii) S doesn't think this is sensible. (iii) Utah follows the Restatement rule. (iv) We don't know what "entire file" means in La.

Rule 1.16 Declining Or Terminating Representation

(a) Except as stated in paragraph (c), a lawyer shall not represent a client or, where representation has commenced, shall withdraw from the representation of a client if: (1) the representation will result in violation of the rules of professional conduct or other law; (2) the lawyer's physical or mental condition materially impairs the lawyer's ability to represent the client; or (3) the lawyer is discharged. (b) Except as stated in paragraph (c), a lawyer may withdraw from representing a client if: (1) withdrawal can be accomplished without material adverse effect on the interests of the client; (2) the client persists in a course of action involving the lawyer's services that the lawyer reasonably believes is criminal or fraudulent; (3) the client has used the lawyer's services to perpetrate a crime or fraud; (4) the client insists upon taking action that the lawyer considers repugnant or with which the lawyer has a fundamental disagreement; (5) the client fails substantially to fulfill an obligation to the lawyer regarding the lawyer's services and has been given reasonable warning that the lawyer will withdraw unless the obligation is fulfilled; (6) the representation will result in an unreasonable financial burden on the lawyer or has been rendered unreasonably difficult by the client; or (7) other good cause for withdrawal exists. (c) A lawyer must comply with applicable law requiring notice to or permission of a tribunal when terminating a representation. When ordered to do so by a tribunal, a lawyer shall continue representation notwithstanding good cause for terminating the representation. (d) Upon termination of representation, a lawyer shall take steps to the extent reasonably practicable to protect a client's interests, such as giving reasonable notice to the client, allowing time for employment of other counsel, surrendering papers and property to which the client is entitled and refunding any advance payment of fee or expense that has not been earned or incurred. The lawyer may retain papers relating to the client to the extent permitted by other law. (1) Note that the only limitation on the lawyer's ability to fire you is when you are appointed by the court. The general rule is that the client may fire you for any reason. (2) What if you wish to end the relationship? What is "other good cause"? What if you just want to fire your client because he's a jerk? You can withdraw at will as long as it does not materially affect the client. (3) Note part (a): There are certain situations where you MUST withdraw: (a) Violation of law (b) Violation of a rule (c) Client fires you (d) Mental/physical condition prevents you from representing your client with zeal. (4) Part (b) describes situations in which you may withdraw. There are two extremely broad situations: (a) Where you have good cause (even if it would cause a material adverse effect to your client!) and (b) If there is no material adverse effect on the interests of the client. (5) (c): You must comply with the court rules about notice. You must stay with the client if the court says so. (6) (d): See the La. provision below. La.'s rule is similar to this, but has a more extensive second sentence. (7) The entire files have to be turned over? TX says so (8) In Utah they said not everything, the internal work product does not have to be turned over b/c its was not the client's papers (9) LA Rule 1.16 Declining or Terminating Representation (10) Lien à keep the clients papers for security until paid (a) Some jurisdictions allow (b) However if it would cause prejudice to the client than you have to turn over whether paid or not (c) Common law retaining lien (d) LA CoA decisions recognize this, but not LASC case says this

Investigation Before Filing a Complaint

1. Issue: Does the lawyer for the party initiating the proceedings have enough of a factual and legal basis to justify starting a case that will impose burdens of time and expense on others? While lawyers are not required to file only "sure winners," they also cannot file frivolous lawsuits. 2. Required Investigation by Lawyers Filing Civil Cases a) See R. 3.1, which prohibits the filing of "frivolous" claims without defining the term. Rule 3.1: Meritorious Claims and Contentions A lawyer shall not bring or defend a proceeding, or assert or controvert an issue therein, unless there is a basis in law and fact for doing so that is not frivolous, which includes a good faith argument for an extension, modification or reversal of existing law. A lawyer for the defendant in a criminal proceeding, or the respondent in a proceeding that could result in incarceration, may nevertheless so defend the proceeding as to require that every element of the case be established. b) How much more than mere speculation is required? See cmt. 2: The facts need not be "fully substantiated" before suit is filed. The lawyer may need to use discovery to "develop vital evidence." On the other hand, lawyers must "inform themselves about the facts of their clients' cases and the applicable law and determine that they can make good faith arguments in support of their clients' positions." c) Appears to set up a lower bar for criminal matters, since there is usually a lot more at stake there. You can assert a not guilty claim even where you know your criminal client is guilty - this will not be a violation of the rule. d) You can let him plead not guilty even if your know they are guilty. e) Federal Rule of Civil Procedure 11(b): Similar to R. 3.1, but more detailed. By presenting to the court a pleading, written motion, or other paper — whether by signing, filing, submitting, or later advocating it — an attorney or unrepresented party certifies that to the best of the person's knowledge, information, and belief, formed after an inquiry reasonable under the circumstances: (1) it is not being presented for any improper purpose, such as to harass, cause unnecessary delay, or needlessly increase the cost of litigation; (2) the claims, defenses, and other legal contentions are warranted by existing law or by a nonfrivolous argument for extending, modifying, or reversing existing law or for establishing new law; (3) the factual contentions have evidentiary support or, if specifically so identified, will likely have evidentiary support after a reasonable opportunity for further investigation or discovery; and (4) the denials of factual contentions are warranted on the evidence or, if specifically so identified, are reasonably based on belief or a lack of information. (1) This standard is subject to widely ranging interpretations. (a) Legal theory: Must be warranted by existing law or by a nonfrivolous argument for the extension, modification or reversal of existing law or the establishment of new law. (b) Factual assertions: Must have evidentiary support, or if specifically so identified, be likely to have evidentiary support after a reasonable opportunity for further investigation or discovery. (2) What are the differences between R. 3.1 and Federal Rule 11? (a) Sanctions: (i) 3.1: Bar disciplinary action against the attorney (ii) 11: Punished by the judge in the civil action; may result in nonmonetary directives or monetary sanctions against the lawyer or a party (b) Safe harbor: (i) 3.1: No safe harbor, although bar counsel would be unlikely to file a charge against a lawyer for filing a frivolous case or defense that the lawyer withdrew pursuant to the safe harbor provision of Federal Rule 11. (ii) 11: If opposing party makes a motion that 11 has been filed, the lawyer may withdraw the allegedly frivolous pleading within 21 days after the motion and suffer no sanction other than paying atty fees for filing the motion. (3) What penalties may apply to lawyers who bring unsubstantiated suits under Federal Rule 11? (a) A lawyer who initiates a federal claim in good faith, and later finds out that the suit is groundless, may still be subject to sanctions. (b) May have to pay the other party's atty fees. (c) In some states, a D who has been sued on the basis of virtually no evidence may sue the P or the P's lawyer for the tort of malicious prosecution. This may require a higher burden of proof than for granting sanctions under Federal Rule 11. The elements of this claim usually include: (i) D won the previous suit (ii) Suit was without probable cause (iii) Suit was brought with malice (iv) D was injured despite having won (v) (Sometimes) Special injury (4) CCP Art. 863: By signing a document or pleading submitted to the court, the lawyer certifies that, after reasonable inquiry, that the claim does not violate the equivalent requirements of rule 11. Art. 863. Signing of pleadings, effect A. Every pleading of a party represented by an attorney shall be signed by at least one attorney of record in his individual name, whose address shall be stated. A party who is not represented by an attorney shall sign his pleading and state his address. B. Pleadings need not be verified or accompanied by affidavit or certificate, except as otherwise provided by law, but the signature of an attorney or party shall constitute a certification by him that he has read the pleading; that to the best of his knowledge, information, and belief formed after reasonable inquiry it is well grounded in fact; that it is warranted by existing law or a good faith argument for the extension, modification, or reversal of existing law; and that it is not interposed for any improper purpose, such as to harass or to cause unnecessary delay or needless increase in the cost of litigation.

Factors considered in Aggravation [get worse penalty]

1.) Discipline History 2.) Motive of dishonest or selfish 3.) Multiple offenses 4.) Obstruction of the disciplinary process 5.) Present false evidence 6.) Was the victim vulnerable 7.) Refuse to acknowledge wrongful nature 8.) Experience in the practice of the law

Dividing fees with other firms or with non-lawyers

a) Division of fees between lawyers not in the same firm (1) If two lawyers in different law firms work on a case, both should be paid for their services. (a) With hourly fees, it is easy to calculate which lawyer is entitled to be paid what amount. (b) If its a contingency fee agreement, the allocation of the fee is unclear. (2) See rule 1.5(e), which allows the lawyers to share a fee, but imposes conditions to protect clients. (e) A division of a fee between lawyers who are not in the same firm may be made only if: (1) the division is in proportion to the services performed by each lawyer or each lawyer assumes joint responsibility for the representation; (2) the client agrees to the arrangement, including the share each lawyer will receive, and the agreement is confirmed in writing; and (3) the total fee is reasonable. (3) A referral fee can only be collected where the lawyers form a joint venture: (a) The referring lawyer takes on financial and ethical responsibility for the representation as if the lawyers were associated in a partnership (share responsibility in the event of discipline), and (b) The proposed share that each lawyer receives must be disclosed to and approved by the client, in writing. Note that the fee must always be REASONABLE! (4) If a fee splitting agreement is unenforceable, then a lawyer may still recover a share of the fees on a quantum meruit basis. (5) Hypo: He refers a case to you. You, being very grateful and wanting him to refer more cases to you, give him a referral fee of $1K. Under 7.2 (an advertising rule) however, you cannot pay for the recommendation. 1.5(e) might work in this instance though, but the referring lawyer is going to have to assume responsibility for your malpractice like he would if he were a partner. (6) LA Rule 1.5(e) (a) Little more specificity à requires that each lawyer render meaningful legal services for the client in the matter. b) Sharing fees with non-lawyers (1) You can't do it. Lawyers are NOT allowed to share legal fees with non-lawyers, except in very narrow circumstances (like where a partner dies and you distribute his fee earnings to his estate). (2) See rule 5.4(a). (a) A lawyer or law firm shall not share legal fees with a non-lawyer, except that: (1) an agreement by a lawyer with the lawyer's firm, partner, or associate may provide for the payment of money, over a reasonable period of time after the lawyer's death, to the lawyer's estate or to one or more specified persons; (2) a lawyer who purchases the practice of a deceased, disabled, or disappeared lawyer may, pursuant to the provisions of Rule 1.17, pay to the estate or other representative of that lawyer the agreed-upon purchase price; (3) a lawyer or law firm may include non-lawyer employees in a compensation or retirement plan, even though the plan is based in whole or in part on a profit-sharing arrangement; and (4) a lawyer may share court-awarded legal fees with a nonprofit organization that employed, retained or recommended employment of the lawyer in the matter. (3) The purpose of the rule is to protect the lawyer's independent judgment from being affected by influence or pressure from non-lawyers. (4) Law firms may pay salaries and bonuses to non-lawyer employees but may not routinely divide profits among lawyers and non-lawyers. (5) Cannot share fees with summer clerk, secretary or paralegal What about "runners"? (a) In most states, lawyers may not pay referral fees to non-lawyers who send them clients. (b) Some personal injury lawyers send runners to the scene of an accident or the emergency room to hand out flyers about the P's attorney - some of them has been disbarred. (c) In La., the Office of Disciplinary Counsel has been very vigilant about this. (i) Sting program

1. Revolving Door Problems (class)

a) Suppose that A works for the IRS. At the IRS, A helps plan the legal strategy against ABC Corp., which has claimed some questionable deductions and credits amounting to over $160K. Shortly after doing the work on the IRS strategy, A joins law firm X, Y & Z. It turns out that this firm represents ABC Corp. in tax matters, and the firm is engaged in negotiations with the IRS about the corporation's tax situation. The firm asks A to take the lead in the negotiations. (1) Prohibited under 1.11(a) (2) He is disqualified UNLESS the government gives consent. b) Same situation. But this time, the firm concludes A cannot be involved in the matter. The firm decides to keep A totally screened off from the representation, but it continues to represent ABC Corp. against the IRS. (1) Under 1.11(b), A must be screened and given written notice to the government. (2) Note that consent from the government is not needed. (3) Screening will solve the problem. (4) Note this is only the firm which is no longer disqualified (A still is!) c) Suppose that lawyer Jackson used to work as a federal prosecutor. While in that capacity, Jackson investigated CX Computers Inc. for a possible antitrust violation, and learned some facts that would tend to support an antitrust claim. Now in private practice, Jackson brings a civil antitrust action against CX on behalf of Western Electronics, a competitor. Jackson finds that the knowledge he gained in government service is helpful against CX. (1) This is an 1.11(c) problem, and Jackson can be screened to solve the problem. (2) What about Rule 1.6? Duty v. Bar. This is a use situation. (3) What about we switch this hypo so that A movies from CX to GOV. Well gov is not a firm. d) Suppose that we have the same facts, but this time Jackson is not at all involved in the litigation. Jackson's law firm screens Jackson out of the litigation against CX. (1) Screening will solve the problem. No notice (or consent) is required here. (2) Why no notice? Well not protecting the government's interest here, protecting CX. And we don't give notice to the opposing party! CX will never know. e) Suppose that we are back in the situation with A, the IRS, and ABC Corp. A is a government lawyer who represents the IRS. X, Y & Z law firm is on the other side. A does a good job for the IRS, and the firm lawyer ho are working on the matter are impressed by A and his abilities. So, during a break in the negotiations, one of the firm's lawyers tells A that the firm would like to hire him when the ABC matter has been concluded. A is interested, and over the next several days, works out income and other details of his future employment with the law firm. (1) This is a problem under 1.11(d)(2)(ii): A lawyer currently serving as a public officer or EE shall not negotiate for private employment with any person who is involved as a party or as lawyer for a party in a matter in which the lawyer is participating personally and substantially. (2) Should A be reported under 8.4? Perhaps not because he hasn't technically violated the rule yet, since it won't be negotiated until after the matter has been completed. (a) Haven't really looked at this part of the rule before in great detail (3) There might be a problem under 1.7(a)(2) - A might pull punches to get the job - the lawyer's personal interests are divergent from his client's. f) B, a lawyer in a very small town, has now been elected mayor. One of the first matters that comes to him concerns whether or not the town should build a new schoolhouse on land that a corporation desires to use for a parking facility. While in private practice, B did work for the corp., and lobbied the then mayor not to have the town use the land for a schoolhouse. B is uncomfortable about deciding the issue as mayor. But the town is so small, and there are only two officials - the mayor and the chief of police. And the chief of police is concerned only with crime and crime prevention. (1) This is a 1.11(d)(2)(i) problem: A lawyer currently serving as a public officer or EE shall not participate in a matter in which the lawyer participated personally and substantially while in private practice or nongovernmental employment, unless the appropriate government agency gives its informed consent, confirmed in writing. (2) The government agency in this instance must give informed consent. The problem is that B is the only one that can give consent. The other possibilities might be going to the city council or at least talking with the chief of police. 1.7 would probably not create a conflict here because as mayor, B is not acting as a lawyer.

a) Suing one client on behalf of another client

(1) If the two clients are aligned directly against each other in the same litigation, then the conflict is nonconsentable. (2) A lawyer may not represent opposing parties in the same litigation, regardless of the clients' consent.

"Advance payment of fees" and "nonrefundable retainers"

(1) Retainer: Paying a portion of the lawyer's fee for a specified service before the service is performed. (a) These are usually paid where the client asks for work to be done over a period of time. (b) If the client pays a retainer at the outset, the lawyer has some protection against the claim if he becomes unwilling/unable to pay. (c) This advance is deposited in the lawyer's client trust account, and portions are withdrawn as they are earned. (d) Any unearned portions of the advance must be returned back to the client. (e) If the advance payment is exhausted, then the lawyer usually continues to do work, billing the client periodically. (2) Lump sum payment: Secures the lawyer's availability for a period of time or for a particular task. This is considered to be earned by the lawyer when it is received, since it is a promise for future services. (3) May a lawyer require a nonrefundable advance payment from an individual client with whom the lawyer has no prior relationship? Maybe, although public policy strongly discourages it. It is argued that this locks the client into the relationship and tends to constrain the client from firing the lawyer where necessary. (a) There was a case out of NY that upheld a nonrefundable retainer of $1M because the client knew about it from the beginning and agreed to pay it. (b) Nonrefundable retainers are rarely needed, but that are used to retain the availability of lawyers in the firm - the lawyers have to turn down other opportunities because of this arrangement. (c) These agreements compromise the client's ability to fire the lawyer. (d) Distinguish among two types of retainers: (i) Advance fee payment (refundable) (ii) True retainer (general availability) (nonrefundable) La. rule 1.5(f) elaborates more on this: (f) Payment of fees in advance of services shall be subject to the following rules: (1) When the client pays the lawyer a fee to retain the lawyer's general availability to the client and the fee is not related to a particular representation, the funds become the property of the lawyer when paid and may be placed in the lawyer's operating account. (2) When the client pays the lawyer all or part of a fixed fee or of a minimum fee for particular representation with services to be rendered in the future, the funds become the property of the lawyer when paid, subject to the provisions of Rule 1.5(f)(5). Such funds need not be placed in the lawyer's trust account, but may be placed in the lawyer's operating account. (3) When the client pays the lawyer an advance deposit against fees which are to accrue in the future on an hourly or other agreed basis, the funds remain the property of the client and must be placed in the lawyer's trust account. The lawyer may transfer these funds as fees are earned from the trust account to the operating account, without further authorization from the client for each transfer, but must render a periodic accounting for these funds as is reasonable under the circumstances. (4) When the client pays the lawyer an advance deposit to be used for costs and expenses, the funds remain the property of the client and must be placed in the lawyer's trust account. The lawyer may expend these funds as costs and expenses accrue, without further authorization from the client for each expenditure, but must render a periodic accounting for these funds as is reasonable under the circumstances. (5) When the client pays the lawyer a fixed fee, a minimum fee or a fee drawn from an advanced deposit, and a fee dispute arises between the lawyer and the client, either during the course of the representation or at the termination of the representation, the lawyer shall immediately refund to the client the unearned portion of such fee, if any. If the lawyer and the client disagree on the unearned portion of such fee, the lawyer shall immediately refund to the client the amount, if any, that they agree has not been earned, and the lawyer shall deposit into a trust account an amount representing the portion reasonably in dispute. The lawyer shall hold such disputed funds in trust until the dispute is resolved, but the lawyer shall not do so to coerce the client into accepting the lawyer's contentions. As to any fee dispute, the lawyer should suggest a means for prompt resolution such as mediation or arbitration, including arbitration with the Louisiana State Bar Association Fee Dispute Program.

Particular Applications of R. 1.9

1. Suing former clients a) A lawyer, on behalf of a new client, may sue a former client, without the former client's consent, in a matter that is not substantially related to the previous representation. No consent is required unless the new matter is the same or substantially related, and the new representation is materially adverse to the interests of the former client. b) Although a lawyer is permitted to sue a former client on an unrelated matter, it may not always be a good idea. 2. Representing the competitor of a former client a) Cmt. 6 to rule 1.7: Ordinarily, representation of economic competitors poses no serious conflict of interest. If this is not a concurrent conflict, then one would think it would be less problematic in the successive conflict context. b) However, if the firm learned a great deal about the operation of the former client, and that information could be used on behalf of a competitor to the disadvantage of that former client, the firm could have a serious conflict! c) Maritrans GP, Inc. v. Pepper, Hamilton & Scheetz (Pa. 1992) (1) The firm (Pepper) knew a lot about how Maritrans did business through its representation of the corporation. Maritrans sues to enjoin Pepper from representing its competitors, as Pepper has a lot of information about the corporation that it might share with its competitors. At the time this suit is brought, Maritrans is no longer a client of Pepper. (2) Held: There was a substantial relationship between Pepper's former representation of Maritrans and their current representation of Maritrans' competitors, as Pepper's involvement was extensive as was their knowledge of sensitive information provided to them by Maritrans. In general, the relationship between the attorney and his client is a fiduciary relationship, and the concept of fiduciary relation by definition does not permit conflicts of interest. A fiduciary who breaches his duty of loyalty to his principal is liable to his principal, and an injunction is a proper remedy for the breach. (3) Moral of the story: It might be risky to take on competitors of a client with whom you have a close economic relationship. d) Chinese wall: Lawyers in the same firm that are working on conflicting matters will erect a "wall" between them and not share information. This is also called cone of silence or more frequently screening. Must get informed consent from the former client in order to use this method. This is specifically allowed by rules 1.11 (revolving door) and 1.18 (prospective clients), and may be allowed under the state rules. (THIS YEAR: 1.10a allows for Chinese walls and screens) e) Walker v. State Dept. of Trans. and Development (La. 2002) (1) Facts: Lawyer that represented DTD in a case where the P alleged that the road was poorly designed and hence caused the accident. He represented many such suits for the state. He left the DTD to go to private practice. At the firm, he was given a personal injury matter (bad roads case) where he had to sue the DTD on a claim similar to that which he used to defend against. (2) Is this the same matter? No - it will never be the same accident under these facts. (3) Substantially related? Yes - While representing the DTD, he could have obtained information that would have harmed the department. (4) The court gives a narrow test for "substantially related": So interrelated in fact and substance that a reasonable person would not be able to disassociate the two matters. (5) Does this really help us? The test laid out here is quite different from that employed for the model rule. There will always be some argument as to how "substantially related" applies and what it means - it is a necessarily vague test. S doesn't think the test laid out by the LASC is very helpful - it seems to just be restating the "same matter" test! This court seems to decide something contrary to the comments to 1.9

Client Protection Funds

This is a fund in each state for which money is set aside to help clients recover if their lawyers steal their money. Mal practice does not help you for intentional acts OR theft. We have on in LA to help but likely not that helpful. There is a 25,000 cap.

A. Truth and Falsity in Litigation

1. Once a case has been filed, lawyers are bound by court and ethical rules to be honest with the tribunal. 2. The Rules on Candor to Tribunals a) R. 3.3 Rule 3.3 Candor Toward the Tribunal (a) A lawyer shall not knowingly: (1) make a false statement of fact or law to a tribunal or fail to correct a false statement of material fact or law previously made to the tribunal by the lawyer; (2) fail to disclose to the tribunal legal authority in the controlling jurisdiction known to the lawyer to be directly adverse to the position of the client and not disclosed by opposing counsel; or (3) offer evidence that the lawyer knows to be false. If a lawyer, the lawyer's client, or a witness called by the lawyer, has offered material evidence and the lawyer comes to know of its falsity, the lawyer shall take reasonable remedial measures, including, if necessary, disclosure to the tribunal. A lawyer may refuse to offer evidence, other than the testimony of a defendant in a criminal matter, that the lawyer reasonably believes is false. (one of the few places where 1.6 gets crushed) (b) A lawyer who represents a client in an adjudicative proceeding and who knows that a person intends to engage, is engaging or has engaged in criminal or fraudulent conduct related to the proceeding shall take reasonable remedial measures, including, if necessary, disclosure to the tribunal. (c) The duties stated in paragraphs (a) and (b) continue to the conclusion of the proceeding, and apply even if compliance requires disclosure of information otherwise protected by Rule 1.6. (d) In an ex parte proceeding, a lawyer shall inform the tribunal of all material facts known to the lawyer that will enable the tribunal to make an informed decision, whether or not the facts are adverse. (1) Bars false statements to courts by lawyers themselves, as opposed to false testimony by clients. If the lawyer discovers he made a false statement, he must correct it. (2) Lawyers must affirmatively disclose directly adverse law in the controlling jurisdiction if the opponent doesn't do so. (3) If the lawyer knows his client or other witness is going to lie, he may not allow the witness to do so. If the witness does lie, the lawyer must call on the witness to correct the lie, and if he won't, then the lawyer must disclose the lie. This applies to trial testimony, depositions and other testimony related to adjudication. (a) "Knowingly" = not just subjective certainty, but there is an objective element as well (b) Note that this duty to correct only applies to false statements of material fact or law. (4) If a lawyer reasonably believes that evidence is false, the lawyer may refuse to offer the evidence. Exception: In criminal cases, the lawyer must allow the D to testify if the lawyer reasonably believes the evidence is false. (5) Lawyers have a duty to prevent false testimony, and also "criminal or fraudulent conduct" in connection with a case before a tribunal. (a) If you know your client wants to present a false alibi, and you as the lawyer knows that the alibi is going to lie, but has not testified yet, then you can probably prevent him from testifying (the constitution only requires that you let the criminal D testify). (b) The information that you have, that the alibi is going to lie, wouldn't that be 1.6 confidential information? How can this be disclosed then? 3.3(c) basically says that it trumps 1.6. We've seen 1.6 crush other rules, like the whistle blowing rule. Here, we have a rule that crushes 1.6, and that is something to note. (c) If the false testimony by the alibi is not material, then there is no duty to correct or disclose it. (6) Unless the case has been completed, if a lawyer learns that a witness gave false testimony, the lawyer must take steps to correct the record. This overrides the duty of confidentiality. (7) Where only one side makes a presentation to the court, the lawyer has a duty to tell the court about adverse facts AND law. b) R. 8.4(c) Rule 8.4 Misconduct It is professional misconduct for a lawyer to: (c) engage in conduct involving dishonesty, fraud, deceit or misrepresentation. (1) This applies to all conduct by lawyers. (2) The ban on deceit and misrepresentation may be broader than the R. 3.3(a)(1) ban on false statements.

1. Fee disputes

a) Prospective limitations of lawyer' liability and settlement of claims against lawyers (1) Can a lawyer ask each client to agree in advance not to sue the lawyer for malpractice, no matter what? Rule 1.8(h) says YES, but only if each client has independent counsel in making the K. (2) Scope of work under Rule 1.2 à but cannot upfront limit the mal-practice liability (h) A lawyer shall not: (1) make an agreement prospectively limiting the lawyer's liability to a client for malpractice unless the client is independently represented in making the agreement; or (2) settle a claim or potential claim for such liability with an unrepresented client or former client unless that person is advised in writing of the desirability of seeking and is given a reasonable opportunity to seek the advice of independent legal counsel in connection therewith. (3) What about settling a malpractice claim? (a) If a lawyer makes a mistake that might cause her to be liable in malpractice, she will usually contact her malpractice insurance co. and/or notify the client. (b) A lawyer might settle this malpractice claim with a client who does not have independent legal advice, but the lawyer must advise the client in writing that it is a good idea to get advice from another lawyer before making such a settlement. (c) The lawyer must give the client a chance to consult with another lawyer. (d) This is a conflict of interest rule because the lawyer would be very tempted to low-ball the amount of the malpractice settlement. This is why outside independent counsel. (4) Lawyers may also organize their firm to limit personal liability for the misconduct of others in the firm. As an LLP, partners are liable for their own conduct and for that of those he supervises, but is otherwise not vicariously liable for the conduct of his partners. b) Fee arbitration (1) May bar associations have established committees to intermediate between lawyers and clients who have disputes over fees. (2) Comment. to rule 1.5: If a jurisdiction has set a mandatory mediation or arbitration process for resolution of fee disputes, a lawyer must comply with it. (3) Many lawyer include clauses in their retainer agreements with clients in which both parties agree to go to binding arbitration in the event of a fee dispute. Such an agreement will be proper where the client is advised of the advantages and disadvantages of arbitration, the client gives informed consent and the provision does not insulate the lawyer from liability that might otherwise by imposed by law. c) Collection of fees (1) What if your client doesn't pay up? (a) Lawyer may contact the client to request payment (b) Lawyer may file suit against the client to collect the fee (but the client may counter with malpractice) (c) May use a collection agency or hire another lawyer to collect the fee (d) May withhold documents prepared by the lawyer for which payment has not been received, but only if doing so will not unreasonably harm the client. (2) Lawyers are subject to the Fair Debt Collection Practices Act if they regularly engage in consumer debt collection activities. (3) Several state consumer statutes apply to lawyers and forbid various unfair and abusive practices in fee collection. (4) Rule 1.8(i) allows a lawyer to "acquire a lien authorized by law to secure the lawyer's fee or expenses." Each state has law that authorizes a lien in this context - by statute, by CML or by K. (a) Such a lien, acquired by contract, constitutes a business transaction with a client and is covered by rule 1.8(a) - the client is entitled to fair terms, a clear explanation of the terms in writing, and written encouragement and opportunity to seek the advice of an independent lawyer. (b) Recall the special rule in La. about liens: If the client makes special written demand for the file, then the papers must be handed over. No retaining lien can be accomplished where the client asks for the file in the right way. (a) Recall 1.8(i): A lawyer may not acquire a propriety interest in the cause of action or subject matter of litigation the lawyer is conducting for a client. Obviously, there are a few exceptions in these cases about liens, contingency fees, etc. § In Saucier the LASC said 1.8(i) and §218 were in conflict, due to the language "of a suit." They said the ethics rules have the force and effect of law, and will trump legislative pronouncements. The court interpreted §218 just to say that the lawyer is acquiring a lien in the settlement proceeds. The statute has not been amended, despite this decision. o A security interest- a lien; they don't look at it like it is property interest o Do not read the words literally b) Fees owed to a lawyer who withdraws or is fired before the matter is completed (1) Whether the lawyer is entitled to payment for the work done before withdrawal or dismissal depends on the reason for the relationship's severance. (2) Where the lawyer is justified for withdrawing under 1.16(b), the lawyer would be entitled to compensation on a quantum meruit basis (equitable assessment of the value of the work done). (3) If the lawyer withdraws without good cause, then the lawyer is breaching the K and may forfeit her right to recover any fees for the work done. (4) If a client fires his lawyer in a contingent fee case, the lawyer may get a quantum meruit fee award, assuming that the services provided are deemed to have some value. (5) Saucier: A is retained and fired without cause. Client then retains B, and B sees the case to its conclusion. (a) How should the fee be divided between A and B? (b) There are a few options: (i) 1/3 to A, 1/3 to B, 1/3 to client? (ii) 1/3 to B and quantum meruit to A? (c) The LASC rejected both. They concluded that only one contingency fee should be paid here, and that fee should be allocated between the attorneys, apportioned based on the 1.5 factors. (i) Relevant contributions of each lawyer

a) 7-3: Police Brutality, Scene 1

(1) Police officers assaulted an arrestee in a bathroom. You represent two of the cops who have been charged with the crime, but who have not confessed to anything. The Policemen's Benevolent Association is paying the legal fees. (2) Under 1.8(f), if you are going to allow a third party to pay the legal fees, then the client must give (1)informed consent and (2)the third party must not interfere with the relationship and (3)must not violate 1.6 (confidentiality) (3) Apart from the PBA paying the legal fees, there could be a conflict just by representing the two cops that are charged with the same crime. b) 7-4: Police Brutality, Scene 2 (1) Gutman was the officer that committed the assault. He is also an officer of the PBA. The PBA offers you a $10M contract to represent Gutman. (2) In this instance, the PBA's interests are likely going to conflict with the lawyer-client relationship, since the PBA will want you to point the finger at a lower-ranked officer.

"Screening"

(1) 1.0(k): Screening refers to isolating a lawyer from any participation in a matter through procedures . . . reasonably adequate . . . to protect information that the isolated lawyer is obligated to protect. (2) Screening is generally not permitted as a remedy for conflicts between: (a) Two present clients or (b) A present and former client of the same firm. (3) Screening is permitted in some states and by some federal courts as a remedy for imputed conflicts in the private sector. (4) There was a proposal to amend rule 1.10 to include language allowing screening as a remedy in this context, but it was rejected by about 45 votes. Several states, however, have amended their rules to take account of changes in the model rules that were adopted in respond to the Ethics 2000 Commission also have decided, notwithstanding the ABA's rejection of the device, to permit screening in this situation.

a) Honesty and communication under the ethics rules

(1) 3.3(a) specifically directs lawyers not to lie to tribunals or to persons other than clients. (2) Strangely enough, the portion of the rules dealing with the Client-Lawyer Relationship does not explicitly require lawyers to be honest with their clients. The reason is probably because lawyers are assumed to be truthful with their clients, and would probably rather lie to someone else on their client's behalf. (3) The drafters make it clear that lawyers should generally be truthful, and this intention extends to clients as well as others. (4) Recall rule 8.4(c) that prohibits lawyers from engaging in conduct involving dishonesty, fraud, deceit, or misrepresentation. Lies may not be fraud, depending on the state law applied, but it may amount to deceit or misrepresentation, such that it could be disciplined under 8.4(c). (5) Rule 1.4 requires that a lawyer shall provide information to a client about: (a) Matters that require informed consent, (b) Matters which a client must make a decision, (c) The status of a matter, and (d) Matters on which the client has requested information.

Rule 1.7 Conflict Of Interest: Current Clients a) In general

(1) A concurrent conflict of interest is one between two current obligations of the lawyer. (2) There are two types of conflicts that might preclude representation of a client because of a concurrent conflict: (a) "Directly adverse" = A lawyer is acting directly against the interests of one of his own clients. This is a direct concurrent conflict. (b) "Significant risk of material limitation" = Representation will still be prohibited without direct adversary if there is a significant "likelihood that a difference in interests will eventuate and, if it does, [that] it will materially interfere with the lawyer's independent professional judgment." This is an indirect concurrent conflict. (3) Exception to the general rule against representation, if all of the following are met: (a) If the lawyer reasonably concludes that he will be able to provide competent and diligent representation; (b) The representation is not prohibited by law; (c) The parties are not adverse in the same matter; and, (d) Informed consent in writing is received from each affected client. b) Direct adversity (1) There will be direct adversity where the lawyer's conduct on behalf of one client requires the lawyer to act against the interest of another current client. (2) Note that both litigation and nonlitigation situations can involve direct adversity. c) Material limitation (1) This where there is still a concurrent conflict despite a lack of direct adversity. The other responsibilities that might materially limit a lawyer's representation of a client include obligations to: (a) Another client; (b) A former client; (c) Someone else to whom the lawyer owes a duty; (d) Someone other than the client who is paying the lawyer's fee; (e) The lawyer's own financial, employment, personal, or other interests. (2) Pulling punches: If a client would receive less vigorous representation from a lawyer because of a lawyer's other responsibilities, there might be a "material limitation" conflict. (3) Note that the mere possibility of harm is insufficient to present a conflict. There must be a significant risk. Evaluate the probability of the conflict occurring with the magnitude of the interference to determine the significance of the risk. Ex. Say your representing Jones, suing Sears, but my firm is at the same time has been trying to entice Sears to choose my firm for it's representation. If the atty eases up on Sears, then the atty is "pulling his punches."

Civil liability for dishonesty to clients

(1) A dissatisfied client may sue the lawyer in tort for fraud or for breach of fiduciary duties. (2) To fulfill fiduciary duties, under the Restatement, a lawyer must: "comply with obligations concerning the client's confidences and property, avoid impermissible conflicting interests, deal honestly with the client, and not employ advantages arising from the client-lawyer relationship in a manner adverse to the client." (3) This duty will be breached where the lawyer fails to act consistently with the trust that a client reposes in a lawyer because the lawyer has special skills and knowledge. (4) Most cases of breach of fiduciary duty arise from disloyalty, or in other words, undisclosed conflicts of interest. This is treated as a constructive fraud, but the breach can arise from actual fraud as well. (5) An attorney who "willfully and deliberately lied to, misled, and stole from his clients," could be sued for punitive as well as compensatory damages, although punitive damages are not available for negligence-based malpractice claims.

a) Publication rights (media rights)

(1) Can lawyers agree to take on a matter in exchange for a promise that when it is all over, you will have exclusive rights to publish a book/movie about the case? Rule 1.8(d) says no. (d) Prior to the conclusion of representation of a client, a lawyer shall not make or negotiate an agreement giving the lawyer literary or media rights to a portrayal or account based in substantial part on information relating to the representation. (2) The problem is that if the case settles quickly, no one will want to read the book. On the other hand, the trial might be widely publicized and make a bestseller. A lawyer in such a situation might do things that would be bad for the client and good for the book. The lawyer might be inclined to draw the case out to make it a bigger show, rather than settling it at an appropriate time. (3) This prohibition only stands until the case is over. At this point, you and your client can agree to forgive all or part of the debt in exchange for transfer of literary or media rights. (4) This rule applies only to the story of the representation. IT does not restrict lawyers representing clients in book or movie K's where the book or movie is not about a case handled by the lawyer. Such an agreement would, however, be subject to 1.8(a), so the lawyer would have to comply with disclosure requirements. (5) Note that the rule only prohibits the giving of media and publication rights to the lawyer. Under 1.8(d), the client could technically sign over the rights to the lawyer's son who is a writer. However, 1.7(a)(2) might apply - the lawyer might still pull his punches or be overzealous to make a better book. A lawyer may also not "negotiate" an agreement of this type, so if the lawyer is just using his son as a stand0in for his own interest, that will also not work. (6) Maxwell (CA): This prohibition can be waived with informed consent, if the lawyers seeking the rights disclose in the K that their interest in the publication may be detrimental to the case. The court said that the disclosure is fine and that informed consent was given - a valid waiver of the prohibition occurred. Note that there is NO indication of a waiver under the ABA version.

a) 4-9: The Candid Notes

(1) Facts: Lawyers instruct paralegals to take down candid notes on their impressions of clients. Representation ended prematurely because client ran out of money. She requests a copy of her "entire file" for a cheaper lawyer. Found a memo inside that describes paralegal's impressions of client, and it isn't very nice. (2) Issue: Should you send the memo to your former client? If you must, can you redact the paralegal's impressions? (3) Response: While the Restatement requires a lawyer to allow a client to have a copy of documents relating to representation, this may fall into an exception: (a) Substantial grounds to refuse? Yes. Not only would this offend the client, but its also imperative that the business judgment behind memos such as this remains confidential within the firm. If word got out that the firm engaged in this, it may have trouble getting clients. (b) Exception for law firm documents? Although this is labeled as a "memo" it is more like the law firm document reasonably intended only for internal review as described in the comment to the Restatement. There is a need here for the lawyers/paralegals to be able to set down their thoughts privately to ensure adequate/efficient representation. May have to disclose matters discussed, but that could be sugar-coated. (c) Redact? This might work, but the former client or her new lawyer might want to know what was redacted. This might draw more attention to the offensive statements.

a) 6-3: A Dysfunctional Family Business

(1) Her Inc. v. Her Fashions. Two brothers who split off into different businesses are fighting over the trademark of "Her." The lawyer had previously represented both of them when they were both under the same trademark. The lawyer is now just representing "Her Fashions." (2) Under 1.9(c), because the lawyer formerly represented the entity that split up into two, he is privy to information that might be used against the adverse party. (3) Same matter? Defending a trademark may or may not be the same thing as acquiring a trademark. There may be something about the trademark acquiring process that could be used to hurt the adverse party. (4) Substantially related? If the lawyer was in a position to gain confidential information that could be used against the former client, then he cannot use that information under 1.9(c) and must get informed consent from the opposing party before continuing. (In LA, standard is whether a reasonable person could reasonably associate the 2.)

a) Modification of fee agreements

(1) If a lawyer represents a client over a long period of time, and the regular hourly rate increases, may the lawyer simply begin billing an existing client at the increased rate? (2) Under contract law, this would be a modification of an ongoing contract. An agreement modifying the initial contract is enforceable if it is fair and equitable in view of circumstances not anticipated by the parties when the K was made. (3) The case law is varied: (a) Some courts hold that a lawyer may not simply notify a client of an increase in the hourly rate charged. (b) Others require client consent before a lawyer increases the percentage f a settlement retained in as a contingent fee. (4) 7-2: Rising Prices (a) Consider how to communicate an increase in fees when you change your hourly rate from $150 to $215 an hour. In light of 1.5, would any of the options listed in the problem be unethical? (b) Including a notice in the next month's bill: This could probably only be ethical if there was a term in the K that said something like the fee may be increased at any time. (c) Sending a letter explaining the situation: You don't want to tell the clients too much. A letter explaining why the fee is going up doesn't do much good if the client has no say in whether they pay that amount or not. (d) Send the same letter with the caveat that if they continue to use your services, then they assent to the modification: This at least gives the clients a chance to speak up or switch lawyers. (e) Maintain your present rate for existing clients: This is probably the best and most common solution to this problem. Then you would only charge new clients the increased fee. (f) Note 1.5(b): The scope of the representation and the basis or rate of the fee and expenses for which the client will be responsible shall be communicated to the client, preferably in writing, before or within a reasonable time after commencing the representation, except when the lawyer will charge a regularly represented client on the same basis or rate. Any changes in the basis or rate of the fee or expenses shall also be communicated to the client. (i) Any changes in the fee should be communicated to the client. All of these choices literally satisfy this rule, but some are more client friendly than others.

Lying versus deception: Is there a moral distinction?

(1) In evaluating the morality of a statement, is the important question whether it is a false statement, or that the speaker intends to deceive? (2) What if the deceiving party merely remains silent - is the withholding of information immoral? (3) Like lies, some deceptions may be justifiable, but this largely depends on the circumstances. Many lawyers believe that deception is only problematic if it is accomplished by making a false statement.

a) Truth versus truthfulness

(1) Is it dishonest to make a false statement that you believe to be true? Probably not. (2) If a lawyer is diligent and honest with her client about what she learns in researching the issue, then she is being truthful even if she makes a mistake.

a) Communication about fee arrangements

(1) Many conflicts are caused by inadequate disclosure about fees at the outset of the representation. (2) See 1.5(b), that requires lawyers to make some disclosures to clients about the fees they intent to charge. (b) The scope of the representation and the basis or rate of the fee and expenses for which the client will be responsible shall be communicated to the client, preferably in writing, before or within a reasonable time after commencing the representation, except when the lawyer will charge a regularly represented client on the same basis or rate. Any changes in the basis or rate of the fee or expenses shall also be communicated to the client. (3) What must be disclosed? (a) "Information about the scope of the representation," or the general nature of the legal services to be provided. The lawyer must also disclose the "basis or rate of the fee and expenses." (b) It is suggested that the customary fee arrangement be disclosed, as well as the basis, rate or total amount of the fee and any costs/expenses or disbursements that will be charged to the client. (c) The lawyer should tell the client his hourly rate if he bills by the hour. (d) The client should be informed about any administrative costs that might be incurred, like billing per page for copies. (4) The disclosure only needs to be in writing if the lawyer intends to charge a contingency fee. It is recommended that it be in writing. (5) This disclosure must occur before or within a reasonable time after commencing the representation. Basically, the only time the disclosure should happen after work commences is when the client needs emergency legal help. (6) The lawyer need not estimate the amount of time she will spend or the total fee, but it is a good business practice to provide a realistic assessment to clients. If there is uncertainty as to how much time will be spent on a matter, the lawyer might give a high and low prediction. (7) 7-1: An Unreasonable Fee? (a) The disputed amount was $70K in the client's case, but the attorney charged a fee of $60K. A satisfactory result had not even been reached yet - no trial yet - and the fees are already this much. (b) Applying Fordham, we would look at what other attorneys in the field were charging and would say that he could not charge for research time. (c) If the lawyer came up with a really novel argument, then that might help to substantiate the fee. If the basis of the fee charged was set out in writing ahead of time, then many of these problems could be avoided. The problem is that only contingency fee arrangements must be in writing under the rules. (d) Rule 1.5 does NOT require lawyers to give an estimate of the fee. It would not be a good idea to provide a precise estimate anyway, as the client will hold you to it.

c) 5-3 The Injured Passengers, Scene 2

(1) One of the client's injuries is more serious than the other. You have concluded that you can represent them both, and you have obtained informed consent. The taxi cab co. contacts you about settlement - they may offer $350K for the injured client's claim if you'll accept only $50K for the other client's claim. (2) This would pin one client against another - direct concurrent conflict. This seems to meet all the requirements of 1.7(b), so as long as the clients are fully informed about the deal and they accept the settlement, you may proceed without violating any rules.

a) Juveniles

(1) Same standard is applied to juveniles as is applied to mentally impaired adults. Children are regarded as having opinions entitled to weight in custody proceedings. (2) Delinquency cases: Usually older children and the cases resemble criminal cases; most follow representation similar to the norms followed when representing adults in court (3) Custody, abuse and neglect proceedings: Usually younger children than in delinquency cases; must balance between the child's best interests and the child's stated wishes (4) ABA Standards of Practice (a) "Child's attorney" (preferred): Lawyers owe the same duties of loyalty, confidentiality and competent representation to a child as is due to an adult client. (i) Must advocate the child's articulated position. (ii) Should explain the situation in such a way as to allow the child to articulate an informed decision. The lawyer should not overbear the will of the child. (iii) If the expressed preference would be seriously injurious the lawyer may request appointment of a separate guardian ad litem and continue to represent the child's expressed preference. (b) "Lawyer appointed as guardian": If lawyer is appointed as "guardian ad litem" then he must protect the child's interest, but is not bound by the child's expressed preferences.

a) Representation prohibited by law

(1) Some conflicts may not be consentable because the representation of a client in the face of some conflicts is prohibited by statute or case law. (2) This will differ between jurisdictions.

a) 6-6: Top Gun

(1) The city is suing a gun manufacturer because they knowingly sold guns to dealers, that then sold to criminals. Your firm takes on the city's claim. The firm also represents a large pharmaceutical company. The pharm co. doesn't want the firm to take the city's case, because it might set a bad precedent for them (may face liability for selling drugs). (2) This is called a positional legal conflict. Traditionally, lawyers are like taxi drivers - the client can jump into the taxi and the lawyer will take the client anywhere they want to go. Lawyers should not be limited in their ability to assert legal theories. (a) In this case, this is not a conflict of interest because the lawyer is responsible for coming up with legal theories, it is their responisibility (either side may be made but that us the lawyer's job) (3) What if the pharm co. calls you up and says they'll fire your firm if you don't drop the gun case? While there doesn't appear to be any direct adversity here, this could materially limit your representation against the gun manufacturer, since you might be pulling punches by giving up a good claim to suit your big clients. This threat ratchets up the positional conflict. (4) Cmt. 24 to rule 1.7: Maintains that a lawyer is a taxi cab, but there might be an exception for indirect conflicts. "Ordinarily a lawyer may take inconsistent legal positions in different tribunals at different times on behalf of different clients. The mere fact that advocating a legal position on behalf of one client might create precedent adverse to the interests of a client represented by the lawyer in an unrelated matter does not create a conflict of interest. A conflict of interest exists, however, if there is a significant risk that a lawyer's action on behalf of one client will materially limit the lawyer's effectiveness in representing another client in a different case; for example, when a decision favoring one client will create a precedent likely to seriously weaken the position taken on behalf of the other client. Factors relevant in determining whether the clients need to be advised of the risk include: where the cases are pending, whether the issue is substantive or procedural, the temporal relationship between the matters, the significance of the issue to the immediate and long-term interests of the clients involved and the clients' reasonable expectations in retaining the lawyer. If there is significant risk of material limitation, then absent informed consent of the affected clients, the lawyer must refuse one of the representations or withdraw from one or both matters." b) Ask if there is something going on that would cause you not to be as competent in representation as you usually would. If the answer is yes, then the positional conflict will turn into an indirect one, and needs to be evaluated under 1.7.

1. Candor and communication a) Is it ever okay to lie?

(1) There are many different lies that a lawyer could tell: (a) White lies, to avoid embarrassment or to avoid hurting another person's feelings (b) Lies to protect people (c) Lies to protect your own privacy (2) The immorality of lying is largely left up to individual standards, but here are a few questions to assess whether the lie is justifiable: (a) Is the subject matter of the lie trivial or private? (b) Is anyone harmed by the lie? (c) Is the purpose of the lie to protect someone? (d) Does the person lied to have a right to know (or a strong interest in knowing the truth)? (e) If there is a reason to tell a lie, can the problem be solved without lying? (f) If you tell this lie, will you need to tell other lies to cover up the first one?

a) 4-3: The Washing Machine (1) Guy buys a washing machine who is the coach of your daughter's soccer team. He doesn't have much money and is having trouble making the payments on the machine, so you try to work out a deal with the appliance store to get him lower payments.

(1) Would it be competent to immediately accept an offer without doing any research to determine whether the seller was trying to gouge this guy? (a) Duty: you need to pass the offer to the principle. Pass along offer. Now client says should I take it? So he takes it and fulfills terms of agreement. Anything more you should do? (b) A counter offer would probably be a good idea. (2) If you had gotten a case like this, then you would want to look at consumer protection law. In the actual case that this was based on, the lawyer found a law against creditor harassment, and after confronting the creditor about excessive phone calls made to his client, the client got to keep the washing machine at no cost.

7-8: Two Masters

(1) You are an attorney in a large firm that represents both the insurance company and the actual D (the insured). The insurance company pays the legal bills. The insurance company wants to settle, but the insured doesn't want to. (2) Change the facts: Suppose the insurance policy will cover only $100K in damages, and on the eve of trial, the P makes an offer to settle for $98K. The insured would be in favor of settlement, because then he wouldn't have to pay anything, but the insurance company would have an incentive to go to trial and try to win altogether. (3) What if the insurance policy has a term that says that the insured must settle when the insurer says it wants to? If this is in the K, then the insured doesn't have much of a choice. (4) If the insurance company is impinging on the lawyer-client relationship between you and the insured, then you can tell the insurance company that they have a duty to fund the defense of the insured, they need to do a reasonable job, or otherwise their paying for a deficient defense. In such a case, either the insured or the lawyer may sue.

Rule 7.3 Direct Contact With Prospective Clients

(a) A lawyer shall not by in-person, live telephone or real-time electronic contact solicit professional employment from a prospective client when a significant motive for the lawyer's doing so is the lawyer's pecuniary gain, unless the person contacted: (1) is a lawyer; or (2) has a family, close personal, or prior professional relationship with the lawyer. (b) A lawyer shall not solicit professional employment from a prospective client by written, recorded or electronic communication or by in-person, telephone or real-time electronic contact even when not otherwise prohibited by paragraph (a), if: (1) the target of the solicitation has made known to the lawyer a desire not to be solicited by the lawyer; or (2) the solicitation involves coercion, duress or harassment. (c) Every written, recorded or electronic communication from a lawyer soliciting professional employment from anyone known to be in need of legal services in a particular matter shall include the words "Advertising Material" on the outside envelope, if any, and at the beginning and ending of any recorded or electronic communication, unless the recipient of the communication is a person specified in paragraphs (a)(1) or (a)(2). (1) Suppose Premus calls up the woman, and says that if she doesn't accept the representation, then Premus will cut off her welfare benefits. This is clearly prohibited under (b). (2) If you know that someone needs legal assistance on a matter, then you have to put this legend on the communication under (c). This only applies where the communication is targeted. If the communication is widespread, then no disclaimer is necessary. (3) The La rule is pretty consistent (4) If you want business so you can eat that is an issue! Cannot go up to a person in the street or call them on the phone. No live chats or ims. (5) Frisbees à if go to people that are known to need a lawyer that is not ok. If just handing out that is fine. (6) NO LIVE OR IN PERSON SOLICITING where pecuniary gain deal but ok for assertion of constitution rights (7) Advertising v. soliciting

Rule 1.4 Communication

(a) A lawyer shall: (1) promptly inform the client of any decision or circumstance with respect to which the client's informed consent, as defined in Rule 1.0(e), is required by these Rules; (2) reasonably consult with the client about the means by which the client's objectives are to be accomplished; (3) keep the client reasonably informed about the status of the matter; (4) promptly comply with reasonable requests for information; and (5) consult with the client about any relevant limitation on the lawyer's conduct when the lawyer knows that the client expects assistance not permitted by the Rules of Professional Conduct or other law. (b) A lawyer shall explain a matter to the extent reasonably necessary to permit the client to make informed decisions regarding the representation. (1) Cmt. 1: Reasonable communication between the lawyer and the client is necessary for the client effectively to participate in the representation. (2) This rule does not require lawyers to communicate every minor event that occurs in the course of a representation with the client. He must only give the client enough information so that he can participate intelligently in decisions about objectives and means. (3) It's unclear whether 1.4 prohibits lying to clients, but it certainly does require the lawyer to inform clients about important developments in their cases. (4) Under (a)(2), the lawyer must consult with clients about the means to carry out the client's objectives. However, rule 1.2 gives the lawyer some discretion about the means used to carry out the representation. (5) Status = significant developments affecting the timing or substance of the representation (6) If a lawyer cannot respond promptly to a request for information, then he should explain when a response may be expected. In addition, client telephone calls should be promptly returned or acknowledged.

Rule 1.9 Duties To Former Clients

(a) A lawyer who has formerly represented a client in a matter shall not thereafter represent another person in the same or a substantially related matter in which that person's interests are materially adverse to the interests of the former client unless the former client gives informed consent, confirmed in writing. (b) A lawyer shall not knowingly represent a person in the same or a substantially related matter in which a firm with which the lawyer formerly was associated had previously represented a client (1) whose interests are materially adverse to that person; and (2) about whom the lawyer had acquired information protected by Rules 1.6 and 1.9(c) that is material to the matter; unless the former client gives informed consent, confirmed in writing. (c) A lawyer who has formerly represented a client in a matter or whose present or former firm has formerly represented a client in a matter shall not thereafter: (1) use information relating to the representation to the disadvantage of the former client except as these Rules would permit or require with respect to a client, or when the information has become generally known; or (2) reveal information relating to the representation except as these Rules would permit or require with respect to a client. 1. The ethics rules are generally less restrictive to successive conflicts than they are to concurrent conflicts - informed consent will usually solve most conflicts. The lawyer's duties to a former client are limited to protecting confidences, avoiding side switching, and refraining from attacking the work the lawyer did for the former client. The passage of time reduces the likelihood that a lawyer will deliberately or inadvertently misuse information received from a former client. B. Duties to Former Clients 1. The primary duty owed is to protect their confidences. Rule 1.9(c) bars use or revelation of confidences of former clients or clients of a former firm to the same extent as such use or revelation is barred for present clients. 2. Under 1.9(a), a lawyer may not do work on behalf of a new client, without the informed consent of the former client, IF: a) That work involves the same or substantially related matter as the former representation AND b) The new client's interest are materially adverse to the interests of the former client. 3. A lawyer is ALWAYS permitted to proceed with the new representation if the lawyer can secure informed consent from the former client. 4. Remember that the test is not substantially similar, but substantially RELATED.

Rule 1.6 Confidentiality Of Information

(a) A lawyer shall not reveal information relating to the representation of a client unless the client gives informed consent, the disclosure is impliedly authorized in order to carry out the representation or the disclosure is permitted by paragraph (b). (b) A lawyer may reveal information relating to the representation of a client to the extent the lawyer reasonably believes necessary: (1) to prevent reasonably certain death or substantial bodily harm; (2) to prevent the client from committing a crime or fraud that is reasonably certain to result in substantial injury to the financial interests or property of another and in furtherance of which the client has used or is using the lawyer's services; (3) to prevent, mitigate or rectify substantial injury to the financial interests or property of another that is reasonably certain to result or has resulted from the client's commission of a crime or fraud in furtherance of which the client has used the lawyer's services; (4) to secure legal advice about the lawyer's compliance with these Rules; (5) to establish a claim or defense on behalf of the lawyer in a controversy between the lawyer and the client, to establish a defense to a criminal charge or civil claim against the lawyer based upon conduct in which the client was involved, or to respond to allegations in any proceeding concerning the lawyer's representation of the client; or (6) to comply with other law or a court order. (1) This rule requires lawyers to protect as confidential all information relating to the representation of a client. The information need not be received from the client in order for it to fall under the confidentiality duty. (2) Part (a) describes the basic duty of confidentiality. There are exceptions already built into the basic rule: (a) Informed consent by the client (b) Impliedly authorized disclosure necessary to carry out the representation. (3) Part (b) lays out the exceptions. There are other exceptions that are not put in the rule. (4) Note the scope of the duty: It covers information relating to the representation of a client (all information - this is huge!). (5) Cmt. 4: The prohibition in para (a) also applies to disclosures by a lawyer that do not in themselves reveal protected information, but could reasonably lead to the discovery of such information by a third person. The use of a hypo is to discuss issues is permissible, as long as there is no reasonable likelihood that the listener will be able to ascertain the identity of the client. (6) Information that is "generally known" will not fall under the confidentiality duty. Perhaps a lawyer may even reveal confidences if the layer keeps the identity of the client sufficiently hidden, as long as no harm could result from telling the story - the rule doesn't say this, but that's how its understood. Generally, most important details about a client's representation cannot be divulged, even years after the representation has terminated. (7) Note that information acquired before the representation begins and after the representation terminates is protected by this duty. (8) Restatement - more forgiving stating that so long as disclosure if reasonable prospect will adversely affect client. (9) Authors of the text stick with anonymity and if there is a marginal call air on the side of caution and don't speak. b) What could happen if a lawyer fails to protect confidences? A number of things: the client might fire the lawyer or refuse to pay the fee, professional disciplinary action, liability in tort or K, disqualification from representation, or an injunction from further revelation. c) What is the policy behind the confidentiality rule? The purpose is to facilitate open communication between lawyers and clients. d) Is the fact that a lawyer is representing a particular client confidential? Maybe. If a lawyer, with the client's permission, reveals that she is representing a particular client, she must then avoid disclosing confidential information about the client's matter. In cases where the client doesn't want anyone knowing that they have consulted an attorney, the fact of consultation or representation will be confidential. e) 2-1: Your Dinner with Anna, Scene 1 (p. 155) (1) A non-lawyer friend asks what you have done at your new job. What can you tell her? Most courts say that as long as you don't give identifying specifics about your client (names, dates, etc.), then you won't breach the duty. Under rule 1.6. (2) The purpose behind this confidentiality rule could be to avoid embarrassment, and to encourage truth telling from client to lawyer. This is similar to doctor-patient confidentiality, it aids to encourage truth-telling from the patient/client. (3) What if you swore her to secrecy? This is probably not a good idea, as Anna may prove to be unreliable. There is also the risk of telling her this stuff in a public place - a third person might overhear. You probably need to be more cautious in smaller towns. (4) It is also dangerous to breach client confidences just because of the reputation you would develop (economic consequences as well as disciplinary). (5) There's not much in this conversation, about Joey's story, that should be shared.

Rule 1.18 Duties To Prospective Client

(a) A person who discusses with a lawyer the possibility of forming a client-lawyer relationship with respect to a matter is a prospective client. (b) Even when no client-lawyer relationship ensues, a lawyer who has had discussions with a prospective client shall not use or reveal information learned in the consultation, except as Rule 1.9 would permit with respect to information of a former client. (c) A lawyer subject to paragraph (b) shall not represent a client with interests materially adverse to those of a prospective client in the same or a substantially related matter if the lawyer received information from the prospective client that could be significantly harmful to that person in the matter, except as provided in paragraph (d). If a lawyer is disqualified from representation under this paragraph, no lawyer in a firm with which that lawyer is associated may knowingly undertake or continue representation in such a matter, except as provided in paragraph (d). (d) When the lawyer has received disqualifying information as defined in paragraph (c), representation is permissible if: (1) both the affected client and the prospective client have given informed consent, confirmed in writing, or: (2) the lawyer who received the information took reasonable measures to avoid exposure to more disqualifying information than was reasonably necessary to determine whether to represent the prospective client; and (i) the disqualified lawyer is timely screened from any participation in the matter and is apportioned no part of the fee therefrom; and (ii) written notice is promptly given to the prospective client. (1) La. has also adopted this relatively new rule. (2) Note that there is a duty of confidentiality for information learned from a prospective client. Another major duty is to avoid conflicts of interests between the prospective client and present clients.

Rule 1.7 Conflict Of Interest: Current Clients (through 1.12)

(a) Except as provided in paragraph (b), a lawyer shall not represent a client if the representation involves a concurrent conflict of interest. A concurrent conflict of interest exists if: (1) the representation of one client will be directly adverse to another client; or (2) there is a significant risk that the representation of one or more clients will be materially limited by the lawyer's responsibilities to another client, a former client or a third person or by a personal interest of the lawyer. (b) Notwithstanding the existence of a concurrent conflict of interest under paragraph (a), a lawyer may represent a client if: (1) the lawyer reasonably believes that the lawyer will be able to provide competent and diligent representation to each affected client; (2) the representation is not prohibited by law; (3) the representation does not involve the assertion of a claim by one client against another client represented by the lawyer in the same litigation or other proceeding before a tribunal; and (4) each affected client gives informed consent, confirmed in writing.

(1) 4-4 Lying to Clients

(a) Explains many different situations where a lawyer would have the opportunity to lie or tell a half-truth to his client. Are any of them ever ok? (b) Exaggerating expertise? This is better characterized as misrepresentation rather than lying. This would be a clear cut violation of 1.4. (c) "I never reviewed the document." If this is an outright lie, then it would also violate 1.4. (d) What if a lawyer misses a filing deadline, remedies the situation, and didn't report to the client? There may be no duty to disclose the error here, since it didn't cause any harm to the client. This might, however, fall under a violation of the duty of competence - depends on how quickly you discover and remedy the error. What if a client is sent a document that is meant for someone else and you blame your secretary? The client may never know the difference, but if it gets back to your client, she'll make your life miserable.

(1) 4-8: The Foster Child

(a) Facts: You represent 8 year old girl, Grace (not guardian ad litem). Removed from mother's home because mom doing drugs and beating her. She has two younger brothers, but they have not been removed. Placed in temporary foster care with Valerie Waite (older black woman; career foster parent) - very good care. Grace wants to return to mom's house to take care of brothers, but changes her mind when mom's BF is there. Hearing soon - will determine whether she will stay with Waite (your preference) or be placed with white, late 20's couple (Social Services' preference). (i) Grace: Wants to live with her mom (if BF isn't there), or grandma (ii) Mom and grandma: Want Grace returned home; if not, with grandma (but grandma has no training in taking care of children - did a bang up job with mom) (iii) White couple: Invested a lot in being trained as a therapeutic foster parents and want Grace placed with them (iv) Psychiatrist: Recommends keeping her with Waite or with the white couple. Will not get medication with mom. Said she is capable of knowing what she wants. (b) Issue: Should you advocate a particular placement for Grace? Response: Under ABA standards, must represent the interests of the child. Must balance the expressed preference of the child with the child's best interests. Perhaps she should be placed with Waite until Social Services can assess whether grandma would be an adequate guardian. Placing her with her grandmother is a good compromise - still within family, but not an abusive household. Not sure if she would get medication, but grandma would be a good buffer between mom and Grace. Otherwise, she should just stay with Waite - this appears to be her preference when mom's BF is there, even though she says her second preference is with grandma. Perhaps the best option is to just buy some time.

Rule 1.14 Client With Diminished Capacity

(a) When a client's capacity to make adequately considered decisions in connection with a representation is diminished, whether because of minority, mental impairment or for some other reason, the lawyer shall, as far as reasonably possible, maintain a normal client-lawyer relationship with the client. (b) When the lawyer reasonably believes that the client has diminished capacity, is at risk of substantial physical, financial or other harm unless action is taken and cannot adequately act in the client's own interest, the lawyer may take reasonably necessary protective action, including consulting with individuals or entities that have the ability to take action to protect the client and, in appropriate cases, seeking the appointment of a guardian ad litem, conservator or guardian. (c) Information relating to the representation of a client with diminished capacity is protected by Rule 1.6. When taking protective action pursuant to paragraph (b), the lawyer is impliedly authorized under Rule 1.6(a) to reveal information about the client, but only to the extent reasonably necessary to protect the client's interests. (1) How can a lawyer asses his client's mental capacity? See factors in R. 1.14, cmt. 6: (a) Client's ability to articulate reasoning leading to a decision; (b) Variability of state of mind and ability to appreciate consequences of a decision; (c) The substantive fairness of a decision; and (d) The consistency of a decision with the known long-term commitments and values of the client. b) Clients who may have mental disabilities (1) Several options for the lawyer: (a) Follow client's instructions (treat the client as competent) (b) Impose own ideas of what is best (c) Invite others to provide substitute guidance (d) Petition the court to appoint a third party to represent the interests of the client (i) Guardian at litem: Can speak for the client in a legal matter (ii) Conservator: Can manage the financial affairs (iii) Guardian: Manages the client's financial affairs and may make medical or other personal decisions for the client

Rule 1.10 Imputation Of Conflicts Of Interest: General Rule

(a) While lawyers are associated in a firm, none of them shall knowingly represent a client when any one of them practicing alone would be prohibited from doing so by Rules 1.7 or 1.9, unless the prohibition is based on a personal interest of the prohibited lawyer and does not present a significant risk of materially limiting the representation of the client by the remaining lawyers in the firm. (b) When a lawyer has terminated an association with a firm, the firm is not prohibited from thereafter representing a person with interests materially adverse to those of a client represented by the formerly associated lawyer and not currently represented by the firm, unless: (1) the matter is the same or substantially related to that in which the formerly associated lawyer represented the client; and (2) any lawyer remaining in the firm has information protected by Rules 1.6 and 1.9(c) that is material to the matter. (c) A disqualification prescribed by this rule may be waived by the affected client under the conditions stated in Rule 1.7. (d) The disqualification of lawyers associated in a firm with former or current government lawyers is governed by Rule 1.11. a) This rule takes the position that a firm of lawyers is essentially one lawyer for purposes of the rules governing loyalty to the client. If one lawyer has a conflict (is "infected"), then so do all the other lawyers in the firm (Smith likes to call this "Typhoid Mary"). b) Some conflicts between a client's interest and a lawyer's personal interest are not imputed to other lawyers in the firm. c) Such conflicts may be waived by a client so affected by the conflict, subject to the restrictions articulated in 1.7(b). d) Some jurisdictions allow law firms to avoid imputation of conflicts by screening the conflicted lawyer from any contact with the matter that presents the problem. In these jurisdictions, if the screening procedure is adequate, then the firm need not get the client's consent to proceed. In very limited circumstances, the model rules do not permit screening as an alternative to client consent to avoid imputation of a conflict within a firm. e) What is a "firm"? This term includes "lawyers in a law partnership, professional corporation, sole proprietorship, or other association" as well as "lawyers employed in a legal services organization or the legal department of a corporation or other organization." This rule applies regardless of the size of the firm and regardless of how many offices around the world the firm may have. The larger the firm, the larger the number of conflicts! f) Lawyers who have separate practices, but share the same office space may be considered a "law firm" for conflicts purposes if their file management and communication is such that one lawyer in a suite might have access to confidential information about a matter being handled by another lawyer. g) The imputation rules for government lawyers are covered in rule 1.11. h) The imputation rules only apply to lawyers. They do not apply to conflicts presented by law clerks, paralegals, secretaries or other nonlawyer employees. However, nonlawyers are still expected to protect client confidences like lawyers are. If a conflict is presented by a nonlawyer employee, then a comment to 1.10 explained that the conflicted person ordinarily must be screened from any personal participation in the matter.

(1) 5-10: Representing the McCarthys

(a) You represent H and W in estate planning. There is a separate woman who is suing H in a paternity suit, and you also represent this woman. There was a mistake in the client records that made you think there was no conflict of interest. (b) There is a direct concurrent conflict here in regard to the H. What happens when the W finds out that the H may be the father of this child? In regard to the W, because she isn't a party to the paternity suit, there is only an indirect conflict. (c) What if you withdraw from representing this strange woman? There may still be a conflict with W because there is a claim out there that she doesn't know about that will affect her estate. Also, you know confidential information about the strange woman's case that could influence your work for the W. (d) What if you withdraw from representing H and W? There is still a conflict because you know information about H's financial condition that might be to his disadvantage in the paternity suit. (e) There isn't a clear approach to the "publicly known infromation" under 1.6 confidencitality rule (2) Other hypo's that might arise in a H and W situation: (a) The W has assets that she doesn't want the H to know about. If H finds out about them, then it might make a difference in dividing their estate or in a divorce settlement. Can you disclose this to the H? (b) Can you represent both H and W in an uncontested divorce? It would probably not be a good idea, because of all the things that might come up - insurance premiums for kids, education, etc.

(1) 7-3: An Impoverished Client

(a) You represent a 62 year old man that recently lost his job as a night security guard. If he wins his case, he'll be awarded $50K, and the lawyer will collect 1/3. The client is broke - he can't pay his rent, etc. Can the lawyer pay such living expenses for the client? (b) See 1.8(e). Court costs and expenses can be advanced, but this rule does not allow the lawyer to pay living expenses like rent or cable TV. (c) What about medical bills? Cmt. 10 says that the lawyer can pay for medical examinations (since its an expense as a result of the litigation). However, medical care for actual treatment cannot be paid for by the lawyer. (d) This reflects traditional notions of champerty and maintenance (these were prohibited kinds of advances under old CML notions). (e) Note how the result would change under the La. rules.

6-6: The Secret Affair

(under 1.18) These communications might be unilateral and is there a rereal attorney-client relationship? If this is an atty-client relationship (Togstad), then all those rules come into play. There likely wasn't a formal atty-client relationship. Also, if there was no "discussion" then this doesn't engage 1.18. If we've decided we can't represent the woman, the husband would like to get that info. What do we need to disclose to the husband to get informed consent?

1. Responsibility for client property

1. a) Prompt delivery of funds or property (1) Once the lawyer receives a settlement check or other funds to be paid to the client, he is obliged to: (a) Notify the client and (b) To make prompt payment of all funds due to the client. (2) If the client requests a record of the amount received and of how much was paid to whom, the lawyer must provide it. b) Disputes about money or property in lawyer's possession (1) Under 1.15(c), if there is a dispute about the amount of the fee, the lawyer is to distribute undisputed portions of the settlement and keep the disputed portion in the client trust account. If you transfer the disputed amount to your account for earned money, then this will be considered conversion (or stealing). (2) Note that if the client gives the lawyer his property to watch over, the lawyer must safekeep it. Therefore, if the property is damaged while under the lawyer's watch, the lawyer will be liable. (3) Now the amount you agree on can be moved to the operating account (4) The amount in dispute is the amount you think you should get minus what they think you should get. It sits in the trust account until the matter is resolved. (5) Note LASC case found that even if the client's claim is no goof you still have to do this! If not you could be disbarred. c) Lawyers' responsibilities to clients' creditors (1) Comment. to rule 1.15: If a third party has a "lawful claim" against funds that are in the lawyer's custody, the lawyer might have some obligation to the third party. (2) The lawyer should hold the property until the claims are resolved. (3) However, a lawyer is not a collection agency for all his client's creditors. Third party creditors have no right to ask the lawyer to give them money that the client owes, even if the debt is legitimate and overdue. (4) Unless the creditor has a legitimate claim to the particular funds in the lawyer's possession, the lawyer's duty is to his client, not to the third party. This might happen where the creditor has obtained a judgment against the client and obtains an order against the lawyer to surrender the funds.

Advertising and solicitation

1. Back when the world was new, lawyers could not advertise AT ALL. They could have small signs, but all they could say was your name. You could not advertise on billboards, TV or radio. 2. In 1977, the Bates case was decided. They took out an ad in the local newspaper. The SC said that the first amendment protects lawyers and you cannot have blanket prohibitions against truthful statements. Advertising is now ok. Government is free to prevent dissemination of commercial speech that is false, deceptive, or misleading, or that proposes an illegal transaction. Otherwise, commercial speech may be restricted only in the service of a substantial governmental interest, through means that directly advance that interest. And, as with other speech, commercial speech may be regulated as to time, place, and manner. a) Commercial speech = protected by the First Amendment b) Constitution allows lawyers to advertise. But also allows states to prohibit false or misleading advertising. 3. Zowderer: Ran an ad in Ohio that had picture of a birth control device. The bar went after him because it was undignified. The court said they weren't concerned about the content. They relied on Central Hudson - "Government is free to prevent dissemination of commercial speech that is false, deceptive, or misleading, or that proposes an illegal transaction. Otherwise, commercial speech may be restricted only in the service of a substantial governmental interest, through means that directly advance that interest. And....as with other speech, of course, commercial speech may be regulated as to time, place, and manner." a) The Supreme Court said the 1st amendment is violated by prohibiting this type of image, but the bar can require a disclaimer. 4. Now we have rules that take into account the 1st amendment a) Rule 7.1

A. Imputations of Former Client Conflicts to Affiliated Lawyers

1. In general a) To what extent are conflicts caused by work done for former clients imputed to other lawyers in a firm? b) See rule 1.10 for the imputation rules for private sector lawyers. Rule 1.10 Imputation Of Conflicts Of Interest: General Rule (a) While lawyers are associated in a firm, none of them shall knowingly represent a client when any one of them practicing alone would be prohibited from doing so by Rules 1.7 or 1.9, unless the prohibition is based on a personal interest of the prohibited lawyer and does not present a significant risk of materially limiting the representation of the client by the remaining lawyers in the firm. (b) When a lawyer has terminated an association with a firm, the firm is not prohibited from thereafter representing a person with interests materially adverse to those of a client represented by the formerly associated lawyer and not currently represented by the firm, unless: (1) the matter is the same or substantially related to that in which the formerly associated lawyer represented the client; and (2) any lawyer remaining in the firm has information protected by Rules 1.6 and 1.9(c) that is material to the matter. (c) A disqualification prescribed by this rule may be waived by the affected client under the conditions stated in Rule 1.7. (d) The disqualification of lawyers associated in a firm with former or current government lawyers is governed by Rule 1.11. (1) General rule: If one lawyer in a firm has a conflict, all are precluded, with the exception of conflicts that do not involve client loyalty or protection of confidences (personal interest conflicts). (2) If a lawyer leaves a firm, his former firm should use this rule to evaluate new business that conflicts with the former lawyer's work at the firm. The conflict will leave the firm with the lawyer unless: (a) The matter is the same or substantially related and (b) A remaining lawyer knows material confidential information. (3) Most imputed conflicts can be waived. (4) New and fairly elaborate (5) This has not been adopted in LA (a) So choices are: (i) Not to hire (ii) Hire and roll the dice (iii) Get consent (b) This rule does not require consent once you make the wall and give the notices. Not ok in LA!

Conflicts in Nonlitigation Matters: Representation of Both Parties to a Transaction

1. Can you represent both the buyer and the seller of a business? In most circumstances, it is ok, but you would need to provide the client with information about the possible disadvantages of joint representation, and also obtain their consent. 2. This rule generally applies whenever a lawyer is approached by two clients seeking legal assistance with a common goal. 3. Some people think that there is no way to represent both sides of a transaction aggressively. You always end up pulling punches. 4. Consent need to be obtained when there is an actual or potential conflict that is reasonably apparent to the lawyer. If the clients' interests appear entirely harmonious, then there may be no need to obtain consent. 5. A lawyer in these instances is not a "mere scrivener" - "mere scrivener" meaning I was just a scribe for the deal A set up with B. this defense will not work. He is still giving advice, even if all he sees himself doing is drafting documents. He should endeavor to provide the same range of services to each of the two clients as he would if he were representing only one of them. a) Does not work; when he provides work to people they are clients when they are looking for legal services; the lawyer lost 6. The ethical duty to protect client confidences is less clear in this context. The lawyer owes a duty of loyalty to both clients, but this might be compromised if the lawyer keeps secrets from either of them. Cmt. 31 to Rule 1.7 says that a lawyer usually should not keep confidences from one joint client received from the other. The lawyer should give the clients notice at the beginning of the representation that all information will be shared. 7. What if the two clients wind up in litigation against one another - does the lawyer have to withdraw altogether or can the lawyer represent one against the other? a) The lawyer definitely cannot continue to represent both. If the lawyer withdraws from representing one client, then that client becomes a former client. b) Here, the lawyer usually cannot represent one client in a suit against a former client because the suit would be the same or substantially related matter per rule 1.9 and its initiation would be materially adverse to the interests of the former client. c) In this case, representation is allowed only with the consent of the former client. d) A lawyer might be able to continue representation of one client if a valid advance waiver was obtained at the outset of the transactional work. Part of the informed consent process at the beginning of the representation would include a discussion of what might happen if litigation developed between the parties.

A. Practice Issues Relating to Former Client Conflicts

1. If a former client is approached about giving consent in these situations, the former client: a) Has little reason to give consent where the new client's interests conflict with those of it; b) Might claim that it is actually a current client; or c) Might later claim that the request for a waiver constitutes an admission that a waiver was needed. 2. Because of these complexities, lawyers often don't seek consent from former clients. If a lawyer proceeds despite a successive conflict without consent from the former client, the lawyer may face a motion to disqualify. This is frustrating because there is no reliable formula as to when matters are related or unrelated. 3. In evaluating a motion to disqualify, the judge does not learn details on the confidences at issue or inquire into whether relevant confidences actually were disclosed in the first representation. The judge evaluates the possibility that confidences had been disclosed that will be harmful to the former client if used in subsequent representation. 4. In determining whether to grant a motion to disqualify, one looks at the type of information that might ordinarily be disclosed in the course of prior representation. If such information could be used adversely to the client in a subsequent matter, the two matters are deemed substantially related. a) The test is not whether the lawyer actually learned information in the first matter that could harm that first client. If this were the standard, then it might require the former client to give evidence that may violate her right to confidentiality. b) The test is also not whether the lawyer could possibly have learned information harmful to the first client while representing that client. This would disqualify too many lawyers. 5. 6-2: The District Attorney a) You represented a kid in a car accident case 12 years ago back in private practice. Now you are a district attorney, and this same person has committed a crime (murder) that you must prosecute him for. He uses the defense of mental insanity. b) Could you have reasonably acquired information that could now be used to his detriment? Is there a substantial relationship between the mental insanity defense and the car accident? c) As a practical matter, you probably wouldn't be able to remember facts from 12 years ago that would impact his defense. However, in the real case, the lawyers were disqualified from the prosecution. 6. Note that these rules are usually applied like a smell test or under an "appearance of impropriety" test - if it seems unfair under the circumstances for you to be involved as a lawyer in the case, then the court will disqualify you.

A. Successive Conflicts of Present and Former Government Lawyers

1. In general a) Revolving door conflicts: This conflict comes up if a lawyer works for a government agency for a period of years and then joins a private law firm - some of the lawyer's previous work may conflict with her new practice. b) Rule 1.11 imposes less stringent standards regarding successive conflicts that arise from a lawyer's present or past government service. c) The reason for the lower barrier here is to not discourage movement between private and public sectors. d) How does 1.11 differ from rules that apply to other lawyer? (1) LESS "CONFLICTS": 1.11 has a slightly narrower standard for what constitutes a conflict: rather than "substantial relationship," this rule only requires "personal and substantial involvement." Does the lawyer possess confidential government information that could be used adversely in private practice? (2) NO IMPUTATION: The conflicts of former government lawyers are not imputed to other lawyers in a firm if the former government lawyer is screened from contact with the conflicting matter. This is the second of THREE situations in which the model rules permit screening.

A. Distinguishing Present and Former Clients

1. In general a) Sometimes clients that a lawyer regards as former clients may consider themselves to be present clients. The clients' beliefs may be reasonable. When a matter is over, lawyers will usually not write a letter that says - "The representation is over." The lawyer usually wants to maintain a bond with the client so that it will send more business. b) If there is no clear statement by the lawyer or client that the representation has ended, then the lawyer's representational authority can also end because the lawyer has completed the contemplated services. 2. Maintaining contact a) If the representation has not been formally concluded, a court may still find that the client is "present" and that a conflict with another client should be evaluated as a concurrent, rather than successive, conflict. b) 6-1: Keeping in Touch (1) Your firm did some tax work for Almond, a small business, about 5 years ago. The work took one day. Your firm sends out a newsletter to all current and former clients, and it has updates to tax law in it. One purpose of it is to bring in more business from past clients. The firm has called Almond a few times and they have declined your invitations to help them with anything. Star wants to hire the firm to sue Almond to collect unpaid bills. Is Almond a current or past client? (2) If Almond was treated as a present client, then there would be a direct, concurrent conflict. (3) If we treat Almond as a former client, then 1.9 applies instead. Almost all conflicts with former clients can be resolved with informed consent. (4) It must be determined whether Almond is a present or former client. It appears that the firm is trying to maintain a present relationship with Almond. c) If you look at the comments to 1.9, you won't see anything about what a "former client" is. However, if you look to cmt. 4 of rule 1.3 (diligence), it gives you some hints as to when the attorney client relationship has ended: "Unless the relationship is terminated as provided in Rule 1.16, a lawyer should carry through to conclusion all matters undertaken for a client. If a lawyer's employment is limited to a specific matter (this one was), the relationship terminates when the matter has been resolved (this one was). If a lawyer has served a client over a substantial period in a variety of matters, the client sometimes may assume that the lawyer will continue to serve on a continuing basis unless the lawyer gives notice of withdrawal (Tagstad). Doubt about whether a client-lawyer relationship still exists should be clarified by the lawyer, preferably in writing, so that the client will not mistakenly suppose the lawyer is looking after the client's affairs when the lawyer has ceased to do so. d) For example, if a lawyer has handled a judicial or administrative proceeding that produced a result adverse to the client and the lawyer and the client have not agreed that the lawyer will handle the matter on appeal, the lawyer must consult with the client about the possibility of appeal before relinquishing responsibility for the matter. See Rule 1.4(a)(2). Whether the lawyer is obligated to prosecute the appeal for the client depends on the scope of the representation the lawyer has agreed to provide to the client. See Rule 1.2." e) NOTE: If there is doubt about the existence of the attorney-client relationship, this should be clarified by the LAWYER in WRITING. f) Hot potato rule: You cannot just drop a client like a hot potato to avoid a potential conflict. The representation must come to a conclusion before you end the relationship. 3. Hiring and firing lawyers to create or eliminate conflicts a) If you withdraw from representing a client to solve a potential conflict, does the "fired" client become a former client for purposes of evaluating conflicts? NO. If the withdrawal is premature and motivated by a desire to dump the first client so that a lawyer can work for a new client, the lawyer's withdrawal represents a breach of the lawyer's duty to the first client. b) If one of the following conditions is satisfied, the lawyer may use the more lenient successive conflict standards to evaluate the conflict: (1) The lawyer withdraws at the natural end point in the representation. (2) The client fires the lawyer for reasons other than an impending conflict. (3) The client triggers a conflict for the lawyer by some action that was unforeseeable to the lawyer. (4) The lawyer withdraws for some other good reason (where the client insists the lawyer assist him in committing fraud or where the client won't pay fees). c) Sabotage: Where clients purposefully go to a preliminary interview or hire a certain law firm to prevent their adversary from doing so. In this instance, the lawyers may refuse the business, or attempt to avoid the conflicts by asking the new clients for advance waivers of future conflicts of interest. 4. Former in-house counsel a) If a lawyer used to work in the legal department of a corporation, is the corporation his former client on all matters that were going on during his employment? Not necessarily. b) Former in-house counsel have been disqualified for such conflicts in several cases, but the mere fact of employment does not make a lawyer the corporation's lawyer on every matter. This is usually matter-specific, depending on how much work the lawyer did for particular matters.

A. Conflicts Between the Interests of a Present Client and a Client Who Was Represented By a Lawyer's Former Firm

1. In general a) What if the former client was not represented by the lawyer personally, but by another lawyer at a law firm where one of the lawyers in the firm used to work? This is addressed in 1.9(b). (b) A lawyer shall not knowingly represent a person in the same or a substantially related matter in which a firm with which the lawyer formerly was associated had previously represented a client (1) whose interests are materially adverse to that person; and (2) about whom the lawyer had acquired information protected by Rules 1.6 and 1.9(c) that is material to the matter; unless the former client gives informed consent, confirmed in writing. b) While the analysis is largely the same as it is for 1.9(a) (same/substantially similar matter and material adversity = informed consent), but there is one crucial difference: consent is not required unless the lawyer has acquired information protected by rules 1.6 and 1.9(c) that is material to the matter. (1) Note that under 1.9(a), you ask whether the lawyer could have acquired confidential information in the first representation that might be used adversely to the former client's interests. (2) Here, however, you ask whether the lawyer actually acquired material confidential information. c) Comment 5 (1) If no actual knowledge of 1.6 information than not a problem if moved to new law firm and they represent the opposing client. (2) Note that (b) does not require that your represent the client, just than you got the information (so sitting in a meeting would count)

A. Lawyer' Responsibilities as Agents

1. Under agency principles, one can become the agent of another through: a) Express authority: Clear relationship of agency. b) Implied authority: It is the agent's belief that they have authority. c) Apparent authority: It is the third party's belief that the agent has authority based on manifestations by the principal. 2. Two forms of actual authority: Express and implied authority a) Express authority is where the client gives the lawyer explicit instructions to act on the client's behalf. b) By asking a lawyer to represent him or her in a matter, a client impliedly authorizes the lawyer to take action that is reasonable and calculated to advance the client's interest. 3. Apparent authority a) If the agent has neither express nor implied authority, she may nevertheless have apparent authority. b) When a client tells a third party that the client's lawyer has the authority to settle a claim on his behalf, the third party may rely on the lawyer's subsequent actions, even if the client did not actually authorize them. c) Apparent authority will be found if a principal places an agent in a position that causes a third person reasonably to believe that the principal had given the agent express authority. d) Note that the lawyer's statements to a third party that she is authorized to act does NOT constitute apparent authority. Only the acts or statements of a client can establish apparent authority. e) These questions of authority usually come up in settlement of a case. Most states require express authority for a lawyer to settle a case for a client, while for others, merely hiring a lawyer to represent them in litigation will authorize their lawyers to settle cases.

Shouldn't a lawyer have a stronger duty to disclose adverse facts than adverse law?

1. Not necessarily, as the lawyer's volunteering of fact that are adverse to a client is contrary to the principles of confidentiality and client loyalty. The premise of this rule is that cases should be decided within the framework of the whole body of law, not just the favorable parts that parties told the judge about. a) The rules with facts are different. It is contemplated that the facts are just going to be brought out - the adversary system is good at developing facts. You don't have a duty to bring out adverse facts. It is assumed that your opponent will do this. b) Exceptions: (1) 3.8(d): The prosecutor in a criminal case shall: (d) make timely disclosure to the defense of all evidence or information known to the prosecutor that tends to negate the guilt of the accused or mitigates the offense, and, in connection with sentencing, disclose to the defense and to the tribunal all unprivileged mitigating information known to the prosecutor, except when the prosecutor is relieved of this responsibility by a protective order of the tribunal. (a) A prosecutor that messes up on this will be penalized. There was a prosecutor in NOLA that was prosecuted a few years ago for forgetting to tell the defense or the tribunal that the murder witness wasn't wearing her glasses when she saw the murder happen. (b) The idea is that the prosecutor is not a conviction machine. They're supposed to be an agent of justice, and not just someone that goes around, convicting people.

Evaluating Successive Conflicts

1. Questions to ask a) Does the lawyer need informed consent from the former client in order to proceed with the representation? This answer will depend on these questions: (1) Is it the same matter as the previous one? (2) If not, is it substantially related to the previous one? b) If no to both, then the lawyer need not obtain informed consent from the former client. However, if one of them is a yes, then the lawyer must ask: (1) Are the new client's interests materially adverse to the interests of the former client? c) If no, then no informed consent is needed. If yes, however, the lawyer must get informed consent from the former client in order to continue. Note that the consent must be in writing. 2. The same matter a) What is a "matter"? Anything that is the subject of representation: litigation, transaction, subjects on which the client requests advice. b) Most obviously, the "same matter" refers to a single transaction or lawsuit. It will also be the same matter where the new representation involves a document that the lawyer was involved in producing. 3. Substantial relationship a) Common sense definition: Is there some connection between the earlier matter and the new matter? Same parties? Same lawsuit? Same legal issues? Same or overlapping facts? b) Legal definition: Whether the lawyer, in the course of her work in the first matter, would normally have learned information that could be used adversely to the former client in the second. Under cmt. 3 to rule 1.9, matters will be substantially related if they involve the same transaction or legal dispute or if there otherwise is a substantial risk that confidential factual information as would normally have been obtained in the prior representation would materially advance the client's position in the subsequent matter. c) Usually whether there is a substantial relationship will be a facts and circumstances type question. d) It does not matter whether the information would be used - the question is whether the information could be USEFUL. e) Where there is no opportunity to make adverse use of the confidential information, there is no substantial relationship. f) Examples: (1) A lawyer who has represented a businessperson and learned extensive private financial information about that person may not then represent that person's spouse in seeking a divorce. (2) A lawyer who has previously represented a client in securing environmental permits to build a shopping center would be precluded from representing neighbors seeking to oppose rezoning of the property on the basis of environmental considerations; however, the lawyer would not be precluded, on the grounds of substantial relationship, from defending a tenant of the completed shopping center in resisting eviction for nonpayment of rent. 4. Confidential information a) If a lawyer possesses only general knowledge that has only slight relevance to the new matter, such knowledge may not create a substantial relationship between the two matters. b) What if the lawyer knows information about how the former client approaches legal disputes? This type of knowledge may give the lawyer and his new client an advantage in subsequent litigation against the former client. Whether the knowledge amounts to a substantial relationship depends on the range of information that a lawyer could be expected to have learned during the former representation. c) If the confidential information that the lawyer learned from the former client has become public, the lawyer is not precluded from representing the new client by the possession of that knowledge. d) If the information learned in the former representation is so out of date to be irrelevant, then it is unlikely that the new matter would be found to be substantially related to the old one. 5. Variations in the definition of "substantial relationship" a) While the rules are set forth to discipline lawyers rather than disqualify them, most of the case law on successive conflicts involves motions to disqualify. Because courts in these cases are not bound to use the ethics rules as standards, courts have articulated various versions of the "substantial relationship" rule. b) For most courts, the issue is whether there is factual information learned during the first matter that could be used adversely to the first client during the second representation. Substantial relationship usually does not include matters that involve common legal issues, but some courts have held the opposite. 6. Material adversity a) The apparent intention of the 1.9 is to require consent if the use of the former client's confidences might harm the former client's interests, although neither the rules nor the comments defines "materially adverse." b) Consent will not be required where the use of confidences would not harm the former client's interests. c) The ABA Ethics Comm. has urged that "material adversity" in 1.9 should mean the same thing as "direct adversity" in 1.7. However, the Restatement concludes that material adversity is limited to potential harm to the type of interests that the lawyer sought to advance on behalf of the former client - hence no material adversity where the new work is unrelated to the previous work.

A. Joint Representation in Particular Practice Settings

1. Representation of criminal co-defendants a) The costs and benefits of joint representation of co-D's (1) In some of these situations, the conflict can be resolved with informed consent. However, some conflicts cannot be resolved at all, and the lawyer must cease representation of at least one of the co-D's. (2) The stakes are higher in criminal cases than in civil cases, as the D's may face imprisonment or even death if convicted. (3) Stonewalling: A technique that can be used by co-D's in a criminal action to keep all persons holding inculpatory information from talking to the prosecution, and thereby will defeat the criminal investigation. (4) When a defense lawyer advises neither client to make a deal because he believes he may be able to win the case for both, he is usually sacrificing a certain success for one of them. Defense lawyers clearly cannot advise one client to make a deal against the other's interest. For a strategy based on total noncooperation, the lawyer can resolve all conflicts with informed consent. (5) If the stonewall defense continues until the government decides to force immunity on one of the D's, then at this point the attorney representing both will have to divide the representation. (6) However, when multiple D's are charged in connection with one crime, the stonewall may crumble and one client may seek to reduce his penalty by giving inculpatory information about another client. b) Case law and ethics rules on joint representation (1) Most case law, ethics rule and commentary discourage joint representation of co-D's by a single lawyer. (2) Cmt. 23 to rule 1.7: "The potential for conflict of interest in representing multiple D's in a criminal case is so grave that ordinarily a lawyer should decline to represent more than one co-D." (3) A firm that does criminal defense work should not take on more than one co-D unless there is no conflict (which is going to be rare). (4) Restatement: A lawyer may not represent criminal co-D unless: (a) The clients give informed consent and, (b) It is reasonably likely that the lawyer will be able to provide adequate representation to the clients. (5) In a federal court case, if a lawyer proposes to represent 2 criminal D's, the judge must hold a hearing and advise both D's of their right to separate counsel. (6) What can happen to a criminal defense lawyer that proceeds in the face of conflict? He can either be disciplined or disqualified from representation. In addition a conviction may be reversed. c) The 6th amendment and joint representation (1) Joint representation of criminal D's may violate the 6th amendment right to counsel. Recall the elements of the D's cause of action under the 6th amendment: (a) Deficient performance (b) Prejudice (more likely than not that it would have made a difference). Note that if there is a conflict of interest, prejudice is presumed!! (2) If a D is represented by a lawyer who has a conflict of interest, he may challenge his conviction on the basis that he was denied the effective assistance of counsel. Not all conflicts that violate rule 1.7 will be serious enough to reverse a conviction. (3) Holloway v. Arkansas (1978): Whenever a trial court improperly requires joint representation over timely objection, reversal is automatic. (4) Cuyler v. Sullivan (1980): If a D who was jointly represented is convicted, and there was no objection to the joint representation at the time, to overturn the conviction on 6th amendment ground, the D must show that there was a conflict of interest that actually affected the adequacy of his representation (this is not the same standard as prejudice). (5) Mickens v. Taylor (2002): If the trial judge is not aware of a conflict (and thus not obligated to inquire), prejudice will be presumed only if the conflict has significantly affected counsel's performance - thereby rendering the verdict unreliable. (6) A criminal D is entitled to give a knowing waiver of conflict created by his lawyer's representation of another co-D in the same case, and if he waives the conflict, he may not challenge a subsequent conviction on the basis of that conflict. Where there is a great risk of prejudice, a judge can disqualify counsel from representing co-D's, even if the co-D's want to waive the conflict.

A. The Duty to Disclose Adverse Legal Authority

1. Rule 3.3(a)(2) prohibits a lawyer from knowingly failing to disclose legal authority in the controlling jurisdiction that the lawyer knows is directly adverse to her client's position, if an opponent has not already informed the judge of the adverse authority. (a) A lawyer shall not knowingly: (2) fail to disclose to the tribunal legal authority in the controlling jurisdiction known to the lawyer to be directly adverse to the position of the client and not disclosed by opposing counsel. a) This seems to be a rule that says you have to do your opponent's homework. This is counter to our intuition and most lawyers don't like this rule. b) What is "legal authority"? What about cases in La. - would that be authority? There is a comment to the CC that says that jurisprudence is secondary authority. Cases will probably still fit within "authority." (1) Suppose you are a La. lawyer that is arguing before a La. appellate court. There are two periods of prescription - you want the longer one. There is a Fla. case that says that in these circumstances, the shorter period should apply. Do you have an obligation to disclose this to the La. app. court? No, because its not legal authority in the controlling jurisdiction. Suppose the case was actually from the same cir. court in La.? In that case, you probably do have a duty to disclose it. Suppose its a LASC case? Must disclose. Different cir. court in La.? Not in the same circuit you're in, so this is a little unclear. Is it controlling? Perhaps not. This same principle could be applied on the federal scale as well. There is no binding authority for cases from other circuit courts. (2) This is unclear because there are commentators that say that controlling jurisdictions should mean STATE authorities. If that is the answer, then one cir. app. court decision should bind another circuit court, and so you must disclose. The Restatement takes the opposite view - one circuit won't control another. There is no La. rule or case that answers this question. (3) This can come up in the context of FRCP rule 11 or the state equivalent. (4) Jeffery Hazard (one of the commentators): How do you know when the authority is directly adverse? The more unhappy the lawyer is that he found it, the greater the likelihood that he must report it. c) Hypos from class: (1) Case in La. cir. court. The case tends to support your position in the litigation. Must you reveal it under this rule? No, but you'll probably reveal it anyway since it would be in your best interest to do so. (2) Suppose that the La. decision is long and confusing, but you think it might be adverse to your position. Opposing counsel thinks that it might support your position, and they have been trying to distinguish it from your case. Do you have to disclose? No. The other side has already disclosed it, although they messed it up. S doesn't think that you could take the position in oral argument that the case supports your position if you don't think it does. That would be misrepresenting the law, which is not ok. The purpose of the rule is to make sure the judge is aware of all the law - you cannot hide the ball from the judge. If it's not directly adverse to your position on the law, then you don't have to disclose it.

A. The Nature of Conflicts Between Present and Former Clients

1. Ways in which the interests of a former client may affect the representation of a present client: a) The lawyer might betray confidences of a former client; b) The lawyer may make adverse use of confidences that the lawyer learned during the representation of the former client; c) He might attack or challenge work that he did on behalf of the former client; or d) He might engage in work that is in some other way disloyal to the former client or at least causes the former client to fee betrayed. 2. If the interests of the former and present clients conflict, a lawyer might be less zealous on behalf of the present client - may pull punches. The lawyer may also favor her current client over a past client since the current client is paying fees presently. 3. If a present and former client's interest conflict, a lawyer would refer to rule 1.7 for guidance on protection of the interests of the present client. "Former clients" are included in the list of persons in 1.7(a)(2) with which the client may have a concurrent conflict with. 4. Protections for former clients are provided by rule 1.9. If a situation involves possible compromises of the interests of both the new and former client, then the lawyer should look to both rules for guidance.

Rule 7.1 Communications Concerning A Lawyer's Services

A lawyer shall not make a false or misleading communication about the lawyer or the lawyer's services. A communication is false or misleading if it contains a material misrepresentation of fact or law, or omits a fact necessary to make the statement considered as a whole not materially misleading. (1) This tracts the language of the cases. b) Rule 7.2 Rule 7.2 Advertising (a) Subject to the requirements of Rules 7.1 and 7.3, a lawyer may advertise services through written, recorded or electronic communication, including public media. (b) A lawyer shall not give anything of value to a person for recommending the lawyer's services except that a lawyer may (1) pay the reasonable costs of advertisements or communications permitted by this Rule; (2) pay the usual charges of a legal service plan or a not-for-profit or qualified lawyer referral service. A qualified lawyer referral service is a lawyer referral service that has been approved by an appropriate regulatory authority; (3) pay for a law practice in accordance with Rule 1.17; and (4) refer clients to another lawyer or a non-lawyer professional pursuant to an agreement not otherwise prohibited under these Rules that provides for the other person to refer clients or customers to the lawyer, if (i) the reciprocal referral agreement is not exclusive, and (ii) the client is informed of the existence and nature of the agreement. (c) Any communication made pursuant to this rule shall include the name and office address of at least one lawyer or law firm responsible for its content. (1) Hypo: What if you were to advertise that you had never lost a jury trial, but you had never had one? This would be misleading and prohibited. (2) Hypo: The state bar passes a regulation that says if you have an ad of any kind, you have to put a disclaimer on it (the state bar doesn't encourage lawyer advertising, you read the ad at your own risk, etc.). This will discourage free speech - will have a chilling effect. This will still run into constitutional problems. c) Not learning in this class, but the Louisiana rules are much more detailed on this subject.

Rule 1.1 Competence

A lawyer shall provide competent representation to a client. Competent representation requires the legal knowledge, skill, thoroughness and preparation reasonably necessary for the representation. (1) The question becomes whether a lawyer who has never handled a particular type of matter can fulfill this duty. (2) Cmt. 2: This is fine under rule 1.1, as long as the lawyer has the time and resources to get up to speed. b) May a lawyer turn down a request for legal assistance if he lacks time, expertise, or interest in the matter? (1) In general, lawyers are allowed to pick and choose the matters they work on. (2) They don't have to accept any particular clients, with three exceptions: (a) Rule 6.1 addresses the lawyers' duty to provide legal assistance to those who cannot afford it (encourages 50 hours a year of pro bono work). (b) A court may assign a lawyer to represent an indigent criminal D, even if the court does not have the resources to pay the lawyer for the work. Rule 6.2 requires lawyers to accept the assignment, except for good cause. (c) A lawyer may not discriminate on the basis of race, religion, nationality, sex, age, disability, or other protected category in her decisions about which clients to represent.

the law governing lawyers motions to disqualify for conflicts of interest

Judicial opinions resulting from motions by opposing counsel to disqualify lawyers because of conflicts of interests form a substantial body of lawyer law. Many courts follow their own CML standards, which may not comport with the ethics rules, when determining disqualification issues.

1. Diligence

a) A fundamental duty of lawyers to clients is to do the work they've been hired to do, and do it without undue delay. This is a distinct duty from competency. b) See rule 1.3: A lawyer shall act with reasonable diligence and promptness in representing a client. c) Cmt. 1: A lawyer should pursue a matter for a client despite opposition or personal inconvenience and take whatever measures are required to vindicate a client's cause. The lawyer must act with zeal in advocacy upon the client's behalf. However, a lawyer is not bound to press for every advantage that might be realized. The rule does not require the use of offensive tactics or preclude a lawyer from treating all persons with courtesy and respect. To be good, you don't have to be mean. d) Cmt. 3: Perhaps no professional shortcoming is more widely resented than procrastination. The opposite of diligent representation is total neglect of clients' cases. You should take care that your reputation is protected, since your professional life is going to suck if everyone thinks you're a jerk. e) The old requirement of "zealous advocacy" should not be used to justify pursuing every possible argument or advantage in litigation, no matter how unethical. "Zealousness" was replaced with "diligence" to avoid this unethical behavior.

Imputation of concurrent conflicts

a) A lawyer can have a conflict of interest because he represents one client whose interests conflict with those of a client represented by his partner. b) See rule 1.10, although this will be discussed in greater depth in the next chapter.

1. Administering estates and trusts

a) A lawyer may accept appointment as executor of a client's estate. b) A lawyer must comply with rule 1.7 in giving advice about whom to appoint as executor or in acting as executor for a client's estate.

1. Contractual duties

a) In addition to the duties imposed by law, lawyers may sometimes undertake contractual duties that are more stringent. b) If a lawyer violates duties that are imposed only by K, the lawyer may be subject to discipline for those contractual violations (under the Restatement).

1. Conflicts in representation of a class

a) Because rule 1.7 doesn't fit very well in the class action context, courts will usually resolve any conflict by applying the court's class action rules. b) The lawyer ends up making a lot of the decisions in this context, since its hard to employ all these rules on such a large scale. c) Usually the court's attention to the class action and the class action rules in the code of civil procedure will take care of any issues. d) Examples of potential conflicts in this area include: (1) A greater concern for the interests of the class representatives than for the unnamed members of the class; (2) A prior relationship with the named D's; (3) A greater concern for receiving a fee than for pursuing the class claim; and (4) The settlement of claims by collusion rather than through a fair process where class members' interests are adequately represented. e) In re Hoffman (La. 2004)- Rule 1.8(g)- the memebers of the class disagree about the amounts the other family member receive from a class action (1) One of the family members was not aware that one family member was getting way more money than him (2) Suspends Hoffman for violating Rule 1.8(g)

1. Specific Conflicts

a) Business Transactions (1) Rule 1.8(a) (2) Lawyers sometimes have money to invest and this rule applies (3) Same if borrow form client (4) IN La lawyer disbarred for not following the rule and screwing over the client (5) The trick to try and get around à try to make a fee and get piece of action. Court did not like! If your gonna try and get piece of the action FOLLOW THE RULE. (6) There are no exceptions for things like buying a car from your client who owns dealership. (a) But the comments do! (b) Exceptions for NORMAL client activities. (c) Remember that LA did not adopt the comments. (7) FL hypo (a) Represented client in aircraft business (b) Owned company which maintained aircraft (c) This business is in completion with his client's business (d) The court said that you acquired a business in conflict with the adverse pecuniary interest of your principle client and you should have jumped through the 1.8(a) hoops (e) So what if I want to get Honda dealership and client owns Chevy dealership. This is ok. (8) Client is a borrower and borrowing money from a lender (a) Note and mortgage going back to lender (b) It is going to default (c) So you want to buy the note and mortgage for 50 cent on the dollar (d) So now you the lawyer are the creditor for your OWN client (e) This is pretty adverse to the clients adverse pecuniary interest b) Gifts (1) Rule 1.8(c) (2) Prohibits soliciting substantial gifts from your clients (3) You are bonehead! Clients do not really like this. (a) Trivial gifts (4) If the paperwork is involved in trafering the vehicle (in this hypo) then it's good (may want to send a thank you note to just to clarify there was an exchange of goods) c) What about wills and acts of donations? (1) Rule 1.8(c) (2) If they want to give you something than another lawyer has to make it! (3) LA hypo à Client wants to give gift to the lawyer. He wrote it in and said if it's invalid than it goes to his wife. Heirs not good with this. Challenge. Rule 1.8(c) invalidates the donation because force and effect of substantial law. Lawyer claims now only can punish! Cannot take my property. He tries to claim that he did not know. They don't care and disbar him! SO DON'T DO THIS. He lost the donation and his practice. Just send it out and protect yourself and your donation. d) Sex (1) Rule 1.8(j) (2) Sex itself is not prohibited (a) You are fiduciary for your client. Don't let the men pick on vulnerable female clients. Like for example divorce clients. (b) Lawyers offer discount for sex acts! Or for late fee payments.... (3) Ok if you were engaged BEFORE you became the client. (4) LA does not have this rule. But still will go after them with other conflict rules. None of it turns out well for the lawyer. (a) Not representing client well b/c you have personal interest in client's affair

Representation of the entity and employees

a) Can a lawyer who represents a corporation provide legal services to individual employees of the organization? YES, unless the interests of the employee and the organization conflict. (1) Also depends on the reasonableness of the client and his belief if you are representing him/her (2) R. 4.3 b) If consent for such a conflict is required, a designated corporate official can give this consent for the corporation as long as he's not the one being individually represented. c) Where the individual and the corporation would assert the same defense to an accusation, there might be no serious conflict in the lawyer representing both parties. However, the lawyer should obtain informed consent from both due to the possibility of divergence later on. d) In derivative suits, the lawyer for the organization should not undertake to represent the D's, even with client consent (this is not true in La.). Exception: If disinterested directors conclude that no basis exists for the claims and all relevant persons consent.

A. Lawyer as Custodian of Client Property and Documents 1. Client trust accounts

a) Client trust account: The bank account in which the lawyer keeps funds that belong to various clients. b) This is where most lawyers are disciplined - from taking money out of client trust accounts. c) Under rule 1.15(a), if a lawyer takes possession of money from a client or third party in connection with representation, she must keep it "separate from the lawyer's own property. Funds shall be kept in a separate account maintained in the state where the lawyer's office is situated, or elsewhere with the consent of the client or third person." (1) Property other than money must be appropriately safeguarded, and the lawyer must keep complete records of the funds or property for a period specified in state rules after the events that they record. (2) The lawyer must keep detailed records of deposits into and withdrawals from the client trust account. (3) The lawyer keeps something called an operating account, where the lawyer makes withdrawals to pay the secretary's salary, light bills, etc. The money in this account CANNOT be commingled with the money from the client trust account. (4) Funds from multiple clients may be kept in the same account, but the lawyer's funds may not commingle with those of the clients'. Lawyers cannot hide their money from creditors in their client accounts. (5) A lawyer may be disciplined for commingling or misappropriating client trust funds even if the violation was unintentional, if no client funds were lost and even if the lawyer's mental capacity is compromised by illness. (6) DO NOT SPEND ANYTHING OUT OF YOUR TRUST ACCOUNT! (7) The only thing you can do is transfer to operating account and than spend once its determined that you are entitled to them. d) IOLTA: What happens to the interest on the client trust account? (1) Interest on Lawyer Trust Accounts (a) This is not chump change! This can be some big bucks. (2) Traditionally: (a) The trust account was noninterest bearing (banks liked this). (b) Two exceptions: (i) If a large amount was going into the trust account for a long period of time, then the lawyer had a fiduciary duty to tell the client that he could put it in an interest bearing account. (ii) Where the client insisted on receiving interest. (3) Under the IOLTA regime, the interest accruals would go to the state bar association. At first this was voluntary, but later became mandatory. (a) The client will only receive interest where he insists on it. We're not sure if the lawyer has a fiduciary duty to tell the client about interest bearing accounts now that we have the IOLTA scheme. (4) There is a caveat to La.'s (1.15(g)) IOLTA rules: ". . . traditional attorney-client relationships do not compel attorneys either to invest clients' funds or to advise clients to make their funds productive." Lawyers don't have to tell clients that they could get interest on their accounts. S thinks this is wrong, if lawyers truly have a fiduciary relationship with their clients. (5) The state bar, working under the order of the LASC, is taking money that is generated by private funds. Does this violate the takings clause? The US SC has said that the interest generated from client funds is client property, but didn't actually rule on the takings question. (6) Under the Wash. IOTLA scheme, the client gets the interest where the bank fees don't eat it up. In that instance, it was held that there was no taking. However, the La. scheme doesn't provide for this, so there might still be a valid IOLTA claim there. Big commercial enterprises would likely want to attack this, since IOLTA money goes to funding indigent clients that are usually suing these big companies. (a) Even in La., under our mandatory IOLTA scheme, if the client insists on receiving the interest, then they can. But it's murky as to whether the lawyer has a duty to tell them about it. (7) If the trust account gets overdrawn that the ODC gets a notice! Why because this means the lawyer might be dipping into the account. (8) Big deal for lawyers in private practice

1. Suing a current client

a) Clients can be directly adverse to one another, even when the cases in which the lawyer represents the clients are unrelated. For example, you cannot represent the husband in a divorce proceeding, and then represent another client in a car accident case where that client is suing the husband from the divorce proceedings. b) The general rule is that you cannot sue a current client without their consent. In some jurisdictions this is absolutely prohibited. 5-2: I Thought You Were My Lawyer! (1) You have agreed to take on H in a divorce case. Your firm is representing W in another matter (she was hit by a bus). While her case is pending, you take on the H's representation in the divorce. (2) This is a direct concurrent conflict - you are suing your own client. (3) Is divorce directly adverse? Is it really litigation? The income from the settlement wouldn't be community property. What about damages for loss of consortium? This is a direct conflict. (4) Imputed conflict—can't just get another lawyer in your firm to take on representation of one of the clients. Rule 1.10. (5) What if she was going by a different name in the bus case? Even if the firm didn't mean to initiate the conflict, you must still take steps to resolve it. (6) 1.7(b)—(1) if the lawyer can still provide competent representation then it is okay if the parties consent. (7) Former client rule- if we dropped the W and still represented the W then we would be suing a 'former client'. 1.9à some issues may arise. (8) If they H and W both want you to represent thenà you still have to sue somebody. How do you sue your own client?? (9) The result might be pulling punches - you may not be as aggressive against the W since she's hurt or you might be more aggressive if you think she's going to recover a large settlement from the bus matter. (10) This would also be an instance of imputed conflict under 1.10. If one lawyer in a firm has a conflict, then so do all the others. (11) In the real case, the H got really mad when he found out about the conflict. The firm didn't make any disclosures or obtain any consents before taking on the H's representation. H fired the firm, and sought to recover the expenses he paid to it. (12) Attorney Client Privilege—if parties agree to represent them jointly, there is no privilege claim anymore in anything now or in the future. (13) Confidentiality 1. Cross-examining a current client a) A lawyer can have an adverse relationship to a current client in litigation, even if the client is no a D in the case. This might happen where the lawyer is representing one client, and then another one of the lawyer's clients is called as a witness. b) This comes up a lot in the criminal defense context.

Representing economic competitors in unrelated matters

a) Cmt. 6 of Rule 1.7: Simultaneous representation in unrelated matters of clients whose interests are only economically adverse does not ordinarily constitute a conflict of interest.b) While it may not be a violation of ethical rules, it may be a breach of a lawyer's fiduciary duty to avoid representation that involves conflicting interests.

1. Competence in criminal trials

a) Denial of effective assistance of counsel: Dissatisfied clients seek redress in a number of ways, but when a criminal D has been convicted, he will sometimes file an appeal urging that the conviction should be reversed because the trial lawyer was incompetent. b) Criminal D's have a constitutional right to effective assistance of counsel. State law may provide even greater protection to criminal D's. c) Most judges are very reluctant to overturn a criminal conviction because a different lawyer might have done a better job for a D - the lawyer's performance must have been really awful to overturn a conviction. d) A D appealing a conviction must prove that: (1) The assistance was unusually poor and that (2) Better representation would have made a difference. e) Strickland v. Washington (1984) (1) Lawyer didn't ask for character references and criminal D ended up being convicted of murder and sentenced to death. The client sues his lawyer alleging ineffective assistance of counsel under the 6th amendment - the D is just trying to lessen his sentence from death to life in prison. (2) Standard: "A convicted D's claim that counsel's assistance was so defective as to require reversal of a conviction or death sentence has two components: (a) Counsel's performance was (1) deficient, or in other words, counsel made errors so serious that counsel was not functioning as the "counsel" guaranteed the D by the 6th amendment; and (b) The deficient performance (2) prejudiced the defense, or in other words, that counsel's errors were so serious as to deprive the D of a fair trial." (3) What if the client meets this standard? Then he will get a rehearing - you redo what was screwed up the first time. Note that this is not a malpractice suit or a discipline proceeding. This is just another kind of claim to look at the same problem. (4) Note that under this standard, the lawyer only gets punished where his incompetency has harmed the client in some way. (a) He didn't interview the family (b) He didn't interview acquaintances (c) He didn't get psych evaluation (5) We should be highly deferential in reviewing the performance of an attorney. Hindsight is 20/20. Note J. Marshall's dissent, that says we should not be so deferential to the judgment of lawyers. The statistics after the case suggest that we shouldn't defer to attorneys. (6) LASC - All criminal defenses in Orleans parish is defective and there has to be a showing that it's not... basically call to Congress to allocate more money. This was about 10 years ago

1. Imputation of conflicts among lawyers sharing office space

a) If lawyers are practicing as a law firm, it is obvious that imputation rules will apply. b) Imputation rules may also apply where lawyers share office space without any formal association of their practices. c) 6-4: The Fatal Shot (1) Two sole practitioners do criminal defense work and share a suit of offices to cut down on operating expenses. Each lawyer takes on D in the same matter, and each D blames the other for the shooting. (2) Are they office sharing? If so, then what. If not, then what? Do office sharers constitute a "firm?" under rule 1.10. If they aren't really a firm, we don't impute the conflict, so it's not a problem that they are represented by the 2 different attorneys. They are purely office sharers and would need to make sure the secretary keeps info separate as not to violate confidentiality. (3) Are these two attorneys treated as being in the same firm? They share a secretary and have access to each others' files. The secretary alone would probably be enough to cause a conflict, since she probably has intimate knowledge about both sides. (4) The lawyers in this situation must take precautions to protect the confidential information of each lawyer's client. They could ask the secretary to only work on things that don't involve confidential information, but this might be impractical. (5) Suppose that one lawyer has gone out of town to a funeral, and is supposed to make an appearance in court for this matter. One lawyer asks the other to represent his client at the hearing. Could this lawyer help out the other? This is problematic, because the two D's have divergent interests. What happens in this instance if that lawyer at the hearing fires the first lawyer's client? This client will become a former client under 1.9. This is very bad under 1.7 and/or 1.9. The client and former client's interests are materially adverse here - this lawyer has backed himself into a corner with a conflict by doing a favor for his fellow lawyer. (6) What if the two solo practitioners decide to form one firm? Then the two D's interests are still divergent (same matter) and this becomes a conflict under 1.7.

Duty to protect confidences of employees

a) If the lawyer only represents the organization and not any employee, then the lawyer generally has no duty to protect the employee's confidences. b) The lawyer should advise the employee that a potential conflict exists and that discussions between the lawyer and the employee are not confidential. If the lawyer fails to give such a warning, and the employee has an expectation of confidentiality, then the lawyer might have inadvertently created a lawyer-client relationship, with a possible concurrent conflict of interest

1. Client waiver of imputed conflicts

a) Imputed conflicts can usually be solved by informed consent from the affected former client. b) The analysis for waiver here is the same as that for concurrent conflicts under Rule 1.7(b). c) If an imputed conflict is waiveable: (1) The lawyer must make the appropriate disclosures and (2) The necessary consent must be in writing.

1. Nonconsentable conflicts

a) In general (1) Note that most conflicts are consentable. (2) Some conflicts are so problematic, however, that the lawyer cannot continue the representation, even where the client wants him to. Go through the steps in 1.7(b) to determine whether the conflict is consentable. b) The lawyer's reasonable belief (1) A lawyer must consider "whether the interests of the clients will be adequately protected if the clients are permitted to give their informed consent to representation burdened by a conflict of interest." (2) OBJECTIVE STANDARD: Can the lawyer reasonably conclude that he will be able to provide competent and diligent representation? (3) Ask: Would there be an adverse effect on the: (a) Relationship with either client? (b) Representation of either client? (4) These are some factors that might help to determine the effect on the relationship/representation, and whether it may be consentable: (a) Factual relation or connection between the matters. If not factually related, then the conflict may be waivable. (b) Joint representation of two parties with very divergent interests. If so, it might not be possible to pursue the interests of one without harming the other. (c) Two present clients or one present and one former client? In the latter case, all conflicts are consentable. (d) Friendship or bond of professional loyalty between the lawyer and one of the two clients? If so, the lawyer may not be able to provide representation to the other, especially if the interests of the two are divergent. (e) Sophistication of the client. If highly sophisticated, then consent will probably solve the problem.

1. Representing insurance companies and insured persons

a) Insurance policies usually provide that if the insured is sued over an event covered by the policy, the company will provide the insured with a lawyer to defend him. The company will cover the damages up to the amount of the policy. b) The lawyer defending the insured will usually be paid by the insurance company and the lawyer's contact will usually be the insurance adjuster. The insurance company's interests may often by divergent of those of the insured. c) Who is the client? The lawyer is being paid by one client (insurer) to represent both the insurer and another client (insured). (1) See rule 1.8(f). (f) A lawyer shall not accept compensation for representing a client from one other than the client unless: (1) the client gives informed consent; (2) there is no interference with the lawyer's independence of professional judgment or with the client-lawyer relationship; and (3) information relating to representation of a client is protected as required by Rule 1.6. (2) Cmt. 13: If acceptance of payment from someone other than the client presents a significant risk that the lawyer's representation of the client will be materially limited by the lawyer's own interest in accommodating the person paying the lawyer's fee or by the lawyer's responsibilities to a payer who is also a co-client, there is a conflict under rule 1.7(b). The lawyer must assess whether the conflict is consentable and seek informed consent from the client. (3) Restatement: It is clear in the insurance situation that a lawyer designated to defend the insured has a lawyer-client relationship with the insured. The insurer is not, simply by the fact that it designates the lawyer, a client of the lawyer. (4) The insurance company may be a client, depending on K or state law. However, the insured is always a client. Even so, the Restatement provides that communications between the insurer and counsel will be privileged, and the insurer may sue the lawyer for negligence. d) Most case law says that it's ok for one lawyer to represent both the insurer and the insured as long as no conflict exists. e) While the lawyer owes the insured a duty of competency in representation, the K between the insurer and the insured may delegate to the insurer authority to make decisions about discretionary efforts or expenses in litigation. If the insured might incur liability exceeding the policy limits, the lawyer may not follow a direction by the insurer that would substantially increase the risk of such liability. f) What if the lawyer learns information from the insured that, if known to the insurer, would cause the insurer to deny coverage? The lawyer must keep the information confidential, even where the insurer is a client of the lawyer. g) If there is a conflict, then the lawyer should act in the best interests of the insured, except that the lawyer may not assist client fraud. If the insurer is also a client of the lawyer, then the lawyer should act in the best interest of both clients. If this is impossible, then the lawyer must withdraw. h) Some insurance policies provide that where there is a conflict between the insurer and the insured that would result in withdraw, then the insurer has to pay for separate counsel of the insured. i) If the insurer and insured disagree about when to settle, then the lawyer's obligations to the insured should govern the lawyer's conduct. An ABA opinion concludes that if the insurer and the insured disagree about whether to settle, the lawyer must withdraw from representing both of them in that matter. Once the insured becomes a former client, the lawyer still cannot assist the insurer in settling the matter against the interests of the former client. j) In La., we have the direct action statute, where the P can sue the D and the insurance company, or either, directly. The jury knows about the insurance company the whole time (would be cause for a mistrial in other jurisdictions) (1) In many other states, the custom practice is that if there is a hint in the trial that the insurance company is going to pay for the damages, then it will be declared a mistrial. (2) In La., we don't have the luxury of saying that your client is just the insured. The jury will always know that the insurance company is involved and paying for all or part of the fees.

I. Relationships Between Lawyers and Clients A. Formation of the lawyer-client relationship Choosing clients

a) Is a lawyer permitted to accept work that requires knowledge of an area of the law in which the lawyer has no experience? (1) YES, if the lawyer compensates for the inexperience through study or affiliation with another lawyer. (2) The rules require lawyers to provide competent representation. See Rule 1.1:

A. Terminating the Lawyer-Client Relationship 1. Duties to the client at the conclusion of the relationship

a) Usually the relationship ends when the matter has been completed. However, the lawyer has an indefinite duty to protect client confidences. b) When work is finished, the lawyer must return to the client "any papers and property to which the client is entitled" and return any unearned fee. c) Rule 1.16 discusses termination of the relationship:

informed consent

a) It is ultimately the clients' decisions whether to waive the conflict by giving consent. If the client declines to give consent, the lawyer cannot take on or continue the conflicting work. b) Definition of informed consent (1.0(e)): "An agreement to a proposed course of conduct after the lawyer has communicated adequate information and explanation about the material risks of and reasonable available alternatives to the proposed course of conduct." c) Informed consent may require divulgence of confidences about the other client, in which case the permission of the latter to disclose the information should be obtained first. If this client declines to give permission to make the disclosures, then the lawyer may not proceed with the representation, since the lawyer won't be able to get informed consent from the other client. d) The lawyer must orally explain to the client the risks, advantages and possible alternatives to the lawyer going forward with the representation. e) Note that the consent by the client must in writing. The lawyer may not assume consent if the client does not respond to a letter disclosing the conflict. f) What if a client gives informed consent, and then wants to take it back later? That's fine - a client always has the right to fire a lawyer. A client who has waived a conflict may revoke the waiver. Depending on the situation, the lawyer may be able to continue to represent the other client, or may need to withdraw from representing both. The lawyer may want to work this out ahead of time by asking who he could continue to work with if a conflict arose. g) A client may give an advance waiver, but this depends on a number of factors: (1) How well the client understands the risks of possible future conflicts; (2) The thoroughness and specificity of the lawyer's disclosure; (3) The client's experience with legal services; (4) Whether the client received independent legal advice before giving the advance waiver; and (5) Whether the conflict that arises may be solved by consent. If the conflict is nonconsentable, then the waiver will not be valid as to that particular conflict.

1. Entity lawyers on boards of directors

a) Lawyers are not forbidden to sit on the boards of directors of organizations that they represent, and while there usually is no conflict of interest, there can be. b) Under the Restatement, if a conflict arises that presents "a substantial risk that the lawyer's representation of the client would be materially and adversely affected by the lawyer's obligations as a director, the lawyer should cease to represent the corporation on that matter unless the organization waives the conflict. Alternatively, the lawyer could resolve the conflict by resigning from the board. c) Note that if a lawyer is also a director of an organization, communications with him may not be protected by attorney-client privilege. d) 7-2: My Client's Subsidiary (1) You represent Dori, a P who was hit by a bus and sues the bus company. The bus company is a subsidiary of a parent corporation which is your client. (2) There are a number of factors to determine whether a related entity is a client (see above). (3) If you had been asked to sue the parent company, then this would have been a direct concurrent conflict. Here, this is only an indirect conflict. (4) This suggests that there is some common management elements between the parent and subsidiary corporations. You might be able to solve this with informed consent under 1.7(b), but you'd have to talk with the parent corporation, who is unlikely to give consent. The only solution may be to withdraw. e) Hypo: You do environmental work for a corporation. The compliance officer of the corporation says that they are having trouble with the expense of environmental compliance, and they have dumped hazardous waste in the night to save on these costs. What do you do as the attorney for the corporation? (1) The corporation is your client, not the compliance officer. You must keep the corporation's best interests in mind. Recall that under 1.13, you represent the entity and not its employees. (2) You have an obligation to protect the corporation. What happens if you report this misconduct to the board of directors and they tell you to shove off? (3) See 1.13(c): The lawyer reasonably believes that the violation is reasonably certain to result in substantial injury to the organization, then the lawyer may reveal information relating to the representation whether or not Rule 1.6 permits such disclosure, but only if and to the extent the lawyer reasonably believes necessary to prevent substantial injury to the organization. (4) You can seek help from outside authorities, regardless of whether 1.6 prevents the disclosure or not. This is an exception to the confidentiality rules. It won't be mandatory to go to outside authorities, but you may do so in order to protect the organization. (5) This may also fall under the "substantial bodily harm" exception under 1.6, since the hazardous waste might cause harm to those living by the corporation's plant.

1. Solicitation

a) Ohralik v. Ohio State B. Ass'n: Ohralick out of Ohio who was an ambulance chaser. He went to the hospital after an accident, and signed up the victim as his client. He also signed up the driver of the car - might be a conflict. This goes to SCOTUS - this is not Bates. This is no longer advertising, but in your face, personal solicitation for personal gain. The person receiving it cannot just turn off the TV - trained advocates can steamroll people. The court can use prophylactic prohibitions. This is not limited to in your face solicitations - on the phone, in chat rooms. These communications can be prohibited where they're for personal gain. (1) Solicitation can be prohibited. b) Premus: Wanted to sterilize everyone that was getting welfare, particularly women. Premus calls up one of these women, and says that he wants to represent her. Premus is an ACLU lawyer (pro bono). SCOTUS distinguishes Ohralick, since this was not for pecuniary gain. (1) Solicitation of pro bono work cannot be prohibited unless the behavior still exhibits what the rule was intended to stop (duress, vexation of client, etc.). (2) Trying to assert constitutional rights, so that is ok! c) See rule 7.3

Taking inconsistent legal positions in litigation

a) Positional conflicts: While a lawyer cannot advocate on behalf of one client against another client, normally a lawyer may make inconsistent arguments on a legal issue in different courts at different times without violating the conflict rules. b) This will usually only present a serious conflict of interest where there is a high likelihood that one client would be materially harmed by a lawyer's making an argument in another case that was contrary to the client's interest. This might happen where a decision favoring one client creates a precedent likely to seriously weaken the position taken on behalf of the other client. c) The comment to rule 1.7 and the Restatement identify factors that are to be considered in making this assessment: (1) Whether the issue is before a trial or appellate court (2) Whether the issue is substantive or procedural (3) The temporal relationship between the matters (4) The practical significance of the issue to the immediate and long-run interests of the clients involved (5) The client's reasonable expectations in retaining the lawyer d) Something else to consider is whether the lawyer might be inclined to "soft-pedal" or otherwise alter one or another argument to avoid affecting the other case.

A. Lawyers' Duties of Competence, Honesty, Communication and Diligence Competence

a) Recall rule 1.1, which requires legal knowledge, skill, thoroughness and reasonably necessary preparation for competent representation. Cmt. 2 says that the most fundamental legal skill consists of determining what kinds of legal problems a situation may involve. b) The comments identify various other features of competent lawyering: (1) Lawyers without experience in an area must get training or assistance to perform competently. If the work is complex or technical, the amount of study or supervision will be greater. (2) Competent performance requires diligence and thoroughness. A lawyer must inquire into and analyze the factual and legal elements of the problem. c) The "MacCrate Report" by the ABA task force expands on this, listing the top ten lawyering skills and four lawyering values. d) Bottom line: All the basic skills that people need to do good work are essential to good lawyering also. e) Matter of Neal (N.M. 2001) (1) This was the case where a statute provided for an automatic appeal in a murder case, so after his client was convicted, the lawyer didn't do anything. (2) Research, analysis and timeliness are fundamental in the legal profession. No lawyer should approach any task without knowledge of the applicable statutes, court rules, and case law, especially in matters with which one is not intimately familiar. (3) The lawyer offered up for mitigation the fact that at the time, he was diagnosed with depression. The court doesn't seem to care though. (4) Other cases with mitigation factors: (a) La. case about alcoholism: (i) There was a LASC case where a lawyer fired a gun out on Airline Hwy while he was drunk. He was a diagnosed alcoholic. The court held that alcohol can be a mitigating factor IF: (a) The alcoholism caused the conduct and (b) Since the time of the incident, the lawyer has taken real, concrete steps to overcome the problem. (ii) The court in that case noted that the evil of alcohol abuse had become "ascendant" in the legal profession - huge numbers of lawyers have this problem. (iii) In response, the state bar set up the Lawyer's Assistance Program (LAP) to help alcoholic lawyers. (b) Ohio case about sex addition: (i) The lawyer was kicked out of the practice. (ii) The court said that he hadn't taken any steps to mitigate the addition, and was only using his condition as leverage in the disciplinary proceeding. (iii) In a similar case, where a lawyer was groping women in bars, he went to AA and moved to a city. In that instance, the court only put him on probation. (5) The lawyer also argues that he wasn't paid much of a fee, so not as much work was required. This is no excuse to be incompetent. A pro bono lawyer is held to the same standard of competency as a paid lawyer.

1. Responding to unlawful conduct by corporate officers and other employees

a) Recall that the lawyer's duty is to the organization, NOT to the senior executives. b) The lawyer should usually report the misconduct to higher authority within the organization, and if need be, to the highest authority that can act (board of directors). If this authority refuses to act, and the lawyer believes the misconduct will result in substantial injury to the organization, rule 1.13(c) permit the lawyer to reveal the misconduct to public officials. (c) Except as provided in paragraph (d), if (1) despite the lawyer's efforts in accordance with paragraph (b) the highest authority that can act on behalf of the organization insists upon or fails to address in a timely and appropriate manner an action, or a refusal to act, that is clearly a violation of law, and (2) the lawyer reasonably believes that the violation is reasonably certain to result in substantial injury to the organization, then the lawyer may reveal information relating to the representation whether or not Rule 1.6 permits such disclosure, but only if and to the extent the lawyer reasonably believes necessary to prevent substantial injury to the organization. a) What if a lawyer is fired for reporting executive misconduct to higher authorities or public officials? The lawyer is directed by 1.13(e) to inform the organization's highest authority of the discharge or withdrawal.

1. Regulation of hourly billing and billing for expenses

a) Recent unethical billing practices include: (1) Inflating the amount of time recorded (2) Working more hours than is necessary to complete the task (3) Billing for time spent doing ministerial tasks (4) Billing for personal expenses, or for legitimate expenses but at a higher rate b) In response to these unethical practices, many firms have written billing policies and guidelines other than those imposed by clients. c) Clients are also demanding more detailed time records, with entries other than "professional services rendered" or "research." d) S told a story about a lawyer representing dentists in CA that billed at higher rate than the clients had agreed to, billed for work that hadn't been done yet, and sometimes billed more than 24 hours a day by bulk billing (bill 3 minutes per every page in a document). He was charged with billing fraud, and in defense, he argued that he actually did the work. A judgment was cast against him to return the unearned fees and he was disbarred by the CA SC. e) Here are some of the boundaries that have been developed by judges and ethics committees: (1) No padding or time inflation: A lawyer billing by the hour may not bill for more hours than she actually worked. Some authorities think that undisclosed rounding up combined with large billing increments impose on clients millions of dollars of unearned legal fees a year. (2) No inventing hours that weren't really worked: If a lawyer inflates his hours and then transmits the resulting bill to a client, he may not only be sued or disciplined, but also may be charged with federal mail fraud or other crimes. (3) No profits on costs: A lawyer may not bill for "overhead" or mark up costs. The lawyer may only bill for the actual cost of the service. General overhead costs should be covered within the lawyer's hourly fee, not passed on to the client. Lawyers are in the business of law, not copy centers. (4) No double billing: A lawyer may not bill two clients for one period of time. If a lawyer does work that benefits two clients, then the lawyer may bill each for ½ the time expended, but the block of time cannot be billed twice. (5) No billing a second client for recycled work: A lawyer who is able to reuse old work product has not re-earned the hours previously billed and compensated when the work product was first generated - this is luck, not hard work. (6) No churning or running the meter: A lawyer may not do unnecessary extra work in order to justify billing more hours. Cmt. 5 to rule 1.5 says that "a lawyer should not exploit a fee arrangement based primarily on hourly charges by using wasteful procedures." (7) No billing clients or the firm for personal expenses or marking up expense receipts. (8) No billing by the hour at lawyer rates for administrative services: While some administrative services may be legitimately billable at lawyer rates, query whether lawyers should bill for administrative tasks that could be delegated to a staff person. (9) Billing for time spent billing: It's not clear whether the time spent explaining billing procedures to clients or time spent preparing bills is billable. The Indiana SC said it was improper, but this opinion is ambiguous.

1. Conflicts in representing family members

a) Representing both spouses in a divorce (1) Lawyers will frequently be asked to represent both husband and wife in a divorce. (2) Some states allow a lawyer to represent both parties in an uncontested divorce. (3) Other states do not allow the lawyer to represent both H and W in the suit for divorce, but allow a lawyer to assist both parties in preparing a settlement agreement, as long as the clients agree and the resulting settlement seems fair. (4) Some jurisdictions do not allow a lawyer to represent both H and W in any divorce action, even with consent. b) Representing family members in estate planning (1) It is very common for lawyers to draft wills for husbands and wives, and for the beneficiaries of the wills to be other family members. (2) Some say that joint representation of two clients should include an agreement that information shared by one is not to be held in confidence from the other. What happen if the lawyer doesn't disclose this upfront, and then a conflict arises? (3) Florida Bar Opinion 95-4 (1997) (a) You represent both H and W in the execution of a will. H wants to leave everything to his mistress. Under rule 1.4, you are supposed to provide adequate information to your clients so they can make informed decisions. However, there is a duty under 1.6 to protect the H's confidences. Do you tell the W or do you protect the H's information? (b) Held: While a lawyer is ethically obligated to protect confidences and also has a duty to communicate to a client information that is relevant to the representation, the lawyer's duty of confidentiality must take precedence. (c) The court advises an attorney in this situation to withdraw from the representing, informing the parties that a conflict of interest has arisen that precludes continued representation of both parties. The lawyer may also want to advise that each party should retain separate counsel. As such, even though the W might suspect an affair and ask the lawyer about it, the lawyer may still not reveal the separate confidences.

Using or revealing a former client's confidences

a) Rule 1.9(c) explains the nature of the duty of confidentiality to former clients. (c) A lawyer who has formerly represented a client in a matter or whose present or former firm has formerly represented a client in a matter shall not thereafter: (1) use information relating to the representation to the disadvantage of the former client except as these Rules would permit or require with respect to a client, or when the information has become generally known; or (2) reveal information relating to the representation except as these Rules would permit or require with respect to a client. b) A lawyer must protect confidences of: (1) His own present and former clients, (2) Other present and former clients of law firms where he works or used to work, and (3) Prospective clients. (4) What about telling prospective employers who you worked with to prevent this? It is necessity. Is there a Rule 1.6 exception? NO! But the ethics committee has read it in because it is necessary to follow the rules. c) Regardless of whether the possible receipt of confidences in a prior matter precludes representation of a new client, 1.9(c) prohibits the revelation of confidences received from former clients and prohibits the adverse use of such confidences. d) 1.9 exempts from protection any confidences that may or must be revealed pursuant to other rules. e) If the information received in confidence has since become generally known, the lawyer need not keep it confidential.

Imputation of the conflict of a departed lawyer to his former firm

a) See 1.10(b). This analysis is similar to that under 1.9(b). b) Consent will be required from the former client in order for the infected lawyer's former firm to represent a new client where there is: (1) Material adversity, (2) Substantial relationship, and (3) Possession of material confidences by a lawyer in the firm. c) What is the difference between 1.9(b) and 1.10(b)? (1) 1.9(b): Whether the lawyer brings the old firm's conflicts with her. (2) 1.10(b): Whether the old firm retains the conflicts created by the work of the moving lawyer.

1. Imputation of the conflicts of an entering lawyer who is "infected"

a) The general rule is that a former client of one lawyer in a firm is imputed to the other lawyers in the firm. If one lawyer cannot take on a matter, neither can the other lawyers in the firm, unless the conflict is consentable and the relevant clients give informed consent. b) If an "infected" lawyer leaves the firm, the remaining lawyers are no longer infected by the ex-lawyer's conflict unless the new matter is substantially related to the old matter and one of the remaining lawyers possesses material confidences learned as a result of the ex-lawyer's previous work. c) If an infected lawyer enters a firm, the outcome will depend on whether the jurisdiction allows screening. Additionally, most imputed conflicts can be solved through consent. The exception is where the conflict is nonconsentable under 1.7 - other lawyers in the firm may not solve the conflict by obtaining consent. d) Even the ABA rules make an exception for law students, so as not to discourage law firms from hiring lawyers who, as students, had summer jobs at other firms. However, non-lawyer employees and law students that worked at a firm that represented a client whose interests are adverse to a client of their present firm are barred from personal participation in work for the client at the new firm. In addition, they must be screened. This is one of only THREE situations where the model rules permit screening to solve a problem. e) Comment 4 (1) Paralegals, legal sec, and non lawyers (law students) (2) Exceptions (3) Screen the person from personal participation, but no need to give notice (4) TSL Case in LA (supplement)à because this rule not adopted

4-2: The Fired Guard

a) The issue was whether the lawyer for the prison guard was able to enter into a settlement for the guard. b) Would the lawyer showing up at a settlement conference cause someone to believe that the lawyer has authority? To the third parties' credit, he did behave as though he had authority, from talking on the phone and other communications with his client. c) Case law says that lawyers do not have implied authority to enter into settlements without the client's knowledge. The lawyer must have express authority from his client in order to settle without the client being there. The question is still open in regard to apparent authority. d) Rule 1.2 (below) discusses the matters that a client is entitled to decide, and one of these includes the decision to settle. (1) Something the lawyer decides and some the client decides. Settlement with civil case it's the client's decision. Comment 3 to the rule says need express authority to settle. e) If a court were to conclude that the lawyer had apparent authority to settle, and enforced the settlement, then the client could come back and sue the lawyer for malpractice. Lawyers MUST abide by their client's decision whether to settle. f) 1.8(k) in the LA rules...even if you get power of attorney [lawyer] you need to get express permission to settle. Likely people were abusing this privilege. This rule is in the ethics rules not statute. g) At one time, the La. civil code did not recognize the theory of apparent authority, but now has a provision that seems to recognize it. See La. CC art. 2995 or 2697. Another article, however, requires express authority in order to enter into a "compromise." Is a compromise the same thing as a settlement? S thinks so, but we're not sure what effect the different terms might have. Need express authority to enter a compromise and includes settlement of litigation in encompassed in the compromise. Most jurisdictions go with express authority to enter settlements.

1. Withdrawal and disqualification

a) The lawyer will have to withdraw if either: (1) The lawyer discovers a nonconsentable conflict OR (2) The relevant client declines to consent. b) This action can be taken on the lawyer's own initiative or by opposing counsel by a motion for disqualification. c) Sometimes motions for disqualification are filed not because opposing counsel is actually worried about a concurrent conflict, but because they want to obstruct litigation or obtain a strategic advantage by forcing a party to change lawyers. For this reason, courts review these motions very carefully. d) Sometimes large corporations, or even individual clients, will needlessly engage law firms, or just go to a prospective interview, just to strategically conflict out firms that may be used against them.

A. Conflicts in Representation of Organizations 1. Who is the client?

a) The lawyer's client is the corporation itself, not the officers or shareholders, and not other corporations owned in part or in whole by the corporation. b) See 1.13(a): A lawyer employed or retained by an organization represents the organization acting through its duly authorized constituents. c) Because lawyers who represent corporations develop close and cooperative relationships with corporate representatives, it may sometimes seems as though those individuals are "the client," but it must be remembered that the lawyer's obligations run to the organization itself. d) Facts affecting whether a related entity is a client (1) Related entity more likely to be a client if: (a) The lawyer received confidential information from or provided advice to the subsidiary. (b) The entity was controlled and supervised by the parent organization. (c) The original client could be materially harmed by the suit against the subsidiary. (2) Related entity less likely to be a client if: (a) The lawyer no longer represents the initial corporate client. (b) The two entities were linked after the lawyer began representation of the corporation. e) Rule 1.13 applies the same principles whether the lawyer is representing a business or a nonprofit organization. All business forms and organizations are covered by the rule.

1. Offering advice as the basis for a lawyer-client relationship

a) The lawyer-client relationship can be formed without an agreement being signed and without a fee being paid. A lawyer should be very careful about what casual advice or assistance you provide to others. b) Togstad v. Vesely, Otto, Miller & Keefe (Minn. 1980) (1) Clamp placed in Mr. Togstad with an aneurism in his artery that probably caused him to have a stroke and become wheelchair bound. (2) Mrs. T and her husband's boss go to talk with a lawyer about a possible med mal case. The lawyer said that he didn't think they had a case, although he never conferred with his partner before making this decision. Mrs. T was not billed for the interview. Because she didn't think she had a claim, the statute of limitations runs. (3) Held: The lawyer was negligent in providing legal services, and held the firm liable for over $650K. The lawyer, while he did not consider himself to be Mrs. T's lawyer, should have informed Mrs. T that there was no lawyer-client relationship. (4) Advice was given to Mrs. T here, so that was enough to establish the lawyer-client relationship. A retainer agreement is not necessary to commence the relationship. (5) S thinks the standard for determining whether there is a lawyer-client relationship is whether the client reasonably believed that you are his attorney. (6) Note that the court here held the lawyer to not only be Mrs. T's lawyer, but also Mr. T's lawyer, even though he wasn't there (he was still in the hospital at the time of the interview). (7) In this case, it seems like the lawyer just wasn't sure whether he wanted to take the case or not. If that was the situation, then the lawyer should have been completely honest with his client, and told her thank you and that he'll get back with her. (8) Remember that informal chats on the street can give risk to an attorney-client relationship, so be careful! The client reasonableness. (9) The LASC has even more frightening formulations of this relationship: The client's subjective belief will largely determine the existence of the relationship. Some recent cases have suggested an objective standard, while others suggest both a subjective and objective prong. When the Supreme Court hammers them out it looks to the reasonableness. See 1.18

1. Payment of fee by a third party

a) There are restrictions on allowing one person to pay a lawyer to represent another person. b) Rule 1.8(f) allows a third party to pay a lawyer's fee, but only if: (1) The client consents after being advised, (2) The third person does not direct the lawyer's decisions or otherwise interfere with the representation, and (3) The lawyer avoids sharing with the third person any confidences learned in the court of representation.

Analyzing former firm conflicts

a) To know if a lawyer has acquired material confidences, one must analyze the specific facts relating to the lawyer's access to or information about the relevant matter. b) Cmt. 6 of rule 1.9: Certain assumptions can be used to facilitate this analysis. For instance, if a lawyer had managerial responsibility they may be presumed to have received confidential information about all firm matters. On the other hand, one might assume that a junior lawyer only had access to or information about matters that she worked on or only matters handled by her department. c) Because it's difficult for the moving party to get the information needed to prevail, the firm whose disqualification is sought should have the burden to prove that the lawyer who changed firms does not possess confidential information that is material to the new matter. d) Cmt. 3 of rule 1.9- matters are substantically related for the purposes of this Rule if they involve the same transaction or legal dispute or if there otherwise is a substanical risk that confidential factual ifnromation as would normally have been ontained in the prior representation would materially advance the client's position in the subsequent matter.

How to evaluate conflicts

a) To resolve a concurrent conflict under 1.7, a lawyer must: (1) Clearly identify the client(s); (2) Determine whether a conflict of interest exists; (3) Decide whether the representation may be undertaken despite the existence of the conflict (is it consentable?); and, (4) If so, consult with the clients affected under 1.7(a) and obtain their informed consent, confirmed in writing. b) Where a conflict is not consentable, the lawyer must decline to accept representation of one or more clients. If a nonconsentable conflict emerges after a lawyer has undertaken representation, the lawyer might be able to avoid the conflict by withdrawing from the representation of one of the affected clients. Sometimes, the lawyer will have to give up both clients. c) Note that it is not sufficient that there be no conflicts at the outset of the representation. A conflict later down the line will still cause you to violate the rule. You must project and anticipate conflicts.

6-1: The Injured Passengers, Scene 1

a) Two passengers who don't know each other get into a taxi. The taxi driver causes an accident. They both approach the same attorney to represent them jointly. b) Is there a conflict?(1) It is possible that the two passengers will have divergent interests. (a) Divergent interest if: Defendant willing to settle more with one if sacrifice some money from the settlement of the other.(2) What if one of the passengers was drunk when she got into the taxi and helped to cause the accident? (3) What if one is forced to testify against the other? Since one was drunk, they may have different perspectives of the facts. You can't cross examine your own client (might use confidential information you received from the attorney-client relationship to the disadvantage of that client's). c) Suppose that one client is hurt much more than the other client. How do you decide to split the settlement amount? Under 1.8(g), you must tell each client the whole deal and get written, informed consent. Conflicts Between Current Clients in Civil Litigation

1. Restrictions on contact with represented parties

a) What should be a lawyer's relationship to a nonlawyer who is involved in a matter in which the lawyer is representing a client? b) See rule 4.2, which addresses restrictions on contact with a person (not only adverse parties) who is represented by a lawyer. Rule 4.2 Communication With Person Represented By Counsel In representing a client, a lawyer shall not communicate about the subject of the representation with a person the lawyer knows to be represented by another lawyer in the matter, unless the lawyer has the consent of the other lawyer or is authorized to do so by law or a court order. c) Hypos from class (1) Hypo: A lawyer tells his paralegal to call up opposing counsel's client and get information from him. You cannot violate this rule through an agent. Even if you just send a letter to the opposing counsel's client, it will still violate the rule, it is still a communication. (2) Hypo: Conflicts aside, suppose that you are making a will for a client that is represented by someone else in a different matter. You can talk with the client about the will. (3) Hypo: You're concerned that the lawyer on the other side is a drunk. You've made several offers to him, but you don't think any of these have reached his client. You tell your client about this, and the client offers to call up opposing counsel's client. Is there a problem with that? No, because the client is wanting to do this himself. If its on the own client's initiative, then its fine. The lawyer may not orchestrate the contact. d) This rule is aimed at preventing lawyers from making "end runs" around other lawyers to get information from the other lawyers' clients. e) This rule applies to ALL contacts with represented persons, not only to parties in litigation. (1) It applies whether the lawyer or the represented initiates the conversation. (2) If a represented client contacts a second lawyer to get a second opinion, the second lawyer may talk with the represented client. (3) If a lawyer starts a conversation with someone they erroneously believed to be unrepresented, the conversation must end upon the lawyer's knowledge that the person has a lawyer. f) The rule applies only to communications with persons known to be represented by a lawyer in the matter that is the subject of the communication. If the communication involves a different matter, then the lawyer may communicate with the person about even closely related subjects. g) A lawyer may NOT circumvent this rule by directing someone else to contact the represented client. Rule 8.4(a) prohibits violating the rules through the acts of another. h) Note that this rule applies to lawyers, not clients. Two people who are represented by lawyers may talk with one another without their lawyers' permission. However, if the lawyer tells her client to call the opposing party for a particular reason, then the lawyer may still violate 4.2 via 8.4(a). i) Do prosecutors' undercover investigations violate this rule? There is no blanket rule. It is argued that "authorized by law" at the end of the rule excuses compliance with 4.2. The Restatement notes that the undercover operations may violate the suspect's constitutional rights. j) Purpose is to keep slick and wily lawyers from getting information they should not. k) Does 4.2 mean that lawyers representing clients who have disputes with government agencies must contact the general counsel's office of the agency and may not make direct contact with government officials? NO - the 1st amendment's guarantee of the right to petition government will override ethics rules. However, the ABA urges that the lawyer first notify the government's lawyer of her intent to talk directly with the policy official, and give the lawyer a chance to advise the official as to what to say. l) What if a lawyer wants to contact an EE of a corporation represented by another lawyer? (1) 4.2 does impose certain constraints on lawyers who want to interview EE's of an adverse corporation without permission of corporate counsel. (2) Cmt. 7 to rule 4.2: The corporation may deny access to the EE in question if he: (a) Supervises, directs or regularly consults with the organization's lawyer concerning the matter; (b) Has authority to obligate the organization with respect to the matter; or (c) Is one whose act or omission in connection with the matter may be imputed to the organization for purposes of civil or criminal liability. (3) The proper test for barring lawyers from speaking with organizational EE's has been extensively debated. These standards will usually vary from state to state. a) Look at La.'s 4.2 (1) Our rule is a lot longer. See §(b). In representing a client, a lawyer shall not communicate about the subject of the representation with: (a) a person the lawyer knows to be represented by another lawyer in the matter, unless the lawyer has the consent of the other lawyer or is authorized to do so by law or a court order. (b) a person the lawyer knows is presently a director, officer, employee, member, shareholder or other constituent of a represented organization and (1) who supervises, directs or regularly consults with the organization's lawyer concerning the matter; (2) who has the authority to obligate the organization with respect to the matter; or (3) whose act or omission in connection with the matter may be imputed to the organization for purposes of civil or criminal liability. (2) This stuff comes out of the comments to the ABA rule. (3) When are you limited when the corporation is represented by general counsel? (b) directly addresses this under the La. rule. (4) Note that this is not the control group test. These are just people who are off the table. This may not be a director or officer, but it may be the employee that actually committed the bad act. (5) It is not always easy to determine whose acts or omissions may be imputed to a corporation. However, the rule has more teeth in La. because its not just in a comment - its actually stuck in the rule. (6) There was a case a few years ago about a lawyer that was disciplined under 4.2. She invited the P to come talk with her about the case, and the P was unsophisticated. She said she didn't know anything about 4.2. The LASC didn't believe it - this is such a basic rule. b) Hypo: Can you contact the general counsel for the corporation on the other side of the case, where corporation is represented by outside counsel? There is an ethics opinion by the ABA that says this is ok - the purpose of the rule isn't served there. The problem is that the rule doesn't say anything about this.

1. Grounds for termination before work is completed

a) When the client fires the lawyer (1) Lawyer must withdraw here (2) Clients always have the right to change lawyers (except when the lawyer was appointed - must get court approval) (3) Lawyer must also withdraw if his illness or incapacity would materially impair the representation. b) When continued representation would involve unethical conduct (1) Lawyer must withdraw if the representation would require the lawyer to violate the law (2) The lawyer may withdraw if: (a) The client has already used the lawyer's services to commit a crime or fraud; (b) Where he reasonably believes the client persists in a course of action that is criminal or fraudulent; or (c) If the client insists on action the lawyer finds "repugnant" (3) When the lawyer wants to terminate the relationship (a) May withdraw if it is possible to do so without material adverse effect on the interests of the client (b) It may be too late to withdraw (right before trial) or if the issue is very complex it might cost the client too much if you withdraw (4) Matters in litigation (a) The lawyer cannot withdraw without approval of the court if he has: (i) Filed suit on behalf of a client or (ii) Entered an appearance in a matter of litigation (b) The court may be reluctant to do so if the case will be delayed or if a substituted lawyer has not been arranged (5) When the client stops paying the fee (a) Lawyer may withdraw in this instance, but must first warn the client that nonpayment may lead to withdrawal (b) If the client wants to pay but doesn't have the money, the lawyer should consider reducing the fee to make the representation affordable for a client with limited means (see R. 6.1: lawyers have an obligation to spend part of their time providing pro bono services). (6) When the case imposes an unreasonable financial burden on the lawyer: may withdraw (7) When the client will not cooperate (a) May withdraw here R. 1.16 also permits withdraw for "other good cause"

A. Who Calls the Shots? 1. The competent adult client

a) Which decisions may a lawyer make without consulting his client, and which require client consultation? Keep in mind that small decisions may have substantial consequences on a case, so there is not always a clear answer to this question. (1) Lawyers have more wiggle room in the "means" category; not nearly as much control over the objectives b) Although some might argue that as the agent for his client, an attorney should consult his client about every decision, since agency law imputes lawyers' decisions to clients. This is impractical however. c) Rule 1.2 offers guidance:

Representation of co-P's or co-D's in civil litigation

a) a) If a lawyer jointly represents two clients who are both either P's or D's in a lawsuit, their relationship is not one of direct adversity, but their interests may conflict anyway. b) This might come up where:(1) One co-party has a claim against the other.(2) Both are suing the same D who has limited assets and cannot satisfy both their claims.(3) Differing views on whether to settle.(4) If both D's are responsible for the harm caused, one might try to pin it on the other.(5) The P's might disagree as to what the remedy should be (injunction or damages) (1) This comes up very often in car accident cases, where two passengers or a passenger and a driver of one car sue the driver of the other car involved in the accident. While some cases have held that a driver and passenger have sufficiently divergent interests that they cannot be represented by a single lawyer, the Restatement takes the view that this type of conflict is waivable by the client after full disclosure.

Conflicts in public interest litigation

a) he different groups interested in the litigation may have opposing goals in this context. Many times, both groups cannot prevail. 5-4: The Prisoners' Dilemma(1) You are a lawyer in legal services. You are handling two different cases - woman's prison and a mental health facility. The women are in really terrible facilities. The offer is to place some women in the mental health facility.(2) While this is a decent offer for the women, the people in the mental health clinic might be really pissed off.(3) Can this be resolved with informed consent under 1.7(b)? (4) Note that legal services will be considered a "firm" under 1.0(c), and therefore the imputation rules apply. (5) If both sides agree to the offer, then there won't be a conflict where everyone gives informed consent. What if both sides hate the offer? Technically there is no conflict between the clients; however, in the actual case, the court held that there was still an irresolvable conflict in this instance.

Torture: post 9-11, the CIA caught a terrorist and wanted permission to waterboard him. What kind of legal advice must we give? Under 2.1> "candid advice"...

· It's okay to mention moral views in the response · Rule 2.1- In representing a clinet, a lawyer shall exercise independent professional judgement and render candid advice. In rendering advice, a lawyer may refer not only to law but to other considerations such as moral, economic, social and political factors, that may be relevant to the client's situation.

a) 3-3 The Fatal Bus Crash

(1) A man was injured in a bus crash that had bought a life insurance policy previously. The insurance company refused to pay the claim because the man died from "being taken off life support" rather than directly from the bus crash. The P's lawyer suspects that the insurance company is guilty of fraud. Are communications between the insurance co. protecting the fraud privileged? (2) While all the elements of the attorney client privilege may be met in this case, the privilege will be defeated where the client is using the attorney's assistance to cover up fraud.

Rule 8.1 Bar Admission & Disciplinary Matters problem 1-1: Pot

(1) Applicant to the bar had smoked pot on and off for the last 3 years. Should the applicant disclose this on the bar application, if the question asks: Does the applicant currently, or has been in the last 3 years, engage(d) in the illegal use of drugs? (2) While the question could be manipulated so one could say no, it would most likely be dishonest not to disclose the truth. (3) See Rule 8.1 (in connection with 1.6) This rule does not reach people unless admitted to the bar but one admitted it can reach out to the time you lied when not yet admitted. (4) An applicant for admission to the bar, or a lawyer in connection with a bar admission application or in connection with a disciplinary matter, shall not: (a) Knowingly make a false statement of a material fact; or (b) Fail to disclose a fact necessary to correct a misapprehension known by the person to have arisen in the matter, or knowingly fail to respond to a lawful demand for information from an admissions or disciplinary authority, except that this rule does not require disclosure of information otherwise protected by Rule 1.6 (5) Other options for this applicant: (a) Plead the 5th, but this would probably raise more suspicions than necessary. The bar admissions committee is most likely to investigate it on their own. (b) Sue the committee (c) Get advice from counsel as to how to respond to this question (i) Suppose the applicant discloses everything to an attorney, and the attorney responds that the use is relevant to the bar application, and if it isn't disclosed, then there is a huge risk if it is discovered the applicant was lying. The applicant rejects the advice, and lies on the application. Does the lawyer have an obligation to inform the committee that the applicant was lying? (ii) The lawyer need not disclose the information if it would violate Rule 1.6 - confidentiality of information disclosed to an attorney. The duty under 1.6 to keep the information confidential would probably prevent the disclosure to the committee. The committee would ask the client to waive. (1) Drug use at the time of reading the questions OR habitual OR just anytime in the last three years? (2) No statute of limitations on these things (application to law school or the bar) (3) Lies on your character and fitness application to the bar is a slam dunk! No admission or gets revoked! (4) If you had a question who would you ask? (a) Attorney- privileged information (b) The board (c) Find someone who has never touched marijuana to sue for you

a) Rule 19: Rules for Lawyer Disciplinary Enforcement (La.) ==> This is a very long rule that covers the ins and outs of the La. disciplinary system.

(1) Appointments: To disciplinary counsel (Plattsmire). Also to hearing committees (2 bar members and one public member). (2) Periodic assessment of lawyers: Lawyers must pay a fee to help fund this office ($200/yr). (3) Grounds for discipline Section 9 of the LA Rule 19: (a) Violation or attempt to violate one of the rules (b) Violation of the rules of another jurisdiction (c) Willfully violating a valid order of the court or the board imposing discipline. (can't ignore the ODC) (4) It is important to note that a lawyer can be punished for conduct that has no direct bearing on the practice of law, if the misconduct raises character issues that compromise the integrity of the profession. (5) The people who file these complaints are given immunity. So can't sue sister because when she files the complaint she now has immunity. (if violate usually public reprimand) (a) So can't sue for defamation to the person who brings the complaints (b) So if you sue the defamation than you have violated the rule and are now subject to discipline!!!! (c) Sec 12

a) Mental health

(1) The concern in asking to disclose this information is that perhaps some people who suffer from serious mental illness would disserve their clients because of it or pose a danger to others. (2) Recall the story of Rose Gower and the repeated harassment by the committee regarding her depression and medical treatment that had been sought 7 years before she applied to the bar. It delayed her bar application for a year! (a) Mental health issues (b) Stress and strain b/c close people died and raped (c) Sought counseling but no medicine currently (d) She still goes to therapy à ongoing problems? Better to suspend therapy? (e) Current job and been promoted. Did very well in law school. (f) Her application was on ice for a year as they continued to ask more and more mental questions. (3) While it's probably reasonable for the committee to get more information regarding the mental illness to ensure that she is fit to practice law, this tends to come down on people that honestly disclose their mental status. However, if the applicant were to lie about this, it would be really easy to prove and he would likely be denied admission. The bar was aggressive an unnecessarily harsh with Rose.

A. The Death of the Client 1. Introduction a) 3-4: The Dead Murderer

(1) There was a man dying of cancer on death row. He confesses to you that he committed another murder that is being prosecuted now. Someone else is the D in that case, even though this man was the real killer. He asks whether he would be prosecuted for confessing to the crime. The attorney knows that there is another guy that may be prosecuted for a crime that he didn't commit. (2) Since the client is asking for legal advice, this would definitely be subject the confidentiality rule in 1.6. This might fall into an exception under 1.6: if someone is being prosecuted for a capital offense, there is a chance they would be sentenced to death upon conviction, and therefore disclosure may be needed to prevent substantial harm or death. Even if they just got sentenced to life in prison, that might constitute substantial harm. (3) The man that confessed to the murder is now dead. Does the duty of confidentiality go away once the client dies? A comment to 1.6 says that the duty continues, even after the attorney-client relationship is terminated. (4) Could the information be "used to the clients disadvantage?" maybe embarrassment to his family or to his estate. (5) What is the harm if the lawyer makes the disclosure anyway? The decedent's survivors may be embarrassed and it would tarnish the reputation of the decedent (although he was already on death row). The Attorney-Client privilege also continues after death...EXCEPT California they do not allow attorney/client privilege to continue after death of the individual in question.

Dumaine case: (p.11): court focuses more on the alcohol problem. "The evil has become ascendant." The LASC court created a committee on alcohol and substance abuse. Dumaine remands for some reconsideration of issues. Dumaine fires a gun across Old Hammond Hwy, but if he can show he was drunk (an alcoholic) he may get a lesser penalty if he can prove he's taken steps to remedy the problem.

(1) These are considered mitigating circumstances that would help his cause. (2) OUTCOME: LAP (lawyers assistance program) if you or your partner are on the sauce you can call and get help. Separate from the ODC [privacy concerns you can tell the LAP people and they are not going to rat you out to the ODC]. The court does say alcohol and drug abuse can be mitigating circumstances. It will lessen the blow of discipline. If you do something (like fire a gun) but you were using alcohol which caused the misconduct AND show you have taken steps to overcome alcohol problem and show extent that you are successful [potential to repeat is low] and if it can be shown you have come to grips with problem hammer won't fall so hard. (3) Model Rule 8.4(b) professional misconduct to commit criminal act which reflects adversely... fitness as a lawyer (4) Flasher case à Sex addiction but never does anything to overcome so the court does not mitigate it. Disbarred. (5) Groper à Drunk and gropes and there are criminal sanctions and then goes 72 days for rehab and then moves to new city and new practice. Admits he did and is sorry and says he is in AA plus rehab and the court only have him one year probation.

a) What is fraud? A primer

(1) To comply with all these rules, it is necessary that a lawyer be able to determine what is fraudulent conduct and what isn't. (2) In general, the term refers to deliberate deception, but the term may be defined differently in criminal, tort, contract and legal ethics rules. (3) Rule 1.0(d) defines fraud for purposes of the model rules: Conduct that is fraudulent under the substantive or procedural law of the applicable jurisdiction and has a purpose to deceive. This definition of fraud may not clarify the meaning of the term, since there are so many different standards for conduct to be considered "fraudulent." Note that different standards may or may not require that the conduct be intentionally deceptive. (4) Fraud does not include merely negligent misrepresentation or negligent failure to disclosure material information. For purposes of these rules it is not necessary that anyone has suffered damages or relied on the misrepresentation or failure to inform. (5) In certain contexts, omissions and half-truths as well as false representations can be fraud. (6) How can a lawyer know whether he is assisting in client fraud? It depends on the context . . . (a) Before a tribunal: If the lawyer has offered false testimony, failure to disclose that fact to the court would constitute assisting fraud. The language in 3.3, "assisting a criminal or fraudulent act by the client" is not limited to the criminal law concepts of aiding and abetting or subordination. (b) In dealing with a third party: 1.2 bars only assisting a client in conduct that the lawyer knows is fraudulent. However, note that actual knowledge is not necessarily the standard - disciplinary authorities may infer from the circumstances that a lawyer did know that the legal assistance would be used for fraudulent purposes. (7) A red flag should go up any time a client's past or contemplated conduct appears to involve an intentional or knowing misrepresentation to another person. (8) While the ethics rules give lawyers discretion to reveal client fraud, they will not be protected from civil or criminal liability if they elect not to reveal the fraud. The ethics rules do not set standards for civil or criminal liability - just discipline. (9) What if the lawyer who assists in committing the fraud does not reveal the information - what other obligations are there? If the lawyer decides not to disclose the specifics of the fraud, she must nevertheless withdraw from the representation. This is required by both 1.2(d) and 1.16(a)(1). In the course of withdrawing, the lawyer may, short of actually revealing the fraud, simply "disaffirm" the document prepared that contains fraudulent content. This is referred to as a noisy withdrawal.

a) Fraud by a client, not assisted by a lawyer

(1) What if the client tells her lawyer in confidence about a crime/fraud that she has committed or is contemplating, and the lawyer has nothing to do with it? (2) Rule 1.6 does NOT allow a lawyer who has not assisted in the bad act to make a disclosure to protect another person from injury. In this instance, the lawyer may only make a disclosure to prevent reasonably certain death or substantial bodily harm (since that exception does not require the lawyer to be involved in the bad act). b) Fraud by a client, assisted by a lawyer (1) In this instance, the lawyer may be criminally charged with abetting the client's fraud, and will also violate rule 1.2(d): A lawyer shall not counsel a client to engage, or assist a client, in conduct that the lawyer knows is criminal or fraudulent. (2) What if the lawyer assists the client without knowledge that she's helping to commit the fraud? The lawyer may make a disclosure to prevent the fraud, or to mitigate/rectify the harm. However, the lawyer will not violate 1.2(d), as the lawyer did not knowingly commit conduct that she thought was criminal or fraudulent.

Hypo's from class:

(1) What if your client comes to you in a confidential setting and says that he committed murder and would like for you to represent him? The crime-fraud exception would not apply here, since this was a past crime. (2) The privilege will be broken if the client asks you to participate in the illegal act - the crime has not yet been committed yet and you are precluded from helping your client commit a crime. (3) What happens where the client discloses his criminal plan, asks for your help, but you decline and advise him against it too? It is not settled whether the communication would be privileged here, but the Restatement says it would be. (4) The crime-fraud exception will apply where you help your client commit crime/fraud, but you're not aware that it's fraudulent. Look to the client's intentions. (5) What if the lawyer, in carrying out his representation, commits a crime/fraud? The exception would not apply - we don't care if the lawyer has an evil motive. S isn't sure that this should be protected by the privilege. (6) What if the client mentions committing a crime and you don't do anything about it? Can opposing counsel seek compelled disclosure of the statement from you? Future crimes and fraud are the essence of the exception. The privilege would probably not apply here at all, since the client wasn't even seeking legal advice in connection with the crime. (7) What if client tells you that he's pissed at the judge and "intends to beat him up?" Would that be privileged? We are not trying to raise an evidentiary shield here, but can the lawyer do something, can he warn the judge under 1.6.? and would any exception allow this disclosure (death or substantial bodily harm). (a) Under 1.6b1, the lawyer could warn the judge. (b) The prosecutors are then warned that a threat has been made to him and his family. They bring the defendant's lawyer in before the grand jury to testify. Can the shield be raised? Court said it would be privileged unless the crime fraud exception applied (which didn't apply). Court said that client was probably not contacting atty about a future crime/fraud. Policy says we want client to be open with attorney, especially when it's about potentially killing a person because the atty can then either talk him out of it, or try to prevent the act. We want to keep that communication privileged. The prosecutors good (but failing) argument was that the client wasn't contacting the atty for legal advice so it should not be privileged. The lawyer both raised the shield, and told the judge about the threats.

Rule 5.2 Responsibilities Of A Subordinate Lawyer

(a) A lawyer is bound by the Rules of Professional Conduct notwithstanding that the lawyer acted at the direction of another person. (b) A subordinate lawyer does not violate the Rules of Professional Conduct if that lawyer acts in accordance with a supervisory lawyer's reasonable resolution of an arguable question of professional duty. (1) Lawyers cannot avoid responsibility for "following orders." However, where the supervisor directed the action, the subordinate lawyer may be able to prove that she did not actually know that he action was improper. (2) (b) creates a safe harbor for junior lawyers who defer to the judgment of their superiors on questions that have more than one reasonable answer. (3) Under (b), the supervisor may be disciplined where he turns out to be wrong. If the supervisor was so wrong that his belief that the action was proper was unreasonable, the subordinate may also be disciplined. (4) (b) might have the unintended effect of counseling associates to follow orders and not ask too many questions. b) Rule 5.3 explains the responsibilities of lawyers who supervise nonlawyers employees for ensuring that the employees comply with the rules of professional conduct, and explains when a lawyer may be subject to discipline based on the conduct of a nonlawyer employee.

Rule 8.3 Reporting Professional Misconduct

(a) A lawyer who knows that another lawyer has committed a violation of the Rules of Professional Conduct that raises a substantial question as to that lawyer's honesty, trustworthiness or fitness as a lawyer in other respects, shall inform the appropriate professional authority. (b) A lawyer who knows that a judge has committed a violation of applicable rules of judicial conduct that raises a substantial question as to the judge's fitness for office shall inform the appropriate authority. (c) This Rule does not require disclosure of information otherwise protected by Rule 1.6 or information gained by a lawyer or judge while participating in an approved lawyers assistance program. a) Exceptions to the reporting duty: Differences in the LA rules. (1) Insubstantial misconduct: Only those raising a "substantial question" of the lawyer's honesty, trustworthiness or fitness need reporting. LA is more broad not requiring a "substantial" question. (2) Info protected by 1.6: A report need not be made if it would reveal information required to be kept in confidence under Rule 1.6. Cmt. 2 says that a lawyer should encourage a client to waive confidentiality and permit reporting if that would not substantially prejudice the client. (3) Treatment program: A lawyer assisting a lawyer who is in an "approved lawyers assistance program" LAP is not required to report information learned in the course of this service, under cmt. 5. [LA adds the ethics advisory board beside LAP who can't tell]. b) Note that this reporting duty is mandatory in most states. c) This duty is triggered by a lawyer's "knowledge" of another lawyer's misconduct and the standard for assessing knowledge is objective (more than a mere suspicion that misconduct has occurred). The question becomes whether a reasonable lawyer in the circumstances would have a firm opinion that the conduct in question more likely than not occurred (Restatement standard). d) The reporting duty applies whether the lawyer committing the misconduct is an adversary, partner, boss, etc. A lawyer can't get off the hook by informing senior lawyers in a firm about the misconduct of another lawyer. e) The rule does NOT require that a lawyer get client approval before reporting misconduct of another lawyer. Nor does it allow a client to veto a lawyer's reporting of misconduct. f) If a lawyer learns of misconduct during an adversary proceeding, the lawyer may defer reporting the misconduct until the proceeding has concluded, if deferral is necessary to protect a client's interest. g) The reporting duty has been used as a weapon against rival lawyers - a pretense of reporting unethical behavior just to trash another partner and take over their business.

Rule 5.1 Responsibilities Of Partners, Managers, and Supervisory Lawyers

(a) A partner in a law firm, and a lawyer who individually or together with other lawyers possesses comparable managerial authority in a law firm, shall make reasonable efforts to ensure that the firm has in effect measures giving reasonable assurance that all lawyers in the firm conform to the Rules of Professional Conduct. (b) A lawyer having direct supervisory authority over another lawyer shall make reasonable efforts to ensure that the other lawyer conforms to the Rules of Professional Conduct. (c) A lawyer shall be responsible for another lawyer's violation of the Rules of Professional Conduct if: (1) the lawyer orders or, with knowledge of the specific conduct, ratifies the conduct involved; or (2) the lawyer is a partner or has comparable managerial authority in the law firm in which the other lawyer practices, or has direct supervisory authority over the other lawyer, and knows of the conduct at a time when its consequences can be avoided or mitigated but fails to take reasonable remedial action. (1) If the associate reported the solo practitioner (her supervisor), would she also have to report her own incompetence [1.1]? (2) The LASC has said that there is a duty to supervise staff (paralegals, secretaries), and to make sure that they don't engage in the unauthorized practice of law. (3) Notice that in R. 5.1, any lawyer can be held responsible for the violation of the rules by some other lawyers. This sounds a lot like agency liability. (4) In small firms of experienced lawyers, informal supervision and periodic review may suffice, while more elaborate measures may be needed in large firms. (5) Keep in mind that "law firm" is a broad concept, that includes all kinds of legal organizations. (6) A supervising lawyer will not be responsible for a subordinate lawyer's violation if the supervisor did not direct or know about it. (7) Under (c)(2), any partner in the firm who knows of the improper conduct and fails to take action to reduce or prevent the harm also commits a violation. In organizations without partners, other lawyers with comparable managerial authority are equally responsible. (8) The firm should have systems in place to make sure every lawyer is competent to do their work (9) Responsible for others in certain situations (10) Remember Castro à fudge med records guy à his supervisor got disbarred as well (11) Rule 5.3 is same thing but for non-lawyer assistants (paralegals) b) Rule 5.2 (escape clause) explains when a subordinate lawyer is responsible for her own conduct, and under what circumstances she may follow orders without fear of discipline.

In re Kelle Hinson-Lyles (La.) [could apply again but how show re-habilitation any more]

(a) Facts: Applicant had been a high school teacher, and during this career, she had engaged in sexual relations with a minor student (14 years old). The student's father found out about the relationship, called the police, and the police found the boy hiding in her closet. (b) Criminal sanctions: She was charged with carnal knowledge of a juvenile (not statutory rape). A felony. (c) In this instance, three years had passed since the incident, and she admitted everything freely. The committee let her take the bar exam, she passed, and then the court denied her admission to the bar. (i) Why did they let her take the exam before they knew whether they would let her in? Was that really a favor? (ii) If she had flunked the bar exam, then the committee wouldn't have had to look at the character and fitness stuff. (d) The applicant has the burden of showing good character. The court gives us a hint as to what good character might be: "The primary purpose of character and fitness screening is to assure the protection of the public and to safeguard the administration of justice. The term good morla character includes, but is not limited to, the qualities of honesty, fairness, candor, trustworthiness, observance of fiduciary responsibility and of the laws of the state of La. and the U.S., and a respect for the rights of other persons." Decide based on totality of circumstances just because committed felony does not in and of itself keep you from admittance to the bar. (e) The justices were split as to what her sanction should be. She did not get admitted to the bar at that time, but the court did not say when or if she might be able to be admitted. What more could she have proven. The court majority had a problem with her abusing a position of trust by knowingly and intentionally breaching the trust to gratify her own needs. (i) One of the justices said that she engaged in reprehensible conduct and should never be admitted. (Knoll) (ii) Another said she had been rehabilitated and they should let her in. (Kimball) (iii) Another said they shouldn't have let her sit for the bar in the first place. (iv) This shows how your result may differ depending on what judge you get.

a) Louisiana law In re Rojas (La.) [permanent non admittance to the bar]

(a) Facts: Bar applicant talked during the bar exam, and it was determined that she either cheated or attempted to cheat. (b) Held: Permanent denial of admission into bar. When you sit for the bar exam, you don't talk. (c) Note the commissioner's observations: The communication that she engaged in had to be cheating because there was no other reasonable explanation for the talking. The court does not question the conclusive nature of the commissioner's statement. The commissioner's report did not disclose what the applicant was talking about: it could have been to cheat, but she may have also just needed a pen. (d) This shows how serious the bar authorities treat cheating.

a) Indiscretions and misconduct in law school In re Mustafa

(a) Facts: Took money from moot court account for personal reasons. Before he got in severe trouble, he confessed what he had done, and said he intended to pay the money back. He did repay the loan. (i) The school took some disciplinary action, but only put something on his permanent record. (ii) The hearing committee recommended that he be admitted to the bar. (iii) The court denied him admission. (DC bar didn't think he upheld his burden to prove good character and fitness.) (b) He eventually became a lawyer in another state, and it turned out that he was running around with clients' money. Also failed to show up for client meetings and failed to answer/return phone calls.

(1) 2-7: Reese's Leases (based on OPM Leasing)

(a) Facts: Your client was forging documents that said that his business was making more money than it actually was, in order to get bigger loans from the bank. You put together the original documents, but did not play any role in the forgery. Your client told you what was going on when you asked, but said the fraud operation was over. What do you do when you hear about the fraud? Phony lease and opinion letter and they both go to bank and opinion letter says they own this car and that is true. Are you involved in fraud? Yes you are involved in the fraud. (b) 102(d): she hasn't violated it by "innocently" being involved in the operation. Once you know, you should not continue to write leases for them. 1.2(d) (c) Historical "noisy withdrawals" - If the lawyer dissafirms this, then he disassociates himself from the fraud. Bank would say something is wrong but huge red flag to the bank "noisy withdrawal" historical. Really a 1.6 [I am no longer lawyer and disaffirming] this was comment to the rules (rule 4.1) but were never adopted in LA so it was not clear in LA but 1.6 B2-3 open broad avenues to disclosure. § Look at 1.6(b)(2) [prevent fraud] and 1.6(b)(3) [prevent injury]: you may do these. These are new to the ethics rules. They are present in LA. § Permissive but good to do because you can be insulated from the harm. § Fake leases to create higher profits § Forging signatures § Sarbanes-Oxley § Does it reach bank loans? § Maybe not. But if it does what should he do? § Report them to SEC or not? § Who is hurt here from there scam? The bank. So you would disclose to the bank? § LA Rule 1.6 § We have the newer exceptions! § Comment after rule 4.1 (page 201) § Noisy withdrawal § Can disaffirm an opinion or document § So you could say that you disaffirm all the opinion letters you have for the bank. Now they will dig and investigate. § Some people think that this is really Rule1.6 disclosure that is just not in the rule. § This is based on real story. Lawyers did not do anything about the fraud. The lawyers had to pay 20 million dollars in civil liability when fraud collapsed.

(1) 1-2 The Doctored Resume

(a) Immigrant girl lied on her resume, even though she knew it was wrong. Another student had given her the idea. (i) Lied on her resume (ii) Advice from another student (iii) She is foreign and claims to believe this is custom (iv) Desperate (v) The law school is the one investigating (vi) Should the law school have the ability to discipline you? (a) Yes and most of them do (b) Does this violate R. 8.1? Even though this was not related to admission to the bar, it may still compromise the integrity of the profession and thereby cause her to be denied. Lying is a matter of how serious it is - social lies are customary, but lying on a resume is more egregious. (c) The integrity issue is important for bar admission, and for job searching - no one's going to hire a liar. Once you lie how do you prove good character.

Rule 8.1 Bar Admission and Disciplinary Matters An applicant for admission to the bar, or a lawyer in connection with a bar admission application or in connection with a disciplinary matter, shall not:

(a) Knowingly make a false statement of material fact; or (b) Fail to disclose a fact necessary to correct a misapprehension known by the person to have arisen in the matter, or knowingly fail to respond to a lawful demand for information from an admissions or disciplinary authority, except that this rule does not require disclosure of information otherwise protected by Rule 1.6. (1.6 is confidentiality rule). a) The rule seems to impose a duty to respond to inquiries from the bar regarding an investigation. The only exception is communications that are protected by the attorney/client privilege. The client may sue his attorney in tort if the attorney breaches his duty to him. b) It may be better to respond truthfully to questions than to try to cover them up. There is an obligation to answer the questions truthfully and let the committee make their determinations. c) Notice it reaches the person who was not yet admitted in the bar d) They can go after you for failure to satisfy the requirements in the past!

Rule 8.4 Misconduct It is professional misconduct for a lawyer to:

(a) Violate or attempt to violate the Rules of Professional Conduct, knowingly assist or induce another to do so, or do so through the acts of another; (b) Commit a criminal act that reflects adversely on the lawyer's honesty, trustworthiness or fitness as a lawyer in other respects; (c) Engage in conduct involving dishonesty, fraud, deceit or misrepresentation; (plagiarism, changing teacher evaluations) (d) Engage in conduct that is prejudicial to the administration of justice; (e) State or imply an ability to influence improperly a government agency or official or to achieve results by means that violate the Rules of Professional Conduct or other law; or (f) Knowingly assist a judge or judicial officer in conduct that is a violation of applicable rules of judicial conduct or other law. What they cover: (a) Covers all of the Rules - a blanket prohibition. (c) Covers a very broad range of misdeeds (d) Is a very vague standard (f) Is referring to the Rules of Judicial Conduct (we won't cover in this class). A. Differences with the La. Rule 1. Everything is mostly the same, except it makes it professional misconduct for a lawyer to: 2. Compare (b) and (g) from the LA rule (b) Commit a criminal act, especially one that reflects adversely on the lawyer's honesty, trustworthiness or fitness as a lawyer in other respects. (g) Threaten to present criminal or disciplinary charges solely to obtain an advantage in a civil matter. 3. (b) is different in that it is a broader rule that will punish lawyers for "smaller" crimes. 4. The ABA doesn't list (g), because (b) already prohibits criminal acts. The LASC felt that it was important to highlight criminal extortion. Note that it mentions both criminal and disciplinary charges.

A. Reporting Misconduct by Other Lawyers (Whistle Blowing or the Snitch Rule)

1. A cornerstone of the disciplinary system is the duty of lawyers to report serious misconduct by other lawyers. This section looks at the legal protection available to subordinate lawyers who refuse to commit misconduct or who complain of the misconduct of others. If you take the reporting duty seriously, its likely that you will face some retaliation. 2. In most states, lawyers are obliged to report other lawyers' misconduct to the disciplinary authorities. See Rule 8.3.

waiver

1. Big concern 2. This is a fairly big deal 3. One way it can be waived à client tells someone what you talked about and the advice you gave 4. What if you accidentally send some privileged info to the other lawyer a) Client did not waive b) Attorney actions c) Some say it is the client's privilege and therefore not waived, but other cases the attorney's conduct can waive it. 5. When there is an ongoing fraud than attorney's privilege is going to been blown 6. What if after your tell him the consequences and don't do it and so he says ima get another lawyer to help me. Is it still privileged? Probably not, maybe if it convinces him not too. 7. Client comes in for advice on legit scheme and lawyer gives wicked advice and client does not its illegal. Client's intent is what controls. Privilege still applies if the client meant well and lawyer doesn't and never tells client 8. What about if client comes in with dirty scheme that attorney thinks is ok. This time the privilege falls. 9. Attorney representing father and juvenile. Dad is upset about judge's decision and leaves 6 messages on attorney's answering machine. They are violent and irrational. Says he knows where judge lives and how many children he has and prejudices should be exterminated. Attorney erased them. Can he tell the judge? If you think there is a reasonable prospect that this will be carried out than you can warn the judge. Prosecutor comes after lawyer and says tell us what he said because it's a crime. Lawyer claims attorney client privilege. This was not for getting legal services according to prosecutors. The court found that it was! The prosecutor didn't carry the burden. Worth preserving here for policy reasons.

A Concluding Problem: 3-9: Rat Poison

1. Client operates extermination business, and accidentally placed industrial strength rat poison in a home. A little kid ate it and died. Client has been charged with involuntary manslaughter. The client admits that he also put the poison in many other homes. You instruct client to go clean up these houses so nothing else happens, but months pass and the client keeps promising he'll get to it. 2. Exception to nondisclosure under R. 1.6b1: To prevent reasonably certain death or substantial bodily harm. Is the harm in this instance reasonably certain? Are you assisting the client if you don't disclose? 3. Would R. 4.1(b) require disclosure here, to avoid a crime (state statute makes it a misdemeanor to recklessly engage in conduct that creates a substantial risk of death)? 4. Could you withdraw to solve the problem? The duty of confidentiality continues after the attorney client relationship, however, an exception under the mitigation 1.6(3) could apply. If the attorney's services have not been involved in the fraud, then it might not fall under 1.6 at all. Hypo: Your rep a client sue for fraud, but he hates one of the investors who sued him, he plans to kill him... can you call the intended victim and inform him? You can disclose to the victim under 1.6b1.

A. Waiver

1. Hypo: You client confesses that she stole money from her ER and then lost it all at a casino. You give her advice for when she is contacted by the police. This would probably be protected by the privilege, since the wrongdoing has already occurred. What if she gets so excited about the conversation she had with you that she tells her friend all about it? Has she waived the privilege? Perhaps - the privilege will be waived if the client (or the lawyer) reveals the privileged information to a nonprivileged person. a) Facts are not protected, but the communication is. So if opposing counsel asked my client if he ripped off a bunch of senior citizens, then that is not a privileged question, they are facts.

A. Client Identity

1. In general, the identity of a client is NOT privileged, but the law is not entirely settled. 2. In a few cases, client identity is held to be privileged only where it represents the last link in a chain that shows that the client had committed a criminal offense. 3. Then prosecutors have sought to learn the clients' identities and the lawyers invoke the privilege, judges have sometimes required disclosure while other times the privilege has been upheld. 4. The Restatement takes the view that identity should be privileged when revelation of identity would reveal the content of lawyer-client communications. 5. Example: Hit and Run: Do you have to tell the name the prosecutor wants to know? Her name would be confidential (duty not to tell) under 1.6 because it deals with representation. Am I ok under attorney-client privilege? Can you assert the privilege dealing with the identity. Generally it's not privilege but if it will incriminate them its called last link. Last link for the persecutor, meaning he has all, but the name.

The Duty to Protect Client Confidences A. Introduction: Attorney-Client Privilege v. Duty of Confidentiality

1. Judges will use the word "privilege" to describe "confidentiality" and vice versa. THESE THINGS ARE DIFFERENT!!! 2. Confidentiality comes from ethics. The attorney-client privilege comes from evidence law - it is not a duty, but a shield to protect confidential information from compelled disclosures (where there is some legal compulsion or power of the court behind the command). Confidentiality is not a shield, but a duty to keep quiet. Privilege is a shield, to protect from questions. Confidentiality is an element of both, though. 3. There are certain elements required to even evoke the attorney-client privilege; however, the scope of the duty of confidentiality is unclear. There are exceptions to both, and they are not the same. B. The Basic Principle of Confidentiality 1. Protection of "information relating to the representation of a "client" a) Rule 1.6

I. The Attorney Client Privilege and the Work Product Doctrine A. Confidentiality and attorney-client privilege, compared

1. Need communication, in confidence, between privileged persons, for legal services/advice. 2. The privilege falls within the broad scope of the duty of confidentiality. It is possible that some subject matters of the privilege may fall outside the scope of the 1.6 duty, like when lawyer-client communications fall within an exception to 1.6. 3. Difference in source: Recall that the duty to protect confidences is imposed by ethical rules, violation of which can result in discipline, while privilege is a matter of evidence law, which governs what kinds of evidence can be admitted in court and can be used as a shield to prevent evidence from coming in. 4. Under the privilege, generally, neither lawyer nor client may be compelled to testify in court about protected communications, and neither can be penalized for noncompliance with an order to reveal information. In instances where a court order might override the duty of confidentiality (1.6(b)(6)), the attorney-client privilege can be invoked to crush the court order. If privileged information is sought, the lawyer claiming the privilege might quash a subpoena or object to compliance with a discovery request. 5. Scope a) The confidentiality duty is very broad, covering all information related to the representation. b) The privilege however covers only a relatively small part of information - the part involving communications between lawyer and client in which the client is seeking legal advice or other legal services. The privilege can only be claimed as to the parts of a communication that are privileged. a) Recall that the duty of confidentiality is a duty to keep your lip buttoned, but the privilege is a shield to resist disclosure where it applies. b) "Common law evidence rule" is not technically correct for La. La.'s Code of Evidence provides a sophisticated provision on the privilege - it is not a common law rule here.

The Crime-Fraud Exception

1. No privilege if a client seeks assistance with a crime or fraud a) Even if the lawyer-client communication satisfies all the elements for privilege, no privilege attaches if the client consults a lawyer for assistance (in furtherance) in committing a crime or fraud. Also, there is no privilege for a conversation if the client later uses the lawyer's advice or services to commit a crime or fraud. b) Note that the client's intention in fact to perform a criminal or fraudulent act triggers the crime-fraud exception. It will not matter whether the client knows the act is criminal or fraudulent. (1) What if a client asks a lawyer for advice, learns that the planned conduct is criminal, and doesn't commit the crime? The conversation should be privileged. One purpose of the privilege is to enable clients to get sound advice from lawyers and avoid committing criminal acts - if the privilege were to apply here, that would further this policy. (2) What if a client consults a lawyer about a plan he knows involves a crime or fraud, but conceals facts from the lawyer that would reveal the illegality of the scheme? The lawyer's knowledge or intentions are irrelevant. Only the client's intentions are relevant - no privilege in this case. HOWEVER, if it is the lawyer with the criminal intent and the client is pure, then the conversation will be privileged. WEIRD. c) What if a client asks for advice about a crime he plans to commit, but does not ask the lawyer to give advice that assists him in the act? There is a distinction drawn in this case. A request for advice about whether a certain act is permitted under the law will be privileged in most states. d) In addition, a communication where a client asks his lawyer for advice about a past act that was criminal or fraudulent will be privileged, as long as the past act is really in the past. No privilege will attach if the bad act is continuing. e) The crime/fraud (exception) applies to ongoing crime or fraud. Assistance only goes so far we can assist them to the murder case these are privileged even though criminal. Client comes in and admits to civil fraud so it's not even criminal. All privileged - past. But when on-going crime or fraud will not be covered by attorney client privilege. What if you talk him out of it? Tell him not to do it but leaves and gets another lawyer to engage in fraud risk the information is blown because on-going deal. IF you talk him out of it - he does not do it privileged. Come to us for legal advice for investment scheme and give advice to set it up and phony but we (lawyer) don't know - not privileged. Clients intent controls so if he wants honest scheme but lawyer gives dirty advice and client does not know the privilege applies. (1) He wants help with employment but also going to kill employee. Can't disclose about employment but tell him killing is bad idea (could tell about killing b/c 1.6 exception) but does the privilege apply? One question could be does it fit definition "legal assistance" he just tells you he is angry is that communication for the purpose of legal assistance the privilege won't apply. (2) Judge example case - Think 2 part process 1.) tell under 1.6 but 2.) now go after attorney tell us what he said but judge said it was covered by attorney/client privilege. We think (so does smith) b/c it was not in legal services not privileged OR use the fraud/crime exception. Court said attorney/client privilege protected so said it was covered.

A. Use or Disclosure of Confidential Information for Personal Gain or to Benefit Another Client

1. See R. 1.8(b): A lawyer shall not use information relating to representation of a client to the disadvantage of the client unless the client gives informed consent, except as permitted or required by these Rules. a) This rule applies when the information is used to benefit with the lawyer or a third person. b) The rule does NOT prohibit use of information that does not disadvantage the client. Even where use might disadvantage the client, the information may still be used if the client gives informed consent. 2. Restatement: Prohibits the use of confidences "if there is a reasonable prospect that the use will adversely affect a material interest of a client or if the client has instructed the lawyer not to use or disclose such information."

A lawyer is an agent for his client, who is the principal.

1. There is a concept that says that if you use information that you received from your principal that causes an economic benefit to the agent, then the proceeds must go back to the principal [agency/principle rule but professions rule]. Does it hold true in this instance? 2. Disclose 3. OR USE - engages to client's disadvantage. You know advantage (A) and you know one of your other client's are looking for real estate (B) so you tell (B) hey I know of a good deal and buy it b/c prices are going to go up b/c (A) is developing. You are not using for your benefit OR disadvantage but this would be disclosure not use. Can't do it b/c revealing 1.6 information. Make sure know difference between the two [disclosure or use].

A. The Law Governing Lawyers State Ethics Codes

1. Through the state's adoption (and modification of) the Model Rules

A. The Work Product Doctrine

1. Work product prepared in anticipation of litigation a) This is related to, but separate from, the attorney-client privilege. This doctrine protects notes and other material that a lawyer prepares "in anticipation of litigation" from discovery in pretrial civil proceedings. b) This applies to documents that a lawyer prepares or collects while working on pending litigation or on a matter in which the lawyer knows that a lawsuit is about to be filed. c) This may protect some documents that are not covered by the attorney-client privilege because they do not relate to communications between a lawyer and his client. d) Protection of work-product is not absolute, but will probably apply in cases where the document reflects the lawyer's strategic thinking. e) Hypo: You're working on an auto accident case where there are witnesses that saw the accident. You go talk with them and take down notes about their statements. (1) Do the statements fall under the attorney-client privilege? No, because the communication was not "between privileged persons." (2) The work-product doctrine might protect the notes - it protects all material that the lawyer prepares in anticipation of litigation. (3) What is the point of this doctrine? (a) To protect the lawyer's mental impressions. (b) To prevent opposing counsel from freeloading. So if we got a hypo like that on exam, analyze that it may fall under atty-client privilege and work-product doctrine You can defeat discovery with the privilege doctrine Evidentiary privilege Shield

Mitigation factors

1.) Physical disability 2.) Free disclosure to board; you are cooperative 3.) Absence of past discipline 4.) Absence of dishonest or selfish motive 5.) Emotional problems 6.) Good faith efforts to engage in restitution to fix problems 7.) Small amount of practice time (exception for the lying weasel)

I. Introduction

A. What are the reasons for these rules? 1. Keeps lawyers out of trouble 2. Helps lawyers to evaluate the behavior of other lawyers so they can take protective measures B. Distinction between morals and ethics 1. Members of a "profession" typically must adhere to a code of ethics. Traditional professions include doctors, lawyers and ministers. 2. Morals concern one's personal philosophy of right and wrong. 3. It's not unusual that a rule of ethics might be in conflict with a rule of morals. C. The ABA model rules are not the law. They must first be adopted by a state's supreme court, although states usually don't alter the model rules. Be sure to know any differences, if any, between the state and model rules. In LA we have adopted the Model Rules but not the comments.

Mal Practice in General: Negligence

Duty: Duty to the client: (attorney client privilege). In order to get malpractice need the attorney/client relationship and need duty of care associated with it. Lawyer failed to exercise "the competence and diligence normally exercised by lawyers in similar circumstances." We don't care how long you have been in practice Location we normally don't take into account wither but in LA we look at the standard of care for a lawyer in the "locality" involved. - What if pro-bono: standard of care is the same. - What if a wealthy classmate client wants to pay attorney a lot per hour: the cleanest way to do it is to refer the client to another attorney (avoiding malpractice issue). [or get help from co-counsel or learn it] Breach: With breach if criminal need to prove innocent; civil is "case within a case" need to prove you would have won but for the attorney. Legal mal-practice Lawyer owes a duty to ∏ That the lawyer failed to exercise "the competence and diligence normally exercised by lawyers in similar circumstances" That the breach of duty caused harm to the ∏ Causation Damages

Rule 4.1 Truthfulness In Statements To Others

In the course of representing a client a lawyer shall not knowingly: (a) make a false statement of material fact or law to a third person; or (b) fail to disclose a material fact to a third person when disclosure is necessary to avoid assisting a criminal or fraudulent act by a client, unless disclosure is prohibited by Rule 1.6. ...this one isn't likely to come up in a whole lot of cases, but it does come up. - You know the how deal is a scam [Reese's leases] do you now have a duty? Yes, but 1.6 cuts off the duty BUT under 1.6 its permissible (B)(2-3) so its not prohibited. - You CAN under 1.6(B)(2) - Also have a DUTY under 4.1 (i) Note that whereas 1.6(b)2&3 revelation is permissive, revelation under 4.1 is MANDATORY, unless prohibited by 1.6. (ii) This means that in any situation in which a lawyer's failure to reveal would constitute "assisting a criminal or fraudulent act," 4.1 requires a lawyer to reveal the information. This is clearly a little inconsistent with 1.6 - the lawyer's discretion to reveal information in 1.6 is totally restricted by 4.1. We're not sure why this is the case. It probably has to do with the timing of amendments and the failure to make all the rules consistent. (iii) Cmt. 3 to R. 4.1or the Noisy Withdrawal: This does not require that you have to disclose all the facts of the fraud, but you have the option of disaffirming the documents and withdrawing from representation. The injured third party may inquire further about the fraudulent conduct, but you don't have to do anything else. (a) Why isn't this in the rule? There was a big fight when the noisy withdrawal provision was included in the rule. Putting it in a comment was a compromise. (b) There has been revision since the rule was promulgated, and yet this still remains in the comment. We're not sure where the LASC stands on this. (1) The bank is considering the loan and now you know it's a scam (a) Just looking at Rule 4.1 do you have a DUTY to blow the whistle to the bank?????? (b) Material fact à phony leases that loan based on (c) You could tell the bank about that. Yes because Rule 1.6(b)2 or (b)3 (d) So permission and a duty. (2) Hypo à Client comes to you help for defense of murder. Admits to the murder. Also wants to defraud old people and tells you about the scheme. Then goes back to talking about the murder. Can you disclose the scheme for fraud? (a) Under Rule 4.1 do you need to disclose to not assist in the fraud? (b) Does it have anything to do with the representation? If so that under Rule 1.6. If not then there is no duty not to disclose. (3) Hypo àSuppose representing a client sued for fraud. Sick and tired of person involved in the fraud and he wants to murder him right away. Can you call and warn him? (a) If you really think he means it than an exception to Rule 1.6 and therefore you can warn the guy

Reporting another lawyer's misconduct to disciplinary authorities is an important duty of every lawyer.`

Lawyers are in the best position to observe professional misconduct and to assist the profession in sanctioning it. While a Louisiana lawyer is subject to discipline for not reporting misconduct it is our hope is that lawyers will comply with their reporting obligation primarily because they are ethical people who want to serve their clients and the public well. Moreover, the lawyer's duty to report professional misconduct is the foundation for the claim that we can be trusted to regulate ourselves as a profession. If we fail our duty, we forfeit that trust and have no right to enjoy the privilege of self-regulation or the confidence and respect of the public.

Malpractice Insurance

Most state (not LA) lawyers are required to tell clients if they have malpractice insurance or not it's the idea of let the market decide if they want lawyers with mal practice insurance so it does not become another regulation for lawyers. Actions for legal malpractice... p.20 in supp. Our LA statute is very lawyer friendly 9:5605. We have a prescriptive component and a peremptive component. You are barred within 3 years total from the alleged mal practice but you have prescription of 1 year from time of malpractice OR if didn't know when you knew or should have known. Don't count the day the incident happened. Also, exception for fraud. · La. Rev. Stat. 9:5605. Actions for legal malpractice o Jan 5 2008 § What happened on this day what a real estate closing and there was a defect in the act of sale and now it's invalid. o Sept 20 2008 § Client learns of this o When is the latest the client can sue? § (A) One year from the date of the act or from discovery § Sept 21 2009 o What if the date changes to Sep 10 2010 when client finds out § The preemptive period cuts it off at Jan 6 2011 (as opposed to Sep 11 2011) o There is a 3 year preemptive period and 1 year prescriptive period o Exception for fraud

Breach of Contract Breach of fiduciary

Note that a criminal ∆ has to prove their innocence to sue for mal-practice Civil side à case within a case idea. You have to prove you would have won but for the mal-practice. The duty has to come from the attorney- client relationship. What about 3rd parties? Normally the answer is no because we no privity. But there are some circumstances. Think about if lawyer messes up will, then the heirs could sue. 66% of claims brought against lawyers working for 10 years 80% of claims brought against small firms. But really big payouts are for the bug firms typically. Oregon only place which requires mal-practice insurance. Lot of others states make lawyer reveal if has or not and let marketplace decide. LA does not do this. A lawyer can be liable to a 3rd party who is not his client: Among the 3rd parties to whom a lawyer owes duties are prospective clients and people who are intended beneficiaries of the lawyers work for a client, such as those who will inherit assets under a will drafted by the lawyer. Opinion letter ensuring finance security of corporation and then go under.

Discussion of Ethics vs. Mal practice

The scope of the model rules states "Violation of a Rule should not itself give rise to a cause of action against a lawyer nor should it create any presumption in such a case that a legal duty has been breached. In addition, violation of a Rule does not necessarily warrant any other non-disciplinary remedy, such as disqualification of a lawyer in pending litigation. The Rules are designed to provide guidance to lawyers and to provide a structure for regulating conduct through disciplinary agencies. They are not designed to be a basis for civil liability. Furthermore, the purpose of the Rules can be subverted when they are invoked by opposing parties as procedural weapon. The fact that a Rule is a just basis of a lawyer's self-assessment, or for sanctioning a lawyers under the administration for a disciplinary authority, does not imply that an antagonist in a collateral proceeding or transaction has standing to seek enforcement for the Rule. Nonetheless, since the Rules do established standards of conduct by lawyers, a lawyer's violation of a rule may be evidence of a breach of the applicable standard of conduct." Last sentence takes away that separation of ethics and mal practice. - Many people would expect the ethics requirements to be higher in standard than mal practice if not it makes for a not friendly working environment fro lawyers always after one another. Nevertheless it seems ethics rules can be sued as a breach for mal practice. - LA took out the aspirational reach of any ethical rules. History of the DC (rules which help punishment) and the EC (ethical rules for feel good but could not be punished well people were being punished for violating the EC's).

the law governing lawyers contract law

This usually comes up in cases where courts nullify K's that lawyers were parties to or contracts negotiated by lawyers.

A. Exceptions to the Duty to Protect Confidences 1. Client frauds and crimes that cause financial harm

a) Lawyers are not permitted to assist clients in committing criminal or fraudulent acts, but sometimes lawyers help clients file papers that include false information without realizing that the information is false. What are the lawyer's obligations to his client and/or the harmed party? b) Enron and the Sarbanes-Oxley Act (1) After Enron, which involved massive corporate fraud committed by accountants and lawyers, Congress passed SOX to prevent further episodes like this. (2) SEC rule: Lawyers who practice before the Commission or who advice companies regulated by the Comm. are required to report any information about securities fraud to the highest official to the corporation. It requires the lawyer to take action if within scope he must go up corporate chain to get is resolved. This reach is narrow. Blow whistle up corporate later. Does not require lawyer to go outside. (3) The SEC only refrained from imposing more stringent reporting obligations on lawyers because the ABA amended its model rules to permit lawyers to blow the whistle on their clients' frauds. (4) Under rule 1.13, there is a circumstance that a lawyer could be required to go outside the corporation to report a violation...more later... c) The ethical rules on revelation of client crimes and frauds (1) Before Enron, the model rules gave lawyers no discretion to reveal confidential information to prevent or mitigate the harm from client fraud. (2) The initial proposal was to make disclosure of client crime and fraud mandatory, but this was withdrawn. In substitution, lawyers were allowed discretion to decide whether to reveal those crimes and frauds. These exceptions are now contained in (b)(2) and (3). B2 and B3 are the only provisions under 1.6 that have to deal with the lawyer's "services" (3) These exceptions look very similar but there are subtle distinctions: (a) (b)(2): Applies to situations where the client plans to commit or is committing the crime or fraud. (b) (b)(3): Applies to the situations of past crime or fraud. (c) (b)(4): it's OK to get advice (best phrased as a hypothetical) (d) (b)(5): can disclose information that would harm us but only when necessary- question: do you have to wait for a ruling to come down? No. could be a preliminary thing because of the "allegation" language (4) Under these two rules, a lawyer may reveal client criminal or fraudulent conduct whether it is past, ongoing or future if: (a) There is reasonable certainty that the client's conduct will result in financial injury or injury to the property of another person; (b) The client is using or has used the lawyer's services in committing the act(s); and, (c) The purpose of revealing the confidences is to prevent the criminal of fraudulent act OR to prevent, mitigate, or rectify the harm resulting from the act(s). (5) Physical versus financial harm (a) Physical: The rules focus on whether the harm was in the past or the future, and on the severity of the harm caused. (b) Financial: It's much less clear whether the act that caused the harm was in the past or the future. A lie told last year about the financial status of the company may be relied upon next month by a stock purchaser. Therefore the rules allow revelation of confidences about past, present OR future client fraud that has caused financial harm. (6) The lawyer can ONLY reveal such information if the lawyer's services were used in perpetration of the criminal or fraudulent act. Note that where the lawyer's services were used to commit the act, disclosure in these instances is permissive. (7) Aside from 1.6(b)(2)&(3), there are many other provisions dealing with fraud in the model rules: (a) See R. 1.2(d) (explained further below): A lawyer shall not counsel a client to engage, or assist a client, in conduct that the lawyer knows is criminal or fraudulent, but a lawyer may discuss the legal consequences of any proposed course of conduct with a client and may counsel or assist a client to make a good faith effort to determine the validity, scope, meaning or application of the law. (b) So now we know they are we assisting in a criminal or fraudulent act. Put together 1.6 and 4.1 so now we can take action when we find out we have become involved. Client comes to you for criminal defense case - he said he killed her but he wants your defense but he also has fraudulent scheme to rip off senior citizens but go back to murder case. (i) Can you disclose plans to engage in fraud? Can't use 1.6 because not using 1.6. 4.1 seems to urge duty to disclose [could be a stretch] (c) Coming you to fraud but says he is going to kill someone? Well that is exception to 1.6 to prevent reasonably certain death. This is clear. (d) In either case is it related to representation? No, neither are related to the representation. If it is its covered by 1.6 but what if its not then no duty to not reveal. If it's not 1.6 don't have duty of confidence. Kill someone in lawsuit he asks to defend. This is hard if unrelated. (e) See R. 4.1...

institutions that regulate lawyers administrative agencies

a) Many administrative agencies impose additional ethical or procedural rules on lawyers who appear before them. b) Lawyers who engage in misconduct in practice before these agencies may be subject to civil or criminal penalties.

Rule 5.3 Responsibilities Regarding Nonlawyer Assistants

With respect to a nonlawyer employed or retained by or associated with a lawyer: (a) a partner, and a lawyer who individually or together with other lawyers possesses comparable managerial authority in a law firm shall make reasonable efforts to ensure that the firm has in effect measures giving reasonable assurance that the person's conduct is compatible with the professional obligations of the lawyer; (b) a lawyer having direct supervisory authority over the nonlawyer shall make reasonable efforts to ensure that the person's conduct is compatible with the professional obligations of the lawyer; and (c) a lawyer shall be responsible for conduct of such a person that would be a violation of the Rules of Professional Conduct if engaged in by a lawyer if: (1) the lawyer orders or, with the knowledge of the specific conduct, ratifies the conduct involved; or (2) the lawyer is a partner or has comparable managerial authority in the law firm in which the person is employed, or has direct supervisory authority over the person, and knows of the conduct at a time when its consequences can be avoided or mitigated but fails to take reasonable remedial action.

work product doctrine a qualified protection

a) A judge can still order disclosure of documents falling under this protective rule if the opposing party can show: (1) "Substantial need" for the material, and (2) That it is unable "without undue hardship" to obtain the substantial equivalent of the material by other means. b) This is a vague standard, and usually lawyers don't know when their documents will be protected by the doctrine and when they won't. c) Note that an exception like this does not exist for the attorney-client privilege. d) This showing of substantial need can trump the work product doctrine only for Ordinary work product - compiled by the lawyer but does not contain the lawyer's "mental impressions." (1) Substantial hardship will get you the text of the accident transcript but you are not going to get the mental impressions. (2) Not sure what extraordinary need would never get to the metal impressions. You are paying for litigation in auto accident case. You interview witness and take notes and keep in file. Witness dies before trial occurs. Are the notes work product? If it is there is a substantial hardship for adversary than there is access. Probably can show this is there is a dead person.

A. Exceptions to the Duty to Protect Confidences 1. In general

a) A lawyer cannot reveal confidences unless: (1) The client gives informed consent; (2) The disclosure is impliedly authorized in order to carry out the representation; OR (3) It is permitted by paragraph (b) b) 1.6(b) provides a list of permissive excepted circumstances in which the lawyer may reveal client confidences, to the extent the lawyer reasonably believes necessary (narrow the content as much as you can and only tell as many people as you need to). However, before making a disclosure under (b), you should try to persuade your client to take action that will obviate the need for disclosure. c) List of exceptions: (1) To prevent reasonably certain death or substantial bodily harm. (2) To prevent the client from committing a crime or fraud that is reasonably certain to result in substantial injury to financial interests or property of another and in furtherance of which, the client has used or is using the lawyer's services. (3) To prevent, mitigate, or rectify substantial injury to the financial interests or property of another that is reasonably certain to result or has resulted from the client's commission of a crime or fraud in furtherance of which the client has used the lawyer's services. (a) This addresses the situation where the lawyer doesn't learn about the crime/fraud until after its been committed. (b) This does not apply when a person who has committed a crime or fraud thereafter employs a lawyer for representation concerning the offense. (4) To secure legal advice about the lawyer's compliance with the ethical rules. (5) To establish a claim or defense on behalf of the lawyer in a controversy between the lawyer and the client, to establish a defense to a criminal charge or civil claim against the lawyer based on conduct that lawyer was involved in, or to respond to allegations in any proceeding concerning the lawyer's representation of the client. (6) To comply with other law or a court order.

Revealing confidences to obtain advice about legal ethics

a) A lawyer may invoke this exception to consult another lawyer for advice. The other lawyer may be in another firm, a law professor, or a bar official. b) Compliance with the rules is more important than protecting client confidences. 2. Using a client's confidential information to protect the lawyer's interests a) R. 1.6 provides yet another exception:

1. Waiver as to a conversation by disclosure of part of it

a) A partial disclosure of privileged information may be a waiver of the privilege as to the part of the conversation that relates to the subject matter on which the client volunteered testimony.

(5) to establish a claim or defense on behalf of the lawyer in a controversy between the lawyer and the client, to establish a defense to a criminal charge or civil claim against the lawyer based upon conduct in which the client was involved, or to respond to allegations in any proceeding concerning the lawyer's representation of the client;

a) Cmt.: A lawyer can disclose information in response to an allegation/assertion that he's engaged in fraud. In this instance, the lawyer will not have to wait until suit is filed to make the disclosure. b) There are two main instances where this would be used: (1) To defend against allegations of misconduct (malpractice, disciplinary or criminal proceedings). He has to defend himself. (2) To collect a fee. You can say I will sue you and in that suit I will lay out what I did for you. OR Do you remember you told me about your affairs - do you want me to reveal these...pay me. (a) What I did - yes as to say why I should be paid (b) Affair - no c) No more than necessary may be revealed. In these instances, the lawyer must still take steps to narrow the amount revealed - the disclosure should be no greater than the lawyer reasonably believes necessary to accomplish the purpose. d) The lawyer should notify the client before using confidential information in self-defense and should seek solutions that do not require the disclosure. The lawyer may, however use the information even where the client does not consent. e) The lawyer's right to respond (and thus disclose confidences) arises when an assertion of such complicity in wrongdoing has been made, rather than when the complaint is filed. The lawyer does not need to await the commencement of an action or proceeding before this right is triggered. f) A lawyer may reveal confidences even if the allegation is made by an injured third party rather than by a client, and even if the lawyer is not the primary target of the allegation.

1. Murder for Hire (4-1): another prisoner overhears a prisoner's confession about something. Does privilege apply in a case where a prisoner was shouting and another prisoner overheard? So it was probably not "in confidence." Lawyer should ask for a private room, and tell the people listening that they cannot listen. What if there was no private room? Just find the most private spot. So when an attorney is speaking with the client, the attorney should make sure no one else is in the room or within earshot. If there is a reasonable expectation of privacy, then that should be OK.

a) Communication - yes b) Confidential - ? Practical consequences of this situation. If attorney/client privilege does not apply then the prosecutor can say "When you and lawyer were talking what did you tell her?" Respond with attorney client privilege but if it was not confidence then huge problem. We want to avoid discovery. c) Between privileged persons - yes d) Legal services/advice - yes

1. Communication

a) Communication" basically encompasses any mode of exchanging information between attorney and client. b) The privilege only protects against disclosure of the information itself, not against disclosure of the underlying facts that might have been communicated. c) Note that communications with a prospective client or lawyer are also privileged. d) Last link doctrine e) Privilege protects the communication not the underlying facts. (1) Did he kill wife? That is fact but the answer is protected if he told lawyer and also 5th amendment. (2) What did you talk about? Objection. (3) Objection noted but still answer? No don't need to answer and tell client not to answer. (4) Now you tell him again no to answer but he wants to answer. Tell them its over

1. Privileged persons

a) Communications will only be protected if between lawyers and clients. However, this includes communications with agents of a lawyer. b) While a lawyer should not casually allow a third person to be present during a confidential communication (might be a waiver of the privilege), what other persons might be covered by the privilege anyway? (1) Interpreters (2) Persons to enable or facilitate communication or to provide psychological support during a lawyer-client interview (like a client's psychologist) (3) Parents of a minor child client (4) Guardians of incompetent persons

1. In re Michael G. Riehlmann (La.): Leading case on R. 8.3 issues

a) Facts: Fellow attorney friend (and former assistant DA) who was dying of cancer revealed to Riehlmann that he concealed evidence that could have exculpated a former defendant. At the time R was having personal and family issues. R disclosed the information 5 years later when he discovered the former defendant was on death row. If R had not come forward with the information, the guy would have been put to death. b) This is serious enough that it would have to be reported under 8.3. c) Because he didn't withhold the information maliciously, R was just publicly reprimanded. Was this too light or too heavy? Does this penalty make it seem like the LASC doesn't take 8.3 too seriously? d) Brady Material: information that could shed light on the defendant's penalty or prosecution must be brought forward by the prosecution. They have to come forward with exculpatory evidence. e) What is "knowledge" under R. 8.3? (1) "A lawyer will be found to have knowledge of reportable misconduct, and thus reporting is required, where the supporting evidence is such that a reasonable lawyer under the circumstances would form a firm belief that the conduct in question had more likely than not occurred." (2) Thus, absolute certainty is not required, but there must be more than a mere suspicion. (3) A "reasonable time under the circumstances" is interpreted as "promptly." 5 years was not "prompt" under the circumstances. [he waited too long] (4) The obligation to report is satisfied when, in La., you have reported to the Office of Disciplinary Counsel. If you report to anyone else, it won't count! [didn't report to correct person] 1. Mitigating Circumstances? Lawyer says the reason he didn't report it was due to his mental instability (family member's heart surgery, depression, marriage collapsed, etc.) Even if these were mitigating, seems unlikely these mental conditions would last for 5 years.

1. In re Himmel (IL 1988)

a) Facts: Himmel learned of lawyer misconduct through a client. Himmel sues the lawyer that withheld the client's money. They enter a settlement agreement - will not report if a certain amount is paid. b) Held: Himmel suspended for not reporting the misconduct of another lawyer. c) Himmel's first defense: He didn't report the other lawyer because his client told him not to. (1) It's good leverage to say that if you don't pay your client, then you'll be reported. R. 8.4(g) in La. prohibits doing this: "It is professional misconduct for a lawyer to: (g) Threaten to present criminal or disciplinary charges solely to obtain an advantage in a civil matter." (2) The court rejected this defense. Himmel should have known better than to listen to his client. d) Himmel's second defense (also rejected): Attorney - client privilege. However, the privilege was abrogated as Himmel discussed the misconduct with too many other people, including the client's own mother. When the offending lawyer signed a settlement K admitting his wrongdoing, this also broke any existing attorney-client privilege. e) Most state courts have indicated reluctance to follow the Himmel lead, although NY has adopted a "law firm rule," which is an attempt to hold an entire law firm responsible for lawyer misconduct, even when an individual culprit cannot be identified.

1. In re Peters (Minn. 1988)

a) Facts: Law school dean sexually harassed student workers and secretaries in his office. He didn't rape them, just inappropriately touched their ribs, hair and hips. The victims responded by trying to avoid the situations. b) What part of 8.4 did he violate? It is hard to find a good match, and we must use a stretched argument to make it fit. Part (a) could be violated if we can find another rule that could be engaged by this behavior. c) The sanction in this case was a public reprimand, which seems a little light. This was probably due to the fact that he was the dean rather than a fellow student. Fear of the students getting expelled because this man was the dean. d) Questionable conduct by law professors: [even if not practicing] (1) Sexual harassment (2) Plagiarism (of the writing of their research assistants) (3) Neglect of teaching responsibilities (not performing in class while collecting fees for consulting on the side) (4) Manipulation of grades or teacher evaluations (punish/reward certain students) (a) One teacher engaged in this behavior and remember the ODC can prosecute you for a violation even if the school does nothing. In ODC world this would fall under 8.4(c). Engaging in dishonest, fraud, deceit and misrepresentation. (5) Aggressive or discriminatory behavior

institutions that regulate lawyers law firms and other employers

a) Many employers have their own additional rules of practice. Larger firms usually set up an ethical infrastructure to provide training, offer advice and prevent conflicts of interest. b) Many firms will designate lawyers to be ethics counsel or loss prevention counsel. Others will form ethics committees. c) This is also a type of "private" law to govern lawyers' conduct, and will often times be more strict than the state rules.

1. 3-2 Worldwide Bribery

a) First, as a matter of public policy, should we allow communications regarding bribery to be privileged? Yes, because if we didn't protect the communication, this would discourage corporations from voluntarily looking at their own behavior. The attorney-client privilege always comes at a price, since it restricts the free flow of information. b) In this instance, the officers of Horizon Corporation were engaged in rampant bribery, and as general counsel, you have interviewed employees of the corporation to verify the story. You and the president of the corporation agree to contact the US attorney, admit to the bribery in general terms and offer to pay a civil fine. The US attorney responded that federal prosecutors are willing to cut a deal with you after you turn over evidence of the bribery. It seems reasonable that prosecutors would want to know all the facts before making a deal. The US attorney threatens that if you don't cooperate, then they will come after the corporation themselves, since they now know that the corporation broke the law. c) The bad part? The employees that disclosed information to attorneys are being hung out to dry, since their information is being handed over to prosecutors. The corporation as a whole is helped by the deal, but at the expense of the employees that shared information. d) The people that get shafted here were those that the SC tried to help in Upjohn with the subject matter test. e) The corporation might just want to tell the government to shove off. The government usually won't be able to get their act together to come after the corporation within the statute of limitations, so they might back down. (1) McNulty policy - US attorney office offers cooperation to reduce sentences. Corporation might be disinclined to hand over information it waives the privilege. Giving us the information you will waive the privilege but prosecutes will be nice to you. WAIVE. Story - firm didn't give the information and the government was not quick enough and so didn't charge the firm. Need to make your own decision. (2) With Upjohn is it the employees privilege or the corporation's privilege. If it's the corporations privilege they get to waive the attorney/client privilege. It is not the employee's privilege to assert so if the corporation waives the employees are exposed and have no privilege to assert.

1. The Exception for R. 1.6: 1.6 usually Trumps all other rules, it has a crushing effect.

a) How broad is this exception? R. 1.6 requires lawyers to keep confidential all matters relating to the representation of a client. b) Some think that R. 1.6 swallows the reporting rule. Most of the information lawyers receive comes in connection with the representation of a client! c) This was one of the defenses Himmel tried to raise. It failed there, but that doesn't mean that it might not be successful somewhere else.

1. Compliance with court orders

a) If a judge decides that the communication is not privileged and orders it to be turned over, this will not waive the privilege. b) Turning over information in response to a court order does not waive the issue for purposes of appeal or of other litigation.

A. Institutions that Regulate Lawyers 1. Lawyer disciplinary agencies

a) In La., this is called the Office of the Disciplinary Counsel. They investigate and prosecute misconduct that violates the state ethics code. b) Usually run by the highest state court, the state bar association or by both. c) The body of published opinions in disciplinary cases is helpful to interpreting and applying the ethics codes.

character and fitness inquiry

a) In general (1) The basic point of this is to assess whether the applicant will practice law in an honest and competent manner. An application that raises significant problems of moral character may lead to an investigation by the bar and a formal hearing on the applicant's qualifications for admission. In most states there is no published list of what conduct gives rise to an inquiry and no consistency in practice. (2) Bar admissions committees, courts and the Model Rules say to be scrupulously honest in everything you say, even if your disclosures could delay or prevent your admission to the bar. Lying or concealing information are usually grounds for denial of admission. Just tell the truth and then it's just your burden to prove good character. (if there is an investigation). (3) The information you disclose on your bar application must be consistent with the information you disclosed on your law school application. If your information is incomplete on the law school application, consider making a belated disclosure to the law school of this information in addition to adding it to your bar application. If the information is serious (homicide conviction), then the law school may take disciplinary action.

A. Professional Discipline and the Disciplinary System 1. In general

a) In most states, the highest court runs the disciplinary system. An independent office set up by the court uses paid staff attorneys to investigate and prosecute charges. Some of these offices are part of the state bar associations, but most are independent of them. b) The disciplinary system is not perfect, as the offices dismiss many meritorious complaints and most aggrieved clients don't know anything about it. Also, it can take years to get through the lawyer disciplinary system. c) Grounds for discipline? A lawyer may be disciplined for conduct that has nothing to do with the practice of law. It does not matter whether the violation of the ethics code occurred in course of law practice or not. Many lawyers who have held high public office have been disciplined for misconduct that related to their performance of their duties as public servants (i.e. Nixon) - agencies tend to be vigilant in prosecuting misconduct by lawyers in highly visible positions of public trust. d) What about committing a crime? A lawyer may be disciplined for the commission of any criminal act that violates an ethical rule or that reflects dishonesty, untrustworthiness or lack of fitness to practice. The predicate for discipline is the commission, not charge or conviction, of a criminal act. e) Vicarious discipline? Lawyers may be disciplined for violating a rule by inducing or assisting another person (or employee) to do something that violates the rules if done by a lawyer. f) What about interstate violations? A lawyer may be disciplined for violation of the rules regardless of whether the violation occurs in the state in which the lawyer is admitted. While most ethics codes subject only lawyers admitted to practice in that state to discipline for violation of the rules, some states allow discipline of any lawyer who violates a rule of the jurisdiction. If discipline is imposed in one state, and the lawyer has been admitted in various states, then the lawyer must report this to the other states - a lawyer sanctioned in one jurisdiction often receives the same sanction in any other jurisdictions where the lawyer is admitted.

A. Application to the Bar and Admission to Practice 1. In general

a) In most states, the rules for admission to the bar are established by the highest court of each state. b) The basic requirements for admission to the bar include: (1) Graduation from an accredited undergraduate college; (2) Graduation from a law school that meets the state's educations standards (usually ABA-accredited); (3) Submission of an application for admission to the bar; (4) Obtaining a passing score on the bar exam administered by the state; and (5) A finding that the applicant is of good moral character and is fit for the practice of law.

1. Materials not created or collected in anticipation of litigation

a) In order to be protected, the materials created or collected must be done so in preparation for litigation. It is only the lawyer's need to use the information in litigation that creates a degree of protection. b) If certain information would have been collected routinely, but was also collected because litigation was anticipated, most courts will deny protection to the information. c) Restatement: The work product doctrine applies in rulemaking proceedings as well as in litigation as long as the rulemaking is sufficiently adversarial. d) What if a client gives a lawyer a set of documents relevant to impending litigation? These documents will usually not be protected, unless the lawyer can demonstrate that their selection and compilation of the documents reflects their litigation strategy.

1. La.'s Disciplinary Procedure

a) La.'s disciplinary procedures are very similar to that of other states. b) The reading would suggest that it is not likely that someone who engages in egregious behavior will necessarily get caught. This may not be true in La. c) Office of Disciplinary Counsel: La. has a whole office of staff attorneys who investigate and prosecute La. lawyers. (1) [the hearing committee first level of trouble is made up of two attorneys and one public member] (a) 2 members of bar and one non lawyer (b) Why the non lawyer? Do not want all lawyers to regulate themselves. Might be too generous to the lawyers. (2) Complaint is filed, the office will screen it to determine if it should be investigated. Many cases are screened out this way. (3) If not screened out, the ODC will send a letter to the lawyer, stating that the complaint states a claim and that the ODC requires a response. (4) If the lawyer does not respond, the allegations will be treated as true. (hearing) (5) If the ODC is not satisfied with the response (hearing), then there will be a hearing. It is not a good idea to be your own lawyer in this instance. Also good idea to show up. (6) A decision will come out of the hearing (no jury and prosecutors is part of judiciary), and discipline will be recommended. If don't like it appeal to Disciplinary board. (7) You will go to the disciplinary board, who will then review what happened and recommend what else to do they will issue an opinion and if further appeals it will then go to the LASC. (8) At the LASC level, there will be oral argument just like for any typical litigation. d) La. is 4th in the country as far as lawyers that get disbarred through this disciplinary system. e) Possible sanctions in La.: (Know the names of them and the order they come). Section 10 (A) of Rule 19 in LA. (1) Permanent disbarment (only in La.) (2) Disbarment (can reapply after 5 years and have burden to prove character seems easier than Rojas b/c at least once in you can re-apply after 5 years) (3) Suspension by the LASC (max. 3 years - key can't practice) (4) Probation (not disbarred, but just monitored for max 2 years and get public reprimand - key is can still practice) (5) Public reprimand (opinion published) (6) Private admonition (warning - nothing public) Section 10 (B) of Rule 19 in LA. - Informal disciplinary measures - Written Conditions can be attached to an admonition or a reprimand. Failure to comply with the conditions shall be grounds for reconsideration of the matter and prosecution of formal charges against respondent. (a) Deal or plea bargain to resolve (b) May impose some conditions on the sanction (fine, AA, etc.). f) Factors to consider in imposing sanctions Section 10 (C) of Rule 19 (1) Whether the lawyer has violated a duty owed to a client, to the public, to the legal system, or to the profession; (2) Whether the lawyer acted intentionally, knowingly, or negligently (3) The amount of the actual or potential injury caused by the lawyer's misconduct; and (4) The existence of any aggravating or mitigating factors.

institutions that regulate lawyers clients

a) Large corporations and government agencies are major consumers of legal services and have a great deal of bargaining power. b) Law firms that work for these large institutions must usually agree to comply with certain policies on billing and other things as a condition of employment.

1. Upjohn Co. v. United States (1981)

a) Members of the Upjohn Corp. were bribing foreign officials. The company set up an internal investigation (questionnaires). b) Who has the attorney-client privilege where the client is the corporate entity? The corporation has the privilege, and the employees and officers theoretically do not. c) Before this case, the control group test was used, where the people who have control over the corporation will fall within the scope of the privilege. This only included fairly high level corporate officers. d) This case adopts the subject matter test, where the privilege is extended to communications with any employee or agent, so long as the communication related to the subject matter of the representation. e) Federal common law used "subject matter test" but not for the states. So now we do have protection to Employee if he is giving information to the corporation. Notwithstanding it can still be waived if the corporation waive the privilege it will expose employee to the bribery claim. (1) So the attorney client privilege is for the corporation but the tests determines whos conversations are privileged. f) The "privileged persons" aspect of atty-client privilege elements. g) Held: In federal proceedings applying federal law, corporate entities could claim attorney-client privilege, and the scope of the privilege should depend on the subject matter of the communication, not on who was doing the communicating. The corporate attorney-client privilege in federal proceedings was expenses to communications between lawyers and low-level employees, as long as the purpose of the communication is for the company's legal affairs. h) What about in state courts? They are not bound by this decision. As of 1997, 14 states had adopted Upjohn or some variant of it. 8 states still had the control group test, and 28 states had not decided. i) What about other business forms? The Restatement urges that neither logic nor principle supports limiting the organizational privilege to the corporate form. (1) Also the government does not care if the government would be more convinced by taking the firm's work. It would discourage firms from policing themselves. What if the gov is trying to get the corporation to give result to the internal investigation? Gov wants to compel them to turn it over. If the attorney- client privilege than it would stay privileged. What policy downside is this for doing this? It would prevent corporations from conducting investigations. Upside à saving money Most people believe the privilege is dead in corporation context

1. The Responsibility for Ethical Misconduct by Colleagues and Superiors

a) Notes that the rules that follow do not impose firm-wide responsibility, although some states do embrace the "law firm rule." You will generally only be responsible for the conduct of another lawyer where you supervise them or are in a managerial capacity. b) See Rule 5.1:

institutions that regulate lawyers prosecutors

a) Prosecutors have enormous discretion as to whether to file charges against particular D's, and an increasing number of lawyers are indicted and prosecuted for crimes, some of which were committed during the practice of law. b) Events such as Watergate and the banking crisis in the 80s shattered public assumptions that lawyers would never be involved in criminal activity - this made any hesitation prosecutors had about going after lawyers vanish.

1. The Clandestine Videotape a) Client rents out tables and chairs for parties. At a party, a chair breaks and a guest alleges she is permanently hurt. Guest sues client for failing to inspect chairs. Client hears that guest was not as hurt as she alleged, and you hire a private investigator to secretly videotape her. She realizes she is being videotaped, and her attorney asks you for the tape.

a) R. 1.6 cannot be used here as a shield against this evidentiary issue. We want to assert the attorney client privilege. b) The real question here is whether the video tape is "communication" for purposes of the privilege. It appears that the communication is between the opponent and the lawyer's agent. However, it could be argued that the agent is just communicating what he saw to the lawyer through the videotape. If he were to detail what he saw in a letter, it would be privileged, so why not a videotape? S doesn't think the videotape itself was subject to the privilege, as he doesn't buy that it was a communication. c) Would a conversation between a lawyer and his investigator be privileged? YES - communications between a lawyer and his agent will be privileged. Privileged persons is a broader category than just attorney and client - the scope of the privilege widens when we start talking about all the persons that can be subject to it. d) What if the lawyer has a conversation with his client, and the lawyer has been bugged because he's gotten in trouble with the feds? This would probably not defeat the privilege, as long as the parties had a reasonable expectation of confidentiality. This might be problematic where it was the lawyer that installed the listening device. e) What if the client is asked about a privileged discussion by opposing counsel, but the client wants to answer the question? The client is always free to waive the privilege, but you should probably speak with him privately in the hall about keeping his mouth shut. The opposing counsel should give you this courtesy.

1. 1-3 The Little Hearing

a) Recent law school graduate working with solo immigration law practitioner. He assigns the new lawyer to represent a client in a hearing, but she isn't prepared at all. All he does it hands her a treatise. b) See Rule 1.1 and the requirement of competent representation: A lawyer shall provide competent representation to a client. Competent representation requires the legal knowledge, skill, thoroughness and preparation reasonably necessary for the representation. c) What happens if you don't satisfy the obligations of R. 1.1? Would the sanction have been different if she had been practicing more than 2 days? d) Under rules 5.1 and 5.3 e) Back to R. 8.3: Did the other lawyer observing the associate have a duty to report her incompetence? Note that incompetence may fit into the phrase "fitness in other respects" in R. 8.3. f) Under R. 8.3, do you have to report yourself? (1) The phrase, "a lawyer who knows that another lawyer..." implies that you don't have to report yourself. (2) The LASC has implied that reporting yourself to the ODC is a good thing - the court regards the reporting duty as applying to yourself. However, this is inconsistent with the plain meaning of the rule. (3) While there may be no affirmative duty to report yourself, if you do, it might be a mitigating factor considered by the court.

Protection of information if there is a reasonable prospect of harm to the client's interests

a) Restatement of the Law Governing Lawyers (1) The rule prohibits revelation of such information only if there is a reasonable prospect that doing so will adversely affect a material interest of the client or if the client has instructed the lawyer not to use or disclose such information. (2) "Adverse effects" include frustration of client's objectives, material misfortune, disadvantage or other prejudice, financial or physical harm to the client, or personal embarrassment. (3) This rule is considerably narrower than R. 1.6 - there is a lower standard of secrecy and not as much will fall under "confidential client information." It allows lawyers to expose much more information. (4) 1.6 versus Restatement (a) 1.6: Bright line approach (b) Restatement: Distinguishes between information that could harm the client and that that wouldn't. (5) Despite this rule, the LASC seems to follow R. 1.6, just the way it is written in the model rules. (6) How do you know if there is a reasonable prospect of harm to a client's interests? This depends on whether a lawyer of reasonable caution, considering only the client's objectives, would regard use or disclosure in the circumstances as creating an unreasonable risk of adverse effect either to those objectives or to other interests of the client. This is basically a judgment call. 2. Misc. stuff a) Doctors and other professionals are subject to a duty of confidentiality as well. b) This duty will apply whether you are talking to a friend, your spouse or other attorneys that have no involvement in the case. c) The confidentiality duty lasts forever, not just when the case is resolved. d) The policy behind R. 1.6: To allow the free flow of information between attorney and client, so that the client can get the best representation possible. The administration of justice is the societal interest that is furthered, and this is also so that citizens are better served by the legal profession. e) Notice that there is no exception to R. 1.6 for information within the public domain!! Louisiana is a bit softer about information in public domain meaning if in public domain then not confidential under 1.6 but need to be sure understand no exception if it is in the public domain. 3. Bottom line: There is none! 1.6 doesn't really take into account the fact that lawyers are humans that talk to family and friends. Although 1.6 doesn't mention a reasonable prospect of harm test, cmt. 4 acknowledges that at least in some circumstances, a lawyer may talk hypothetically about a case as long as the identity of the client is protected. Keep in mind that La. has not adopted the comments! In general, try to resolve the gray issues in favor of nondisclosure.

1. Procedure for challenging the privilege on the basis of the exception

a) The opposing lawyer must request documents based on guesses about what might exist or what the documents might contain. b) He might argue for an in camera inspection of the correspondence (judge reviews the documents privately to decide whether they are privileged).

(6) to comply with other law or a court order.

a) Some consider these Rules to fall within "other law." b) Recall that many of the rules prohibit an attorney from assisting a client in committing fraudulent or criminal acts. c) It may be possible that in order to comply with these fraud provisions, that disclosure would not only be permitted, but would be mandatory for you to avoid assisting in the fraud or crime. d) It is supported by solid commentary that it is an exception to the 1.6 duty where a failure to disclose would result in a lawyer assisting a client in fraud. e) Whether a law really requires such reporting by lawyers is a matter of interpretation of that other law, not of the rules of professional conduct.

institutions that regulate lawyers the ABA

a) State bar associations are independent of, and not subordinate to, the ABA, although a majority of the ABA House of Delegates is selected by state and local bar associations. b) Although the ABA is the primary drafter of lawyer ethics codes, the ABA has only limited governmental authority. The Model Rules of Professional Conduct have no legal force unless adopted by the state's highest court. c) State courts are under no duty to consider a rule just because it was proposed by the ABA or analyzed by a state bar association. d) Note: The ABA adopted its first set of Canons of Ethics in 1908, that was largely based on the Alabama ethics code derived from lectures by Sharswood. The adoption of the model code didn't happen until the 1960's, and then the current model rules were adopted in 1983.

3-8 An Investment Project

a) Suppose you purchase a piece of property with knowledge that your client plans to build a shopping center on the parcel next to it. You would violate 1.8(b) if you are doing this to stiff your client, since the client has an interest in this property. Sometimes, mere disclosure isn't considered use. "insider trading" for example is use. If it's not to the client's disadvantage, then it doesn't violate 1.8b. Although the ethics rules might not apply, other rules might. b) What if you tell another client to buy the property? Here, you're not only using the information for the benefit of another, but it's also a disclosure prohibited by R. 1.6. Purchasing the property yourself, you would not violate 1.6. c) What if the client has no interest in the nearby parcel? In this instance, you are using the confidential information provided by your client to your advantage, but not to your client's disadvantage, since the client doesn't care about this piece of land. d) Rule 1.8 Conflicts of Interest: Current Clients: Specific Rules (1) (b) (2) Cannot do anything against your client's interests (3) If you have their consent prob ok à but what if they don't? (a) You only need consent if the rule engages (4) If they do not want the property than the rule probably does not engage. e) Agent using information for the benefit of the agent without harming the principle. (1) General rule is that the agent has a duty to account for the profits made to the principle. f) Use the information v. disclosing the information g) What if you know about the advantage and you know that one of your other clients is looking for real estate investments. You also know that the original client does not want to buy it. So can you go to the investor client and tip them off? (1) What is the problem? (2) You are not using the information of your benefit OR the disadvantage of the client (3) So the rule does not engage. (4) What about Rule 1.6? You are disclosing the original client's information and there is no exception. This is a disclosure and not a use.

1. Express waiver by client

a) The attorney-client privilege can be expressly waived by the client. b) May also be waived by the client's lawyer if the waiver has been authorized by the client. c) Waiver requires a voluntary act by the client or by an authorized agent of the client. d) You should likely tell the client everything is confidential. So they understand.

1. Communications in confidence

a) The client must reasonably believe that the communication is confidential. b) No privilege will attach to communication that occurs in the presence of other people. c) If the lawyer reveals privileged information to a friend, then this will waive the privilege, regardless of whether the conversation took place in private or public. It is the disclosure itself that waives the privilege. d) Ease dropper. Usual standard of reasonable expectation of privacy so don't think ease dropper you don't have threats of privilege. Lose privilege if client tells and if it's discussed in public. Plant watering people and listening to conversation then blown the privilege b/c don't have expectation. Fair number of cases with the cell phone. These conversations can be intercepted so no reasonable expectation of privacy so don't discuss on phone b/c privilege will be defeated. Now technology is so advanced and area problematic and no recent cases.

1. Communication for the purpose of seeking legal assistance

a) The communication will only be privileged if the purpose for seeking it was for legal advice. b) If the client is asking for business advice, the conversion is not privileged. If a conversation contains elements of both legal and some other kind of advice, then only the part relating to the legal advice will be privileged. This usually comes up in the context of in house counsel. c) A promise or exchange of money isn't necessary to create a lawyer-client relationship, so the communication can be for the purpose of seeking legal advice even if the lawyer isn't billing the time. d) Note that the privilege protects not only what the client says to the lawyer, but also what the lawyer says to the client. Some cases have held that lawyer communication is protected only if it includes information about client communications. The Restatement, however, favors a broader coverage of confidential communication by lawyers. e) If a client tells a lawyer some factual information during a privileged conversation, the client is NOT protected from being compelled to testify about these facts. Only the communication is protected. f) The privilege will protect documents as well as conversations, as long as the lawyer-client communications are for the purpose of obtaining legal advice.

institutions that regulate lawyers malpractice insurers

a) The conditions for providing insurance set a body of private law that governs lawyers who contract with these companies. b) Loss prevention structures: Insurers may required to prevent the firm from being held liable. c) The guidance to and supervision of law firms by insurers is an important, though nongovernmental, form of regulation.

work product doctrine 1. Protection of lawyer's "mental impressions"

a) The doctrine gives stronger protection to work product that reveals the lawyer's thoughts, strategies or mental impressions. b) This would include the lawyer's Opinion work product: own notes of his opinions, theories, observations or feelings. c) Some courts have held that even these documents can be obtained by a showing of "extraordinary circumstances." This is a much greater standard than undue hardship - the opposing counsel would have to demonstrate an extreme need for it, but S doesn't know what kind of showing they would have to make.

1. Protection of work product, not underlying information

a) The information contained in the protected documents can usually be obtained from the original witnesses or sources. b) The doctrine prevents freeloading on an opponent's work, but it does not enable the opponent to close off a lawyer's sources of information by getting there first. 2. If information is protected by both the attorney-client privilege and the work-product doctrine, then pick the shield that will give you the most protection. Note that there are exceptions for each, and these exceptions are different.

1. Waiver by revealing privileged communication to a nonprivileged person

a) The privilege will be waived if the client or the lawyer reveals the privileged information to a non-privileged person (lawyer would need the authority from the client). The privilege belongs to the client's so unless the lawyer has express, implied or apparent authority to reveal the information this revelation would not effect a waiver of the privilege. b) What if a lawyer represents two clients jointly? Both clients are privileged, and therefore communication with one joint client in front of the other joint client will not waive the privilege. If these two clients hire the layer separately and the lawyer would like to privilege to apply to conversations where both are present the lawyer should obtain their agreement to participate in a "common" representation in which confidential information is shared among the three of them. That will preserve the privilege.

Waiver by putting privileged communication into issue

a) The privilege will be waived where the client puts the privileged communication into issue in a case. b) For example, if a client sues a lawyer for malpractice and asserts that the lawyer gave her certain incorrect advice, the lawyer may reveal the details of the relevant conversations for the purpose of self-defense.

A. Institutions that Regulate Lawyers 1. The highest state courts

a) The responsibility of self-regulation: The legal profession is largely self-regulated, and in most states, it is the highest court of the state rather than the legislature that is responsible for adopting the rules of professional conduct. Most of those involved in writing the ethical rules are licensed lawyers. As a result, the rules governing lawyers are more protective of lawyers and impose less regulatory constraint than they would if state legislatures wrote them. A lawyer who practices in two states is expected to know the rules of both jurisdictions. The inherent powers doctrine: Unlike typical law, courts make the ethical and procedural rules, implement them, interpret them, enforce them and hear challenges to the validity of them. In some states, the state constitution expressly assigns the courts authority to regulate the conduct of lawyers. In others, courts apply the inherent powers doctrine to say that regulating the conduct of lawyers is a matter of inherent authority because the courts need the authority to govern the conduct of those appearing before them. In some states, courts can actually strike down legislation that conflicts with professional rules (this exclusive authority to govern the conduct of lawyers is called the negative inherent powers doctrine).

1. The Law Governing Lawyers Legal Malpractice, Breach of K and Breach of Fiduciary Duty

a) The tort of legal malpractice may involve either a claim of negligence or intentional misconduct. This may be brought by either a client or a non-client third party. b) Under the Restatement, the elements include: (1) The lawyer owed a duty to the P, (2) The lawyer failed to exercise the competence and diligence normally exercised by lawyers in similar circumstances, and (3) The breach of duty caused harm to the P. c) A lawyer can be disciplined, sued for malpractice and criminally prosecuted all for one act of misconduct. d) A claim for breach of fiduciary duty is different from a malpractice claim in that it is a separate cause of action grounded in the common law of fiduciary duty, which applies to many other persons than just lawyers. A malpractice claim usually includes a claim for breach of fiduciary duty.

A. Exceptions to the Duty to Protect Confidences 1. Revelation of past criminal conduct

a) There is a broad consensus in the legal profession that information about most PAST criminal activity by clients should be kept confidential. The lawyer in this instance can no longer prevent the crime from happening by revealing the information, and society benefits by the fair administration of justice and by clients having unobstructed access to counsel. b) 3-3 The Missing Persons, Scene 1 (Belge case) (1) Facts: Garrow admits to you that he committed the murder he's being prosecuted for. He also confesses that he killed some other kids, and told you where he hid the bodies. (2) The actual attorneys in the case went out and found the bodies, to verify Garrow's story. Knowing the location of the bodies gave Garrow's attorneys leverage in the plea bargain. Garrow later sued his attorneys alleging ineffective assistance of counsel - the court held in favor of the attorneys, on grounds of privileged communication (3) What if Garrow had wished to keep the location of the bodies secret? R. 1.6 may provide an exception to disclosure. Just because the information was going to be used as leverage in a plea bargain does not mean that it was impliedly authorized by the client. (4) What is "informed consent"? See R. 1.0(e): The agreement by a person to a proposed course of conduct after the lawyer has communicated adequate information and explanation about the material risks of and reasonably available alternatives to the proposed course of conduct. (5) The rule permits disclosure of client confidences to save a life or prevent substantial bodily harm. However, you are not compelled to do so - there is no obligation to disclose. c) 2-4 The Missing Persons, Scene 2 (1) What if the parents of the missing kids come to your office, asking if you know whether their kids are alive or dead? (2) Remember that you are permitted to disclose to prevent harm or death. If the harm/death has already happened, then there is nothing left to prevent, and therefore, you cannot reveal anything under 1.6. d) 2-5 The Missing Persons, Scene 3 (1) What if, at trial, Garrow basically admits he had been stalking co-counsel's daughter, he is convicted and put in jail, and then later escapes? As his former counsel, you know where he hides when he escapes. (2) Revelation of confidences might be appropriate in this case to prevent future bodily harm. You know this guy likes to kill kids - substantial bodily harm is likely to result unless you tell the police where he's hiding. In the actual case, counsel told the police where Garrow had hidden in the past and the police found him in that same spot. (3) LA rule used to be consistent with the ABA rule. The old rule said you could make the disclosure to prevent a criminal act that could likely result substantial bodily harm or death.

institutions that regulate lawyers federal and state trial courts

a) These courts play an important role by: (1) Setting rules for the conduct of lawyers in litigation; (2) Sanctioning lawyers that violate these rules; and, (3) Hearing/deciding motions to disqualify lawyers with conflicts of interest that preclude their representation of particular clients. b) If a judge becomes aware of lawyer misconduct in a matter before the court, the judge can impose sanctions under federal/state civil procedure rules. c) Judges must report the misconduct to the lawyer disciplinary agency if it violates an ethical rule that raises a substantial question as to that lawyer's honesty, trustworthiness or fitness as a lawyer in other respects. d) Some federal courts adopt their own ethical rules, but most adopt the same rules that are in force in the states where they sit.

1. Origins of the work product rule

a) This was first recognized in Hickman v. Taylor (1947). It is a relatively new CML rule that protects from compelled disclosure a lawyer's private notes and mental impressions.

institutions that regulate lawyers legislatures

a) Through other kinds of legislation (criminal, banking, securities, etc.), Congress and state legislatures govern lawyers. Most states have statutes that make it a misdemeanor to engage in the unauthorized practice of laws.

1. Protections for Lawyers that Blow the Whistle

a) Typically, if you complain to the disciplinary authorities about your supervisor, then you're likely to get fired under the employment at will theory. Law firms, fearing the loss of clients, bad publicity or higher malpractice insurance premiums, have often been hard on any whistle-blowers. b) A lawyer who is told to do something that the lawyer thinks is unethical has several options: (1) Accept the directions of the superior (2) Argue with the superior (3) Discuss the problem with another superior (4) Do more research or investigation to try to clarify the problem (5) Ask to be relieved from work on the matter (6) Resign (or be fired) from employment c) Wieder v. Skala (NY 1992) (1) The associate blew the whistle on a partner at his firm. (2) Rule: Retaliatory discharge for reporting a violation of the Rules is not allowed. When a firm discharges a lawyer for making such a report, the discharged lawyer can sue for breach of K. It is an implied term of the employment K that you won't be discharged for following the Rules. This is an exception to employment at will. (3) Jacobson (IL) declined to follow the NY court. (4) While this protection seems like a good thing, it may still not help in re: to employment opportunities. d) Kelly v. Hunton & Williams (Wolas case) (E.D.N.Y. 1999) (1) Facts: Kelly and 2 other associates realized that Wolas was billing for hours that he wasn't working (i.e. committing fraud). Instead of working, he was defrauding investors and clients. Some of the partners the associates went to at the firm were investing in this fraudulent scheme. There was a "hearing" within the firm, and Wolas was found not guilty - the associates were told that they had no obligation to take their allegations any further. Kelly was forced to resign (allegedly for "poor performance"), and this action was brought against his former employer, alleging that the firm breached implied contractual obligations owed to him when it terminated his employment with the firm. Because Kelly was not admitted to the bar at the time of the occurrences, H&W argued that Kelly was essentially a clerk, not bound by the disciplinary rules. (2) Wolas' behavior clearly violated R. 8.4(c): Conduct involving fraud, etc. Kelly had a duty to report this. (3) Court's response: (a) "While associates not yet admitted to the bar are not officially attorneys, it would be anomalous to permit these associates to ignore unethical behavior that admitted associates are required to report." (b) Court relied on Weider as an exception to the employment-at-will doctrine (c) "If a law firm fires an associate in retaliation for reporting a lawyer's misconduct to the firm, its action is inherently coercive and necessarily implies an effort to impede post-termination reporting to the Disciplinary Committee. Thus, a cause of action is available under Wieder." (d) The court recognized that Wieder was a narrow exception to at-will employment, but that it was not going to give it a crabbed construction. (e) Summary judgment was denied for H&W, and if the jury resolved the disputed issues of fact in Kelly's favor, it may properly conclude that H&W breached its employment K with him. (f) The 8.3 duty is part of the employment agreement, therefore you can't fire someone for blowing the whistle. (4) La. law on this public policy exception to the employment at will: See R.S. 23:967 ==> Some protection from retaliatory discharge for providing information about a violation of state law (the issue then becomes whether the ethical rules are "state law"). (a) So there's a whistleblower protection in LA

A. Exceptions to the Duty to Protect Confidences 1. The risk of future injury or death

a) Under this exception, it does not matter whether the possible harm will be perpetrated by the client or another person - what matters is the magnitude of the harm and the probability that it will happen. b) Cmt. 6: Harm will be reasonably certain to occur if: (1) It will be suffered imminently or (2) If there is a present and substantial threat that a person will suffer the harm at a later date if the attorney doesn't make the disclosure. c) The contemplated harm need not be criminal in order to fall under this exception. d) Spaulding v. Zimmerman (Minn. 1962) (1) Facts: There was a car crash at an intersection with no stop signs. Spaulding was riding in the Zimmerman car, and was severely injured. Spaulding's Dr. performed an evaluation on him, and said he'd be ok. The Zimmerman's Dr. also performed an evaluation, and the Dr. found an aorta aneurism. (2) Issue: Does the D's dr. disclose this to the victim? (3) The lawyers in this case decided not to divulge the information out of fear that the settlement amount would increase. The surgery that Spaulding eventually had to have costed more than what he was paid in settlement. (4) Here, the court threw out the settlement. While there was no requirement to disclose the aneurism, the settlement obviously did not take this fact into account. (5) The earlier version of 1.6 would not have permitted disclosure, so (6) The lawyers here could have disclosed this information even if the D's had forbade it, since it would have been to save a life. (a) 1.6 didn't include the exceptions we have now. 1st ask client's permission then if they say no use the exception. But remember even in LA this is a MAY under 1.6 but need to read with 4.1 b/c that is a MUST. (7) Suppose clients didn't want you to disclose it (even if they knew the severity of the medical condition)? Now, 1.6b2 lets us do it, but without that rule, e) 2-6: Your Dinner with Anna, Scene 3 (1) Facts: You're working on a case for Diet Kola, and tests have shown that it causes birth defects. You have a friend that only drinks Diet Kola and milk because she's pregnant. Do you tell your friend? (2) It's a stretch, but is may fall into the preventing harm/death exception to confidentiality in R. 1.6. However, if the tests are speculative, then this might not be enough under 1.6(b)(1). (3) While you might want to violate the rule to help your friend and her unborn child, this information likely would not fall into an exception of R. 1.6. (4) Under the Restatement standard, it is possible that you could discuss the case with your friend if you didn't reveal any information from which the client's identity could be ascertained. Even if the identity could be ascertained, the information could be revealed if there was no "reasonable prospect" that the discussion would adversely affect the client's interests. (a) Under 1.6 we don't care where she got the information from if it's an article even if you didn't find out from the client themselves but that does not matter because its "anything that pertains to the representation" regardless where you got the information from. If see article can't give it to her if adversary. (b) What could you do? This is 1.6 information and can't reveal it. This is a lot like Spaulding because do you help friend and break rule or don't help like Spaulding and let her take the risk.

institutions that regulate lawyers state and local bar associations

a) Usually organized as a private nonprofit organization b) May have some governmental functions as delegated from the state's highest court - a state bar that accepts these functions is called an integrated or unified bar rather than a voluntary bar. In unified state bars, membership is mandatory. c) In addition to state bar organizations, most states have a variety of other voluntary bar associations that lawyers are not required to join.

Waiver by inaction

a) What if a lawyer does not realize that a question calls for privileged information, he does not object, and the client answers? b) This will be a waiver by inaction and no appeal will be able to reverse the process. c) It's the clients privilege. Document production and responds to discovery request and in the documents produced there are letters between the lawyer and client by mistake. The letters are about representation and subject to the attorney/client privilege. This is mal practice (you assert privilege in discovery we don't care if they are relevant). Should it be waived - it's the clients privilege but sometimes with discovery they balance to see if waive. Here it was the lawyers conduct. (1) If at cocktail party lawyer isn't going to waive attorney client privilege - client likely didn't give the "authority" to the lawyer to discus as party. Also not his agent at the party. (2) Deposition - waived by client because he said it and that is bad get mal practice ready. (3) Discovery - balance test.

A. The Privilege for Corporations In general

a) While corporate clients may invoke the privilege just like individual clients, the scope of the privilege in this context has been controversial. b) For a long time, only communications between the lawyer and the "control group" (upper management) were privileged. This was changed in Upjohn.


Conjuntos de estudio relacionados

Set theory, permutations and combinations

View Set

11th Physics | Quiz: Acceleration

View Set

Microeconomics chapter 9 - Businesses and the Costs of Production

View Set

Discovering the Universe Chpt. 2

View Set

Surah Naziaat Tafseer Test (questions)

View Set

Lesson 2- Understanding Virtualization and Cloud Computing

View Set

Cisco CCENT/CCNA ICND1 100-101 Chapter 3

View Set